CAT-Advanced-Material-VA-RC-Reasoning

October 8, 2017 | Author: jeevan820 | Category: Television Advertisement, Word, Science, Sentence (Linguistics), Semantics
Share Embed Donate


Short Description

Download CAT-Advanced-Material-VA-RC-Reasoning...

Description

1

Career Avenues

JUMBLED SENTENCES

2

Career Avenues

1. Paragraph anagrams Rearrangement of sentences or paragraph anagrams is the most commonly seen question in the CAT and in most of other MBA admission tests. It interesting to know that this question doesn’t examine any previously learnt concepts, neither is it based on any particular logic. What it tests is common sense and speed. Extensive practice is the key. However, we have a few strategies for you. Study them and use them in the exercise that follow and you’ll be comfortable with them. Paragraph jumbles questions are presented in different forms. a) Four/ five sentences either numbered 1,2,3,4,5 or lettered a, b, c, d, e are given in a jumbled form. The sentences are followed by choices of four possible combinations of arrangements of these sentences. You have to choose the most logical order of arrangement of these sentences to form a meaningful, coherent paragraph. E.g. A. The managers are themselves parts of various hierarchies of managerial influence, coupled to one another by explicit and implicit codes of behaviour and information network. B. Management science and MIS have introduced new elements of complexity into the manager’s world. C. Management deals with complex social systems joined by physical and informational mechanisms to other social systems that are the responsibility of other managers. D. If this seems to make the social system appear complex, it is so intended. 1. DCAB 2. ADCB 3. DBAC 4. CADB b) A set of six sentences is given, in which the first and the sixth sentences are fixed at their respective positions. The four sentences that falls between the first and the sixth sentences are presented in a jumbled form. You are asked to choose the logical order of sentences from the choices. E.g. 1. A. B. C. D. 6.

If the phenomenon of down gradation is understood and accepted, brands have to continuously seek options that offer better value to consumers. Therefore, given an option, the consumer was willing to be satisfied by greater value from trusted brands rather than downgrade. Tea in polypacks doubled their volume share between 1989 and 1991 in Bombay. Biscuits in family packs witnessed phenomenal growth of such brands even during periods of inflation. The onset of the down gradation phenomenon seems to be a good time to launch new products, or better still, create new sub-categories. For instance, the more expensive gel toothpaste category upped its share in Bombay by six percent at the expense of popular and economy brands. 1. BCAD 2. CDAB 3. ADBC 4. DCBA

Strategies for sentence anagrams Strategy 1 Avoid reading the sentences closely; that is not required and is a waste of time. Inspect the choices; if each of the choices begins with different letters, identifying the sentence to begin the sequence leads to the correct answer. The following example illustrates this.

3

Career Avenues

1.

Small companies that compete effectively tend to grow, and growth brings increasing complexity and specialization in each function. A. Technologists talk about processes, new materials, and worry about prototype results and technical problems; as regards language and interests, they have little in common with marketing. B. As the company grows it tends increasingly to fragment into separate functional islands, each trying to solve its own problems, each using its own special language and having its own priorities. C. Marketing people, for instance, talk about market segmentation, market growth, promotions and product image, and worry about changes in share. D. Production people talk and worry about industrial relations, people arriving on time, and plant and equipment breakdown and delays. 6. They in their turn have little in common with either marketing or the technologists who, they consider, live in ivory towers. 1. DACB 2. BCAD 3. ACBD 4. CDAB

In the above question, the choices begin with different letters. The best strategy then would be to inspect sentence 1 and to identify the sentence that logically follows 1 Sentence 1 deals with growth of companies and sentence B begins with the continuity phrase, as the company grows. Consequently, B has to follow 1 and the answer therefore is choice 2. You require minimum steps in arriving at the answer in such questions. Strategy 2 In choices that are closely related, identify a mandatory pair of sentences (two sentences that should form a sequence). A. B. C. D. E.

This linking of politics and music is of course ancient and even Aristotle in his book ‘Politics’ said, “We may compare the best form of government to the most harmonious piece of music.” Mixing metaphors, Mr. Clinton referred to the symphony so central to western music and said, “It is time both nations heard the musical compositions of each other and understood each other better.” The oligarchic and despotic to the more violent tunes; and the democratic to the soft and gentler airs. The US President was confident that if governments and people of the two democracies made a determined bid to understand each other’s perspectives scripts, they could create new symphonies. Such attempts to secure symphony would be so much better than the individual bids for virtuoso improvisations and adherence to classical compositions. 1. BACDE 2. DCAEB 3. DECAB 4. BDEAC

In the above question, observe that the choices are closely related. Two choices begin with D and the other two with B. In such cases avoid working from the first sentence; instead inspect the sentences quickly and identify a particular pair of sentences that are mandatory. In the above case EA is a mandatory sequence. Only choice 4 satisfies this condition, and therefore the most logical sequence. Strategy 3 Look out for quick clues such as pronouns, conjunctions etc. in the beginning of sentences to help you establish a sequence. E.g. 1. A. B. C. D.

As a senior economist associated with the reforms programme used to say, a sure indication that the new economic policy was succeeding would be when foreign investors start trekking to Delhi in the sweltering heat of May and June. If nothing else, India Power ‘94 came as a welcome break to the power ministry; most of the participants had complimentary things to say about government policy. And there were would-be entrants like the Hong Kong-based China Light and Power Company who had turned up armed with a fax from the power ministry listing available projects in India. Were that the only yardstick, then judging by the attendance at India Power ‘94, at least the reforms in the power sector could be declared a success. There were the familiar names - Enron, AES, Cogentrix, and Spectrum Technologies.

4

Career Avenues

6.

This was in striking contrast to the adverse comments the ministry had been attracting so far. 1. ADBC 2. BDCA 3. CDBA 4. DBCA

In the above question, a smart test taker will quickly identify the phrase ‘ were that the only yardstick” in sentence C. This is a thought continuity of the idea in sentence 1 “ foreign investors trekking to Delhi… …”. Consequently C has to follow 1; eliminate choices 1, 2, and 4.Choice 3 is the correct answer. Strategy 4 In six sentences variety, work backward from sentence 6 if there is no obvious clue to work from sentence 1; many a time we overlook the hints that may be valuable Let’s take a look at such an example. 1.

A recent advertisement of Premier Instruments and Controls Ltd., a leading manufacturer of dashboard instruments, in a financial daily, summed it all up. A. The fact is that executives from companies, ranging from Daimler Benz to General Motors, have been scouring the Indian countryside looking for suppliers of cheap components for products made in their European and American plants. B. It obviously does not even have the time to make the investments required to set up a new plant. C. Today, most Indian automobile component manufactures cannot produce enough to meet demand both domestic and international. D. The company was soliciting spare capacities for the supply of intricate machines and sheet-metal components. 6. While some of them do find good deals, many have had to go back empty-handed. 1. ADCB 2. CBAD 3. BACD 4. DBCA

In the question given above, see that “them’ in sentence 6 refers to ‘executives’ referred to in sentence A. i.e. A precedes 6; choice 4 is the answer. Now use these strategies and answer the following exercise. Exercise Questions : 60

Time : 30 minutes

Directions for questions 1–20: Each question has six sentences which when properly sequenced form a coherent paragraph. While sentences 1 and 6 are fixed in their respective positions, the four sentences in between are jumbled. Choose the most logical order of sentences from among the four given choices to construct a paragraph. 1. 1. The psychological obstacles to the restoration of the international association of scientific workers are still too formidable to overcome. a. They can do this by keeping in touch with like-minded people all over the world. b. Men of this kind can aid in the great work of restoring the international societies to health. c. They can also do this by steadfastly championing the international cause in their own spheres. d. This is because there is a minority whose ideas and feelings are of a more comprehensive kind. 6. Success on a large scale will take time but it will undoubtedly come. 1. cbad 2. dbac 3. bdca 4. bacd

5

Career Avenues

2.

1. a. b. c. d. 6.

As long as the artist is an employed workman he must compromise. At the same time he should not sacrifice his own integrity in doing so. It is a dangerous holding of the balance between two sets of forces. He must never lose touch with life and its requirements. For compromise of that kind is not a concession to a lower order of things. The artist, like a chisel, serves a master, but in doing so he gives his master something he never bargained for. 1. cadb 2. abcd 3. dcba 4. badc

3.

1.

The tragedy of the present struggle is that the ends for which people are striving – food, work, security and freedom – are gifts of science. If people could only understand a little of this possibility in which science offers. But instead we have danger and bloodshed, want and misery. These are gifts which science has put within the reach of all. The resources, knowledge and the ability to build a new world are there. People would become more reasonably impatient of their present state and more capable of changing it. 1. cdba 2. bcda 3. dacb 4. dcab

a. b. c. d. 6.

4.

1. a. b. c. d. 6.

The country abounds in the finest natural productions. But the zealous explorer may come upon its gentler beauties in an incredibly short time. Its pastures and cattle are admirable. The more majestic parts of it are at a distance. And the deer are to be met with, in the course of a half a day journey. And the traveler is accompanied, whenever he goes, with music of the singing birds. 1. acdb 3. dcba

5.

1. a. b. c. d. 6.

1. a. b. c. d. 6.

cabd cadb

Ideas of beauty are among the noblest, which can be presented to the human mind. And which, to the rightly perceiving mind does not present a greater number of beautiful than of deformed parts. Invariably it exacts and purifies it according to their degree. It would appear that we are intended by the creator to be constantly under their influence. Because there is not one single object in nature which is not capable of conveying them. As there is scarcely anything in pure, un-diseased nature, like positive deforming but only degrees of beauty. 1. abdc 3. cabd

6.

2. 4.

2. 4.

dcba bcda

Blind courage is an animal virtue. It is indispensable in a world full of dangers and evils. Such animal courage seems therefore beautiful rather than cruel or desperate. And being the lowest and most instinctive of virtues it is the one most widely admired. A certain insensibility and dash are requisite to skirt the precipice without vertigo. But it ceases to be when the love of danger begins to lead men into evils, which are unnecessary to face. 1. adbc 3. bdac

2. 4.

abcd dcab

6

7.

Career Avenues

1. a. b. c. d. 6.

Great books do not spring from something accidental in the men who wrote them. And the expression of the life itself of the authors. It does not succeed until it becomes the vehicle of the vital. And literature cannot be said to have served its true purpose until it has been translated into the actual life of him who reads. They are effervescing of their very core. Progress is the gradual result of the unending battle between reason and instinct in which the former wins with the help of the most powerful engine – literature. 1. cdab 2. dacb 3. bacd 4. adcb

8.

1. a. b. c. d. 6.

Man has so thoroughly eliminated or subdued all serious competitors in the living world. Because of this attitude, his indifference to certain matters can be understood. In fact he thinks of himself as the ruler of these forces. He thinks of himself as akin to the mighty forces of the physical world. He also thinks that his evolution is a movement toward that end. Yet because of this indifference and despite his triumphs he has become the unhappiest species on earth. 1. cadb 2. adcb 3. bcad 4. cdba

9.

1. a. b.

Mass murder of men by men is strictly man' s own invention. And it has no value for the human species at large. Unlike the inter-specific warfare of all other species, it prevents rather than preserves equilibrium of antagonistic forces. It is a cultured disease rather than a biological necessity. It has however inestimable value for dictators; profiteers and fools. Through them mankind has been able to make its unique contribution to the social economy of the living world. 1. bcad 2. abdc 3. bcda 4. cdab

c. d. 6.

10.

1. a. b. c. d. 6.

11.

1. a. b. c. d. 6.

People to day are indifferent to the aesthetic products of the past. Few working class people enjoy culture. That is the position of both the industrial magnate, trade unionist or communist. They are suspicious of them; decline to receive them, until they have been disinfected in Russia. In England, still the abode of private enterprises, indifference predominates. As a rule I am afraid to bore them with it lest I lose their acquaintance. 1. abdc 2. dabc 3. bacd 4. bcda In its labor relations, Britain has, on the whole been more democratic than the USA. The military power has not been so much used to overcome labour. They have not been turned into miniature civil wars. In UK, the strikes have never been moved by such violence as in US. There has been no suffering from illegal and extra legal organizations. Collective bargaining and political trade unionism are among the useful machinery by which British democracy has expressed itself. 1. cbad 2. adbc 3. acbd 4. acdb

7

Career Avenues

12.

1. a. b. c. d. 6.

The share market has been fluctuating with a fairly predictable degree of regularity. They tend to fall before the markets close for the weekend. This is unusual for this time of the year. Share prices tend to rise sharply during the beginning of the week. The phenomena have become so regular over the past few weeks. It has become, therefore, possible to anticipate the market trend a day in advance. 1. badc 2. bcad 3. bdac 4. dabc

13.

1. a. b. c.

One of the basic protections of free speech is the right of free assembly. Hence the constitution guarantee to assemble peacefully and without arms has been provided. In fact, freedom of assembly and freedom of speech go hand in hand. The framers of the constitution knew that the right to peacefully assemble for political activities was important. They also knew that this was essential to make the freedom of speech real. The right to assembly can be restricted only in the interest of public order and the restrictions ought to be reasonable. 1. bcda 2. acdb 3. cabd 4. dabc

d. 6.

14.

1. a. b. c. d. 6.

15.

1. a. b. c. d. 6.

16.

1. a. b. c. d. 6.

All sources of energy upon which industry depends are wasted when they are employed. Already coal had largely been replaced by oil. The industry is expending them at a continually increasing rate. And what is true of oil is equally true of other natural resources. And oil is being used up so fast that East and West alike consider it necessary to their own prosperity to destroy the industry of the other. Everyday, miles of forest are turned into newspaper, but there is no known process by which newspaper can be turned into forest. 1. dbca 2. bcad 3. cdab 4. badc Marx was no Faustian, concerned solely with understanding society. He was centrally concerned with the social problems of the lay group and proletariat. His diagnosis was that their increasing misery engendered endemic class struggle. And there can be little doubt that his work is motivated by an effort to reduce their suffering as he saw it. He was a Promethean who sought to understand it well enough to influence and change it. His prognosis claimed that this would culminate in revolution. 1. dacb 2. acbd 3. bacd 4. cbda Length and time are relative concepts, if two spaceships pass each other with uniform velocity. Observers on each ship will find that the astronauts on the other ship are thinner and moving more slowly. If the relative speed is great enough. they will move like actors in a slow motion movie. All phenomena with periodic movements will seem reduced in speed. This includes tuning forks, watches, heartbeats, vibrating atoms etc. A 6-foot astronaut standing erect in a horizontally moving ship will appear 6 feet tall but will seem thinner in line of travel. 1. dcba 2. abcd 3. cbda 4. adcb

8

17.

Career Avenues

1. a. b. c. d. 6.

The world is very full of people – appallingly full and they are all tumbling over each other. There are two solutions to overcome these dislikes. One is the Nazi solution: If you don't like people, kill them, banish them, segregate them and strut up and down proclaiming you're the salt of the earth. One doesn't like the colour of their skins, or perhaps the shapes of their noses or the way they walk and talk. Most of these people one don’t know and some of them one doesn't like. The other way is much less thrilling, but is one the whole the way of the democracies and is much more preferable. 1. adbc 2. acdb 3. bacd 4. dcab

18.

1. a. b. c. d. 6.

Working of democracy imposes certain responsibilities and obligations on people. If people give a free play to their activities for realising their wishes. It does not mean liberty sans limits. It also places obligations upon men in power. Then it becomes a serious hindrance in the working of democracy. However sometimes the democratic government takes a lenient view of the popular risings because it fears losing support of the people. 1. cbad 2. adcb 3. badc 4. dcab

19.

1. a. b. c. d. 6.

The most vulnerable section of the society from the point of view is the students. Revolutionary and new fledged ideas have a great appeal to them. Agitations may be non-violent methods of protest. They cannot resist the charm of persuasion. They are to be taught that without discipline they cannot get proper education. However if these become violent, the antisocial elements get encouraged and undisciplined and put all proper working out of gear. 1. dcab 2. acbd 3. bcad 4. acdb

20.

1. a.

There is no better way to help one to help him than to bring him to knowledge of himself. There is nothing that will enable him into an awakened knowledge of the powers in his soul than to bring him into the realisation of his oneness with infinite power. These are the great truths of the spirit of infinite wisdom. There is no better way to bring one to knowledge or him than to lead him to knowledge of powers in his soul. We shall find that these same truths lie at the very bottom of the solution of our social situation. And we shall also find that we shall never have a full and permanent solution of it until they are fully recognised and built upon. 1. abcd 2. cabd 3. adcb 4. badc

b. c. d. 6.

Directions for questions 21 – 60: Given below are a number of sentences which when properly sequenced form a coherent paragraph. Each sentence is identified with a number. Choose the most logical order of sentences to construct the paragraph. 21. 1. However, doctors attending on him told us that it was too early to say he was out of danger. 2. His pulse rate and blood pressure were 'normal' according to a medical bulletin issued this evening. 3. The condition of Mr. X improved today. 4. He is still on blood transfusion, glucose and saline. 1. 3.

1324 4312

2. 4.

3241 2134

9

22.

23.

24.

25.

Career Avenues

1. 2. 3. 4.

This will ensure that MBA graduates will be more socially committed. Aspirants have to humane, sensitive and caring. Now it takes more than business sense to secure admission to Harvard Business School. The idea is to reshape the MBA experience from admission to job finding.

1. 3.

3412 1324

27.

3241 2134

1. Whether this amendment will achieve these objectives or not remains to be seen. 2. What is relevant is that it makes more sense to become an NRI because in spite of doing so they can spend five months of the year in India. 3. They can have the best of both worlds. 4. The central government is amending the NRI bill. 1. 3.

4123 3124

1. 2. 3. 4.

The tyre industry has repeatedly stated that the growers should get adequate price for rubber. This was due to the high release price by the STC. For this purpose a mechanism was evolved by the industry. Unfortunately during 1988 and 1989 this scheme did not work at all.

1. 3.

4213 1243

1. 2.

It costs Air India more than a million dollars for the new design. But unlike other international carriers, which announced their image, change through advertising. Air India appears to have done it on a scale nothing short of grandiose. The new look is the result of a yearlong effort by Lander Associates. Air India was following the footsteps of more successful predecessors like British Airways and A Italia.

3. 4.

26.

2. 4.

2. 4.

2. 4.

2. 4.

4231 3214

3241 1342

1. 3.

1342 3214

1. 2. 3. 4.

Kerala's big wigs do not seem to have realised the seriousness of the situation. But quite a few would complain about the ungodly inefficiency and confusion involved in tourist operators. The current high-profile ad campaign touts Kerala as "God's own country". No one who has chanced on Kerala would refute that claim.

1. 3.

3241 23 41

2. 4.

3142 1243

3421 1234

1.

In the others, diagnosis is often made when and if the filaria organism is found in the blood sample. 2. But, as in the case of nearly 17 million people in India, this does not happen. 3. Frightening though it sounds, filaria can be cured quite easily if it is discovered early enough. 4. A majority of them consult a doctor only when the limbs begin to swell up. 1. 3.

2431 3241

2. 4.

3124 4123

10

28.

29.

Career Avenues

1. 2. 3. 4.

Kerala is now poised for take-off. Hoteliers like the Taj group have been sounded out. And a Bangkok based NRI has expressed interest in putting up a hotel near Cochin. To help it along the state government will not hesitate to use private capital.

1. 3.

23 41 4321

1. 2. 3. 4.

30.

31.

This has been the fastest growing area in terms of work for management consultants not only in India but worldwide. The computer has forcefully changed the rules of the game in Indian business in terms of speed and access to information. Managements are only now beginning to accept this tool as crucial in adapting to fast changing market conditions. And has revolutionized the world of business both nationally and internationally. 2314 1432

1. 2. 3. 4.

The venue is the Pune plant of Telco. In recent months, labour history has been in the making at the unlikeliest of places. This symbolic protest focused national attention on what was until recently a local problem. Last month an estimated 3000 members of Telco Kamghar Sanghatana went on a 10-day hunger strike.

1. 3.

1342 2134

1. 2. 3.

Two companies both controlled and owned by Chhabria bought these shares. Old habits die hard, especially so with take over legend Manohar Chhabria. During the last one year, Chhabria has picked up at least five percent shares of the tyre company. After his setback he plans to re-enter the takeover game with a bid on the Mysore-based Vikrant Tyres.

2. 4.

2. 4.

2. 4.

4321 2143

2341 2143

1. 3.

4231 3142

1. 2.

4.

In the travel trade, this euphemistically referred to as ethnic traffic. Also, every year, thousands of NRIs living overseas in Europe, the US and the Gulf, visit India to meet their relatives and friends. For many Indians working and living overseas, coming back home is a tense, unpleasant and often humiliating experience. In fact, the tourist arrivals of 9000000 include a large number of Indians living overseas.

1. 3.

4123 3241

1. 2. 3. 4.

The other category is basically workers, mainly returning from the Gulf. Going through customs is a trying experience for both categories. First the professionals and businessmen most of whom are settled in the US, UK and Africa. The returning Indians can be divided into two broad categories.

1. 3.

4312 4213

3.

33.

1423 2143

1. 3.

4.

32.

2. 4.

2. 4.

2. 4.

2431 2413

2413 3412

1342 4123

11

34.

35.

Career Avenues

1. 2. 3. 4.

In any case, if they have any dutiable articles, they can ask for such articles to be entered on their passports. The customs arrival hall is busiest between 11 at night and 7 in the morning. The foreign tourists or businessmen usually sail through the green channel easily. Most of the flights from the Gulf, Far East and the West arrive at this time.

1. 3.

4213 2314

1. 2. 3.

Children love commercials. Often, one hears them singing jingles verbatim as if they were the latest Hindi pop hits. In India, marketers and advertisers have taken cognizance of this growing segment and are planning to cash in. Not only, do they enjoy watching them enormously but also and more importantly they remember.

4. 1. 3. 36.

37.

38.

2. 4.

3124 2431

1243 1423

1.

A rural exploratory study conducted recently suggests that in semi urban and rural environments, this is a more common phenomenon. 2. Significantly, for mothers, children are a source of information about what's happening in the market. 3. They also provide a brand reminder function and reinforcement when they sing heir favorite commercials. 4. But the more elusive urban markets are not effective by the commercial fundas. 1. 3.

3214 2314

1. 2. 3. 4.

In a study conducted some time ago, children saw commercials as 'fun'. Mothers say that children eagerly await the commercials, and once the programme starts, they lost interest. Adults may watch programmes but children are interested only in the ads. Children’s own perception of advertising reflects the emotional aspect.

1. 3.

4132 4213

1. 2. 3. 4. 5. 1. 3.

39.

1342 3142

2. 4.

1. 2. 3. 4. 1. 3.

2. 4.

2. 4.

2413 1432

1342 2314

Instead their requests are very specific – either a 'Leo' gun or a 'Frooti'. Unlike adults, children's requests for products are usually not need based. Memorability of brand names for children therefore becomes significant. Research also indicates that children, in some categories, seek brands and not products. Rarely do children ask for 'Toys' or 'Soft drinks.' 54312 51432

2. 4.

24513 24315

The company now spends close to 30% of its turnover on development activity. And not only has the company been successful in adopting technology to suit Indian conditions, but also in improving upon it. After 11 years of innovation and development, R & D has emerged as one of Indchem's main strengths. No wonder then that it has many firsts to its credit. 2314 2. 3412 3142 4. 1234

12

40.

Career Avenues

1. 2. 3. 4. 1. 3

41.

1. 2. 3. 4. 1. 3.

42.

1. 2. 3. 4. 1. 3.

43.

1. 2. 3. 4. 1. 3.

44.

1. 2. 3. 4. 1. 3.

45.

1. 2. 3. 4. 1. 3.

The heart of the problem is our import-licensing regime. Sadly, by protecting new industries from any foreign competition, they only succeeded in adding to the list of uncompetitive industries. but his government was ultimately misled into believing that competition from foreigners was best avoided. The Rajiv government tried to improve things somewhat by policies of liberalisation. 3421 2. 2314 1324 4. 1432 The revolution in science is, in fact far more significant than that which occurred at the end of the eighteenth century. The difficulty in getting hold of modern science is that it is moving so fast. In the past decade it has achieved an internal revolution. Although 20th century science rests securely on bases laid down in the 19th century, it has a character of its own. 3421 2. 1432 4312 4. 2341 It comes less frequently from the life stories of isolated groups of people. The most instructive history is that which traces the rise and fall of cities and empires. The story of Chicago's growth has more lessons than the biographies of a dozen great men would have. The most sensational news often comes from conflicts set up in urban conditions of politics and business. 1324 2. 4231 4123 4. 3241 Sometimes. local agencies even when they belong to the multinational agency which has a parent account, are resistant to dictation by the parent body. Multinational corporations often use the same advertising campaign throughout the world. Their argument is that it must suit the culture of the local people. But their local subsidiaries do not always accept it. 3412 2. 1342 2413 4. 4123 Special measures may be undertaken by the government to prevent communal flare up in sensitive areas. Committees may be formed from different ethnic and religious groups to solve differences through peaceful means. Politicians should refrain from giving speeches, which are provocative of religious sentiments of various communities. Special laws and ordinances may be promulgated to punish such politicians. 3421 2. 2134 3412 4. 1342 But the fiery youth can work wonders only if guided by wisdom and sagacity of elders. This is a responsibility for the youth. The great task of national reconstruction demands the sweat and blood of its citizens. Because they have zeal and vigour to take up challenges. 1342 2. 3241 2413 4. 4123

13

46.

Career Avenues

1. 2. 3. 4.

47.

48.

1. 3.

1234 4213

1. 2. 3. 4.

It is some vital energy that gives life to a civilization or people. History has numerous instances of old and well established civilizations fading away or ending suddenly. Without this it is a vain attempt of an aged person trying to play the part of a youth. Vigorous new cultures take their places.

1. 3.

24 13 2431

1. 2. 3. 4. 1. 3.

49.

1. 2. 3. 4.

50.

2. 4.

2. 4.

3421 1342

4123 3214

They are continually mixing with others and slowly changing. There may be a definite break between the old and new or vital links of thoughts and ideals may join them. They may appear to almost die and then rise again as a new people or a variation of the old. No people or races remain unchanged. 1342 2431

2. 4.

2143 4132

The bigness and complexity are apt to overwhelm us, as we try to unravel the thousand threads that go to make it up. Because it is a vast panorama and we are so near, it appears bigger and fuller than the centuries that preceded it. The 19th century is a fascinating period. But the study of it is no easy matter for us.

1. 3.

1423 2134

1. 2.

Proteins are complex molecules made up chiefly of amino acid residues. Those that are analogous in different organisms may show great similarity in their sequence of amino acids. This is more so if they are related by common descent or if they perform the same function in different organisms or both. These are arranged in a chain and the positions are numbered for convenience of investigation. 1324 2. 4132 4321 4. 1423

3. 4. 1. 3. 51.

Through the imposition of religious tests and the diversion of intelligent minds from scholarship to interesting occupations. In the 18th Century these institutions has shrunk very greatly. The aristocratic conception of education through physical education appealed to them strongly and thus revived and expanded sport. The modem expansion of sport followed upon the industrial revolution and is closely associated with the revival of universities.

1. 2. 3. 4. 1. 3.

2. 4.

3421 4132

They can also switch tasks among team members to alleviate boredom. Robots handle entirely the repetitious and unpleasant jobs. Workers are now being trained to handle only the ore complicated operations of the manufacturing process. The men can now stop at the assembly line to ensure quality. 3241 2. 2314 3142 4. 1423

14

52.

Career Avenues

1. 2. 3. 4. 1. 3.

53.

1. 2. 3. 4. 1. 3.

54.

1. 2. 3. 4. 1. 3.

55.

1. 2. 3. 4. 1. 3.

56.

1. 2. 3. 4. 1. 3.

57.

1. 2. 3. 4. 1. 3.

It may even be like gas kept confined in containers under great pressure, that reaction will one day, perhaps sweep back with an equal or greater violence of the physical force. As a result, the instinct to possess has been rooted out that it may not reappear. But this was done by and large by the us of physical force. There have been in our times, attempts to achieve a kind of egalitarian society and a picture of it in action after this achievement. 1342 2. 2314 4123 4. 4321 This is obvious with respect to the bulk of the relevant problems and disappointingly small predictive and controlling power of the available theories. Some of the accident shortcomings of the scientific method are of particular importance at present. There is little doubt that in contrast to the relatively mature state of physics, chemistry and astronomy, the scientific method has yielded, so far comparatively poor results in the social sciences and humanities. The lag in the science is apparent from the largely controversial state of expert opinion. 312 4 2. 23 41 1234 4. 4123 Complaints are frequently made of the vanity and shortness of human life. There is no end to the brilliancy on the variety of these. Ask the sum-total of the value of human life and we are puzzled with the length of the account and multiplicity of items in it. If we examine its smallest details, they present a world by themselves. 2134 2. 3241 1423 4. 4123 He must pay his taxes honourably and keep the laws, which have been made for the general good. It is argued that the poet, painter and musician has a duty to the community. A great deal has been said about the duty of the artist to society. He is a citizen like everyone else; he must pull his weight, not give himself airs or ask for special terms. 3241 2. 1423 4213 4. 2413 Cultivated people are a drop of ink in the ocean. And they become tongue-tied or terror struck in the presence of anyone whose make up was different from their own. Culture, thank goodness, is no longer a social asset, it can no longer be employed either as a barrier against the mob or as a ladder into the aristocracy. They mix easily and even genially with other drops, for those exclusive days are over when cultivated people made only cultivated friends. 4321 2. 1423 3142 4. 2134 They are corrupted by too much power and of a larger class of subjects. They are corrupted by too much passive and irresponsible obedience. Participation in a social order of this kind makes it very difficult for individuals to achieve any distinguishing mark of the ideally excellent human being. To a great or less degree all the civilized communities of the modern world are made up of a small class of rulers. 1324 2. 4213 3124 4. 4123

15

Career Avenues

58.

1. 2. 3. 4. 1. 3.

But if religion stands for a road to achieve the highest truth. Then it will bring people closer and make them stronger. It is what generates fanaticism and consequently differences in thinking and behaviour. If religion means the huddle of superstitions and practices associated with a particular belief. 1342 2. 4312 3412 4. 2143

59.

1.

The economy in India was at one time completely disturbed by the unprincipled sections of society. Hence without a proper economic order there cannot be political stability. This was because there was a connivance of hoarders and black marketers and petty politicians, that it destabilized the economic and political forces in the country. The economy of a country is so closely linked up with its political system. 4213 2. 1432 2143 4. 3421

2. 3. 4. 1. 3. 60.

1. 2. 3. 4. 1. 3.

Freedom is man's birthright. Freedom implies that we should not encroach upon the freedom of others. But, often, social life is so interdependent that one's freedom could mean another's restrictions. But un chartered freedom endangers social life. 2134 2. 1423 4213 4. 4321

16

Career Avenues

Answer Key 1–2 2–1 11–1 12–2 21–2 22–4 31–2 32–3 41–4 42–3 51–1 52–4

3–1 13–1 23–1 33–1 43–3 53–2

4–2 14–4 24–4 34–4 44–3 54–3

5–4 15–1 25–1 35–4 45–2 55–1

6–1 16–2 26–2 36–3 46–3 56–2

7–2 17–4 27–3 37–1 47–1 57–4

8–4 18–1 28–2 38–2 48–4 58–2

9–1 19–4 29–1 39–3 49–2 59–1

10–2 20–2 30–4 40–4 50–4 60–2

Explanatory Answers - Paragraph anagram 1.

(2)

(D) is the appropriate sentence to follow (1). This is because ---- refers to 'too formidable to overcome'. Moreover A, C is a mandatory sequence.

2.

(1)

The pronoun 'he' in (C) refers to artist in sentence (1) which makes (C) ideal to follow sentence (1); In addition, CA is a mandatory pair.

3.

(1)

The phrase 'these gifts' in (C) links the sentences to sentence (1). Only choice (1) has such a sequence.

4.

(2)

(C) is ideal to follow (1). BD is a better sequence than DB.

5.

(4)

CD is a mandatory sequence. Only choice (4) has this sequence.

6.

(1)

The pronoun 'it' in sentence (A) refers to blind courage. C6 is a mandatory sequence - ------it ceases to be (sent - 6) implies - - - -'most widely admired' ~(sent. C).

7.

(2)

(D) is ideal to follow sent (1) and sent (a) is continuation of (d).

8.

(4)

CD is a mandatory sequence and should follow (1)

9.

(1)

AD is a mandatory sequence . Only choice (1) has it.

10. (2)

(D) has to follow (1); bc is mandatory.

11. (1)

(C) ideally follows (1); cb is a mandatory sequence.

12. (2)

ca is a mandatory sequence.

13. (1)

(b) has to follow (1); cd is mandatory.

14. (4)

'b' has to follow (1); dc is a mandatory sequence.

15. (1)

(a) has to follow (1); ab is mandatory pair.

16. (2)

c, d is a mandatory sequence.

17. (4)

(d) has to follow (1) ab and 6 is a mandatory sequence.

18. (1)

Sentence (1) should be followed by (c).

19. (4)

(a) should follow (1) and (b) should precede (6).

20. (2)

Semantically (c) is a continuation of cd; ab is another mandatory pair.

21. (2)

(3) is the best sentence to begin the sequence. (1 ) is the best to conclude the sequence.

22. (4)

(2) is the right sentence to begin the paragraph. (1) should follow sentence (1).

23. (1)

1,2,3 is the right sequence semantically.

24. (4)

(1) is ideal to begin the sequence. Sentence (3) logically follows (1).

25. (1)

(3) is the most general of the statements to begin the sequence. (4) has to follow (3).

26. (2) 27. (3)

(3) is the right sentence to begin the paragraph.

28. (2)

(1) is ideal to begin the sequence. (4) has to follow (1).

17

Career Avenues

29. (1)

(2) should begin the sequence. (3) has to follow (2).

30. (4)

The ideal sentence to begin the para is (2) ; moreover, 4,3 is a mandatory pair.

31. (2)

(2) is the best sentence to begin the sequence. 3, 1 is mandatory, so choice (2) is more logical than (4).

32. (3)

Logically sentence (3) should begin the sequence. Sentence (2) logically follows (3).

33. (1)

4, 3, is a thought continuity.

34. (4)

(2) is ideal to begin the para. 2, 4 is a mandatory sequence.

35. (4)

(1) is the most independent statement to begin the sequence. (4) logically follows (1).

36. (3)

(2) should ideally begin the sequence as (1) and (3) have dependent words. (3) is a thought continuation from (2).

37. (1)

(4) should begin the para as the thought continues to (1) and (3) from (4).

38. (2)

(2) is the most logical sentence to begin the sequence. 5, 1 is a mandatory sequence as (1) is a thought reverser to (5).

39. (3)

(3) is ideal to begin the para. 3, 1 is a mandatory sequence; (2) is a conclusive statement.

40. (4)

4,3 is a mandatory pair; (2) is conclusive.

41. (4)

(2) logically begin the para ; (1) is conclusive.

42. (3)

From the given choices, (4) is most logical to begin the para. (~1) should follow (4).

43. (3)

2, 4 is a mandatory sequence.

44. (3)

1, 2 sequence is more logical than 2, 1 ; therefore choice (3) is preferred to (1).

45. (2)

Sentence (3) is the most general of the sentences to begin the para. (3) should be followed by (2). 2, 4 is mandatory.

46. (3)

(4) is the right sentence to begin the para. (2) should follow (4).

47. (1)

(2) should begin the para. 1,3 is a mandatory sequence.

48. (4)

Only (4) can begin the sequence. (1) must follow (4).

49. (2)

Only (3) can begin the para 4,2 is a mandatory pair.

50. (4)

(1) is devoid of any dependent phrase unlike other sentence therefore ideal to begin the para. 2,3 is a mandatory sequence.

51. (1)

(3) is ideal to begin the para. 4, 1 is a mandatory sequence.

52. (4)

4,3 is a mandatory pair. (1) is conclusive .

53. (2)

(2) should begin the para. 4, 1 is mandatory .

54. (3)

(1) is the best sentence to begin the sequence. (4) should follow (1).

55. (1)

(3) is general enough to begin the para. (2) explains the idea in (1); therefore, should follow (3)

56. (2)

(1) can begin the para and (3) is more of a conclusive statement.

57. (4)

Only sentence (4) can begin the sequence. 2,3 is mandatory,

58. (2)

(4) is the best sentence to start with. (3) follows and (2) is conclusive.

59. (1)

(4) is the best sentence to begin the sequence. 1,3 is mandatory.

60.

(2)

(1) is ideal to begin the para. (4) should follow (1) as it is a thought reverser. 2, 3 is mandatory.

18

Career Avenues

SENTENCE COMPLETION

19

Career Avenues

2.1 DESCRIPTION OF A SENTENCE COMPLETION In CAT you are presented with 5-7 sentence completion questions. A sentence completion question consists of an incomplete sentence. The sentence may have one or two blanks. From the five answer choices given, you are required to choose an appropriate word or words that can serve as the missing word/ words that will best complete the meaning of the given sentence. DIRECTIONS: Each sentence below has one or two blanks; each blank indicates that some thing has been omitted. Beneath the sentence are five lettered words or set of words. Choose the word or set of words for each blank that best fits the meaning of the sentence as a whole. Q.

A thinking man resists the vulgar prosperity that _____ ever to _____ , by preserving and communicating heroic sentiments. A. disintegrates ..... banality B. transforms ..... neutrality C. decays ..... luxury D. defers ..... poignancy E. retrogrades ..... barbarism

The theory underlying a sentence completion stimulus is that the meaning and the tone of a sentence can be gathered without the missing word/words. If so, it is possible for you to supply word(s) to complete the sentence. Invariably in every sentence completion question there will be a clue or two that could be used as leads toward identifying the correct choice word(s). These clues could be structural (grammatical) or semantic (meaning) clues. The word(s) that you choose should agree and merge with the sentence structurally and functionally or semantically. Every sentence completion question can be analyzed for the negative or positive nature of words to be filled in. 2.2

WHAT IS MEASURED

I.

Skill of vocabulary

Sentence completion question is a test of your word power and diction. It is only if you know the meaning of word(s) that you will be able to choose confidently, the appropriate word to fill the incomplete sentence. Moreover the question sentence also may have difficult words in them, knowing the meaning of which is essential for understanding the sentence in totality and to choose the appropriate word(s). Look at the following question: Q.

The department can use information received from dissatisfied employees and _____ to launch an investigation against the company. A. pettifoggers B. buccaneers C. ambassadors D. whistleblowers E. pacifists

The knowledge of the meaning of the words in the choices is essential for you to select the answer to this question with confidence. Let’s analyze the meaning of these words. A pettifogger is one who practices chicanery or unlawful business. Can this word fit the sentence? No. A buccaneer is a pirate; again not a word that would go with dissatisfied employees to supply information about a company. A pacifist is one who tries to appease warring people. Doesn’t match the idea.

20

Career Avenues

A whistleblower is one who informs about corruption or wrongdoing. Yes, this fits the sentence. An ambassador is not one to do anything with a company. In fact this is the first choice you could eliminate. ii.

Skill of semantics and syntax

A sentence has a semantic dimension (meaning) and a syntactical dimension (word arrangement). Knowledge of these two is a mandatory condition for you to be able to choose the appropriate word. You must understand the meaning of a sentence in order to identify the appropriate choice word that fills the blank and completes the sentence coherently. You should have a fair understanding of the functions of structural words in a sentence such as unless, rather, yet, while, unfortunately, in contrast, despite. Look at the following question: Q.

The meaning of words is destroyed if you change them, where as the meaning of a thought _____ whatever word you think proper to use. A. alters B. transforms C. remains D. settles E. belies

When you read the sentence you must recognize the function of whereas in the sentence. It signals a shift in the meaning of the sentence. One thing is destroyed whereas the other is not; the answer thus is remains. The semantic feature of a sentence refers to its meaning, and how meaning of the given words in a sentence help us to identify the word to complete the sentence. Study the following sentence: Q.

A cult brand _____ the imagination of a small group who spread the word, make converts, help turn a fringe product into a _____ name. A. ceases ...... redoubtable B. elevates ..... dynamic C. fires ....... dream D. seizes ..... mainstream E. pools ..... envious

The first clue is the word imagination; hence pools, ceases and seizes can easily be eliminated. You have to work with fires and elevates. Fire one’s imagination is a standard idiomatic usage. Moreover the meaning of the sentence implies turning a fringe (peripheral) product into (opposite of it) a dream product. iii.

Skill of identifying clues

A sentence completion question is not merely a test of your vocabulary. It also tests your ability to analyze the given sentence and to identify the structural and semantic clues that decide on the meaning and nature of the word(s) to be chosen. You will find one or two clues in a sentence completion question. Common key words such as and or but are all the more valuable for us in sentence completion. These are structural clues. (These are dealt with in detail later) 2.3

SENTENCE COMPLETION STRATEGIES

In order to answer sentence completion questions correctly, you should first read the entire sentence as you see it, without trying to fill in the word or words. Desist from working on each blank separately. The second part of the

21

Career Avenues

sentence may have a bearing on the first part of the sentence. And both words must fit into the meaning of the sentence; therefore read your choice into the sentence supplying and evaluating both words. Each sentence contains a few crucial clues that determine the answers, and finding these clues will guide you to the correct answer. Look for what is directly stated. In sentence completion questions, you may also find poetic and literary language, which tests your understanding of figurative usage. The correct answer is the one that is directly implied in the sentence. Ø Determine where the sentence is heading, using structural clues. Ø Anticipate answers in your words. Ø Look for answers that are similar to yours. Ø Fix words in the sentence to see if they gel with the whole sentence. Sentence completion strategy in a nutshell. Strategy 1 Read the entire sentence. In both one and two blank sentences read the entire sentence before you inspect the choices. An overall understanding of the sentence is essential. Strategy 2 Identify structural clues to determine the nature of the word(s) required. You can determine the positive or negative tone of the expected word using structural clues. And identify the words whose synonym, antonym, parallel word or cause/ effect that you have to supply. Strategy 3 Substitute your own words. Place your own words in the blanks assuming that answer choices are not given, keeping in mind the clues you have identified. Strategy 4 Identify words similar to the ones you substituted. If the word you have substituted goes with the requirement of the sentence, you are sure to find a word(s) similar in meaning and tone to the ones you have substituted. Strategy 5 Read the sentence back with word(s) filled in. Now read the entire sentence with the words filled in to see if the word(s) fit literally and figuratively. Identifying clues Now let’s discuss in detail the types of sentence completion clues that are found in the CAT. Identifying these clues is the single most promising method of solving sentence completion questions. A. THOUGHT CONTINUITY CLUES In this type the first part of the sentence and the second part are parallel ideas. There is no shift in meaning. The missing word in the sentence is parallel to or has a meaning similar to a word already given in the sentence. One part of the sentence may support or elaborate another part. One part of the sentence may be complementary to the other part in meaning. These clues continue the sentence in the same direction. The positive or negative tone of what follows is not changed by these clue words. The words that indicate such continuity are: v Similarly v Because v And v Since v Thus

22

Career Avenues

v v v v v

In fact More over In deed In addition Consequently

Study the following sentences: 1. Shakespeare lives forever; in fact he is immortal. 2. His argument was convincing and persuasive. 3. The film script showed depth and maturity; likewise the treatment showed profundity. In the sentences above the words in italics are structural clues. In the first sentence the clue to the word to be filled in is ‘in fact’, indicating that a word similar in meaning to the phrase ‘ lives for ever’ is required to complete the sentence. In the second sentence, the word to be filled in should be parallel to ‘persuasive’, determined by the thought continuity word’ and’. In the third sentence, the word ‘likewise’ indicates that a word parallel to depth and maturity completes the sentence. Now let’s understand more about thought continuity clues from the following exercise. SENTENCE COMPLETION DRILL - 1 DIRECTIONS: Read each of the following incomplete sentences, identify the thought continuity clues and supply your own word to complete the sentence. Your word should complete the meaning of the sentence. 1.

Human beings are not free-living solitary individuals; they are _____ .

2.

His writing lacks _____ ; there is nothing new or interesting in his books.

3.

Albert is famous for his _____ , he shows too high an opinion of himself and his achievements.

4.

Delegates expressed strong _____ to the scheme as it appeared anti-poor.

5.

There is a new feeling of _____ in the state owing to the progressive policies of the new government.

6.

Churchill was a fine political _____ ; thousands of people gathered to listen to his formal speeches.

7.

The employees decided to go on a strike as the authorities were _____ to their plight.

8.

His friends called him a _____ , he is very lively and full of energy.

9.

These days youngsters are highly _____ ; they show a lot of experience of the world and social situations.

10.

He is _____ and does not show allegiance to any group or political organisation.

11.

The company is known for its _____ mode of functioning; it is not organised into a clear and efficient organisation.

12.

_____ people do not show or accept things as they really are.

13.

The timing is _____ as the economy is booming and consumer interest is on the rise.

14. This shows the rise of fundamentalism _____ by selfish groups and power-hungry politicians. [Turn to end of chapter for answers]

23

B.

Career Avenues

PARALLEL IDEA CLUES

These clues are similar to the thought continuity clues. The word that completes the sentence is in the same tone as in the word already given. The word(s) that complete the sentence complements a word or phrase already given. All that the young ask of the political class is compassion and concern/ sympathy/ caring. ii. On questions about political and electoral reforms, the respondents showed great forethought and vision/ rationality. In the above sentences you may observe that the words given and the words that have to be filled in are parallel/ complementary or in the same tone. SENTENCE COMPLETION DRILL 2 DIRECTIONS: In the following sentences, identify the clue words and supply a word that is parallel to the given word to complete the meaning of the sentence. 1.

Global markets demand a more sophisticated and _____ approach.

2.

For those who use knowledge at every stage, information will be both their raw material as well as _____ .

3.

The _____ of intangible knowledge assets will also mean major changes in management structures.

4.

As the buying power of the consumer increases, he will also become more discriminating and _____ of quality and _____ of service.

5.

India is a _____ society with varied religions, languages and beliefs.

6.

The author of ‘An Equal Music’ knowingly and _____ overlooks some of the shortcomings of the artistic creations he refers to.

7.

Joyce’s clarity and _____ in sketching lives on the edge of despair ensures that her stories will be read widely.

8.

The combination of elegance and _____ in his speech enthralled the audience.

C.

THOUGHT REVERSAL CLUES

These change the direction of the sentence. They make one part of the sentence contradict or qualify another part. The word or phrase that completes the sentence is opposite in meaning to a word or phrase already given. The positive or negative charge of an answer is changed by these clues. Such signs include: v But v On the contrary v In spite of v Despite v Yet v However v Unless v Rather v Although v While v Unfortunately v Nonetheless v Ironically v Paradoxically Illustrations: i. His argument was flawed but persuasive. ii.

There is no cynicism in the way the young deal with issues; on the contrary they are optimistic toward life.

24

iii.

Career Avenues

The emphasis will not be on physical assets but on intangibles.

In the sentences above the words in italics are structural clues, which indicate that the words to be filled in are opposite in meaning to the words that are already given in the sentences. SENTENCE COMPLETION DRILL - 3 DIRECTIONS: Now identify the thought reversal clues in the following sentences and supply words that provide contrast ideas to those given in the first part of the given sentences. 1.

Unless organizations learn to create a strong brand value, they may find their contact with their end customer _____ by intermediaries.

2.

Spirituality is something that deals with the celebration of life and not with _____ .

3.

Indians have lost their _____ and have succumbed to a culture of materialism.

4.

Government agencies have failed to arrive at a consensus due to _____ considerations.

5.

Science and religion are not antithetical, but instead hold the prospect of a _____ relationship.

6.

Let our differences be a cause for admiration and celebration, not for _____ and _____.

7.

While we see Indians doing well outside India, inside India they are still _____. SENTENCE COMPLETION DRILL - 4

DIRECTIONS: Read each of the following sentences, identify the clues and write appropriate words in the blanks to complete the meaning of the sentence. 1.

It is heartening to come upon this book at a time when journalism seems to have been overtaken by _____ and meaninglessness.

2.

Although the developmental process provided security for women, it led to the _____ of their involvement in decision-making.

3.

The new approach emphasizes intervention to _____ the problem rather than to challenge the context that gives forth the problem.

4.

One of the greatest threats that faces the nation today is the growing _____ between people of different faiths.

5.

If the growing spread of reactionary and _____ religious assertion is any indicator, then we in our country have yet to learn how to deal with pluralism.

6.

Much of the book cites problems in public health reforms, but very few essays actually point towards _____ .

7.

At a time when children’s stories were exclusively moral and _____ Anderson revolutionized the genre by infusing it with humour, anarchy and sorrow.

8.

Anderson expressed the most painful and _____ emotions with extraordinary aesthetic control.

9. D.

In his simple and _____ way, Anderson told us much about the human condition. CAUSE EFFECT CLUES

25

Career Avenues

While studying a sentence completion questions, you should be able to gauge as to which direction the sentence is heading. If the sentence has a causal idea it is possible to infer what would logically follow the causal phenomenon. In other words, we can anticipate the type of effect that the given cause would lead to. Similarly by analyzing the nature of an effect (sentences in which effect is given), we can also infer the plausible causal factor that would’ve contributed to the type of effect given. The easiest way is to identify the tone of the cause or effect to be supplied into the sentence - negative or positive. Let’s work on sentences with cause effect relationship. Torture is so prevalent and _____ of the hapless so common that these helpless beings are certainly _____ when they pour out their story of agony. A. indemnifications ....... unhinged B. felicitation ....... believable C. disparaging ....... credulous D. victimization ....... credible E. endurance........... fathomable In the above sentence you observe that there are two clues: parallel and causal. Signaled by the word and the first blank needs a word parallel to torture i.e. victimization. The intensity of these two factors results in the possibility of the stories believable or credible. Look at another example: Higher education in India can no longer be allowed to remain as a/an _____ instrument that crushes the spirit of the individual while goading him into a _____ and consumerist mode. A. innerving ...... boisterous B. emasculating ...... materialist C. invigorating ...... pragmatic D. relegating ...... humanistic E. riveting ...... catholic An instrument that crushes the spirit of the individual cannot be innerving or invigorating (both means energizing) nor can it be riveting, since rivet means to hold firmly and relegate meaning to lower in rank is closer but the latter word cannot be humanistic. The word that is parallel to consumerist is materialist. In a similar mode identify the cause effect ideas in the incomplete sentences in the following drill and choose appropriate words to complete them. SENTENCE COMPLETION DRILL - 5 DIRECTIONS: Read each of the following incomplete sentences and identify word(s) from the choices to complete the sentences employing cause-effect clues. 1.

The therapies that try to _____ harmony and balance through physical methods have recently become known collectively as bodywork. A. adjust B. tamper C. facilitate D. expurgate E. aver

2.

The _____ of our natural environment has been accompanied by a corresponding increase in health problems of individuals. A. alterations B. transformation C. deterioration D. transitoriness E. glorification

3.

Rural indebtedness, which has already reached _____ , is also likely to grow into the drought-affected areas. A. magnanimous proportions B. alarming proportions C. unambiguous levels D. national standards E. unparalleled levels

26

Career Avenues

4.

Unlike their civilian counterparts, the army _____ are quick in completing enquiries, and therefore their report should be _____ before long. A. legislations ....... public B. headquarters ..... consigned C. courts ....... available D. machinery ..... commissioned E. headquarters ..... assessed

5.

At the macro-level hate _____ alienation between nations and provoke wars and large-scale cruelty. A. assuages B. alleviates C. exhorts D. aggravates E. alloys

6.

The world’s quest for peace is shaped by the _____ of power which also, _____ , shapes its approach to war. A. paradigm ..... ironically B. perpetrators ..... paradoxically C. framework ..... arguably D. corridors ..... eventually E. zealots ..... hypothetically

7.

The fact of the novel the reviewers did find praiseworthy was Rushdie’s _____ and sustained optimism in the face of _____ . A. forthrightness ..... evidence B. honesty ..... neutrality C. perseverance ..... adversity D. denial ..... criticism E. probity ..... dissonance

8.

The special _____ and privileges writers receive are their reward for using knowledge in the service of others. A. deference B. irreverence C. impetus D. broad-mindedness E. derision

9.

Unless there are continuous efforts to _____ micronutrients, food growing in soil will be _____ in other nutrients. A. advocate ..... bereft B. deliver ..... adequate C. replenish ..... deficient D. impair ..... lacking E. co-exist ..... prominent

10.

There is _____ evidence to _____ the claim that organic food is crucial for the health of human beings. A. yielding ..... rebuke B. captivating ..... draft C. compelling ..... stultify D. persuasive ..... support E. convincing ..... minimize

11.

Because the _____ span of television-trained audiences has been artificially shortened, candidates are forced to _____ their programs in 30 or 60 second summaries. A. campaign ..... augment B. confrontation ..... telecast C. concentration ..... embellish D. life ..... summarize E. attention ..... encapsulate

E.

WORD DEFINITION CLUES

Yet another common clue found in sentence completion stimuli is the meaning or definition of the word to be filled given as part of the incomplete sentence itself. It is similar to the thought continuity clue except that in this case the meaning of the word or definition of the word to be selected is already given. Let us study an example: The great irony of Apple Computers is that the very posture that won Steve Jobs a cult following creating an exclusive group of _____ technicians who _____ refused to bow to IBM, hobbled his company. A. iconoclastic ..... stubbornly B. conservative ..... invariably C. rebellious ...... timidly D. traditional ..... out rightly E. pre-meditated ...... obediently

27

Career Avenues

In the sentence above you would’ve already observed the words ‘refused to bow’, which signals the word to be placed in the second blank. This phrase clearly defines the word stubbornly. Another example: The crisis in modern education is not merely a constraint of _____ of funds but of _____ practices, unsuited for contemporary system. A. scarcity ....... doctrinaire B. fulsomeness .... authoritative C. restrictive ness ..... autocratic D. congruity ...... monolithic E. paucity ........ anachronistic In the above sentence the key to the latter word is ‘unsuited for contemporary education’. You can now easily eliminate unsuitable choices and arrive at choice E, i.e. anachronistic. SENTENCE COMPLETION DRILL 6 [CLUE- WORD DEFINITION] Directions: Choose the appropriate words that complete the following incomplete sentences using clues available in the sentences. The clue is that definition or meaning of words to be selected is already in the sentences. 1.

One approach to defining _____ art has suggested that some sort of female _____ or sensibility exists, involving an imagery and formal style specific to women. A. abstract ..... chauvinism B. modern ..... sensitivity C. contemporary ..... outlook D. traditional ..... beauty E. feminist ..... aesthetic

2.

Many of Herbert’s poems are mere bundles of _____ of metaphor, quaint and crabbed to the last degree. A. inditements B. oddities C. portrayals D. ramifications E. epitomes

3.

Although the hero travels from place to place and meets with a variety of adventures, Tom Jones is more than a _____ story. A. picturesque B. narrative C. picaresque D. thrilling E. sensational

4. Writing thus directly from his temperament, at the suggestion of his moods, Stern is curiously _____ . A. disinterested B. dispassionate C. subjective D. nonplussed E. promissory 5.

Tennyson courageously faced the facts of science, as revealed in geology and biology, but he succeeded in _____ religious consolation from the very things which were _____ as a fatal menace to religion. A. wringing ..... dreaded B. occluding ..... considered C. persisting ..... pertinent D. dispersing ..... highlighted E. inciting ..... fortified

6.

Decorous though they might seem, Austin’s _____ anonymity and her modest description of her miniaturist art implies a criticism. A. studied B. profound C. self-exploring D. self-effacing E. lackadaisical

7.

Most relationships between living organisms are essentially _____ ones, characterized by coexistence and interdependence. A. universal B. dichotomous C. distinctive D. symbiotic E. conducive

8.

Throughout Greek antiquity healing was considered as essentially a _____ phenomenon and was associated with many deities. A. spiritual B. psychotic C. ideal D. idyllic E. pluralistic

28

Career Avenues

9.

The great distinction between teachers and philosophers is that one class speaks from within, or from _____ and the other class from without, as _____ . A. cognizance ..... intuition B. memory ..... observers C. experience ..... spectators D. exposure ..... experimenters E. astuteness ..... forethought

10.

With new push technology, information can be _____ to individual requirements and proactively made available to the people who most use it. A. tailored B. invested C. managed D. heralded E. garnered

11.

It is believed that a teacher is not trusted to give _____ of his pupils, and an average teacher’s grading of his student is _____ by considerations of affinity and political persuasion. A. a subjective analysis ..... supported B. an impartial assessment ..... clouded C. a disinterested evaluation ..... denied D. a controlled measurement ..... experimented E. a pertinent report ..... provoked

12.

Although a physicist, Franck’s work had profound _____ chemistry and on the branch of _____ dealing with the fundamental process of photosynthesis. A. probity ..... natural science B. earnestness ..... pathology C. influence on ..... botany D. dependence on ..... anatomy E. impact on ..... philology

13.

I do not think that ordinary human beings can be happy without _____; in fact rivalry has been, ever since the origin of man, the spur to most serious matters. A. sports B. competition C. art D. cooperation E. concern

14.

He was a _____ in his love for his country and consequently expressed himself in extreme ways. A. partisan B. patriot C. figurehead D. chauvinist E. altruist

15.

Acceptance of an event is not to be mistaken for a life of passivity or submission to _____ characterized by laziness and a sense of helplessness. A. cynicism B. obsequiousness C. fatalism D. authority E. ambition SENTENCE COMPLETION DRILL 7 [CLUE - CONTRAST IDEA]

DIRECTIONS: Each of the following questions has a sentence that has either one or two blanks. The blanks indicate that a piece of the sentence is missing. Each sentence is followed by five answer choices that consist of words or phrases. Select the answer choice that completes the sentence best. 1.

Nature and nature-related knowledge used to be _____ from patentability but recent developments go a long way towards _____ the patentability of life forms. A. derived ..... understanding B. excluded ..... accepting C. demarcated ..... protecting D. inferred ..... safeguarding E. circumscribed ..... denoting

2.

To avoid _____ is a natural tendency, to implicate innocents for reasons of vendetta is not unnatural. A. self-denial B. self-incrimination C. self-depreciation D. self-indulgence E. self-adulation

3.

The cult advocates that change must be on our terms that it must not be under _____, that we must anticipate, and always be _____ to change.

29

Career Avenues

A. rhapsody ..... conforming C. censure ..... welcome E. dominance ..... zealous

B. duress ..... proactive D. grievousness ..... captivated

4.

Despite the honour accorded to them by society and the _____ rewards they enjoy for their work, many professionals complain that they feel _____ . A. optimal monetary ..... unperturbed B. scholastic ..... disturbed C. substantial monetary ..... demoralized D. intangible ..... left out E. perquisite ..... deemed

5.

Natural selection is a beautiful mechanism for turning rare events into _____ ones. A. unique B. advanced C. common D. complex

E. indefinite

6.

Some people have the _____ for learning foreign languages but they have no _____ to speak any. A. mania ..... urge B. capacity ..... ability C. aptitude ..... interest D. stamina ..... fondness E. compulsion ..... inclination

7.

Recent fossil evidence suggests that carnivorous dinosaurs were _____ swimmers, but some paleontologists still think that these dinosaurs _____ water. A. swift ..... entered B. nervous ..... loathed C. accomplished ..... feared D. unskilled ..... avoided E. natural ..... enjoyed

8.

Although we have been _____ universalisation of elementary education, since 1950, there is little hope of _____ the objective in the near future. A. forecasting ..... perpetuating B. predicting ..... internalizing C. carping ..... proliferating D. perpetuating ..... fomenting E. stressing ..... achieving

9.

The _____ of foreign rule has left its trace everywhere in the country but not so _____ as on our language and educational system. A. epitome ..... predictably B. stigma ..... noticeably C. remnants ..... obscurely D. reflections ..... proudly E. shadow ..... debilitating

10.

The language of cinema is often regarded as _____ but an examination of films from various countries reveal the _____ of the medium. A. deferential ..... variance B. appealing ..... dichotomy C. rural ..... permeability D. technical ..... idiosyncrasy E. universal ..... ethnicity SENTENCE COMPLETION DRILL - 8 [PARALLEL IDEAS]

DIRECTIONS: Each of the following questions has a sentence that has either one or two blanks. The blanks indicate that a piece of the sentence is missing. Each sentence is followed by five answer choices that consist of words or phrases. Select the answer choice that completes the sentence best. 1.

Implementation of educational programs is halting, inefficient and often _____ the main purpose. A. obstructive of B. insular to C. rallying around D. eclectic in E. diagonal to

30

Career Avenues

2.

Some people argue that the theory of a female aesthetic really _____ women in that it limits them to certain feminine forms and images. A. thwarts B. restricts C. disports D. reminisces E. quashes

3.

As a result of rapid increases in computer-based technologies, _____ expansion in financial flows, both _____ and within countries has emerged. A. an energizing .......... federal B. a disturbing ......... overseas C. a dramatic ......... cross border D. an unexpected ........ local E. an unprecedented ......... constitutional

4.

It is the _____ and moral _____ of every generation to ensure a better world for the next generation. A. spiritual ........ reward B. ethical ...... obligation C. emotional ...... duty D. cognitive....... right E. prudish ....... exigency

5.

A fall in agricultural income means a _____ fall in rural demand. A. concomitant B. illustrious C. praiseworthy D. debatable

E. detectable

6.

Too many people take a _____ arithmetic view of life; they assume that if numbers go down, living standards will _____ and correspondingly go up. A. advanced ..... debilitatingly B. definite ..... diminish C. simplistic ..... automatically D. narrow ..... indefinitely E. preponderous ..... unwittingly

7.

Han’s hopes and expectations clash continually with the loneliness and _____ with which he must contend. A. alienation B. empathy C. insensitivity D. poise E. selflessness

8.

Though he earned _____ as a painter, the artist felt that he _____ more acclaims for his murals. A. wealth ...... escaped B. fame ...... deferred C. distinction ..... publicized D. reputation ...... deserved E. heroism ...... revered

9.

Under the influence of western materialism, we try to find happiness in added _____, and fail to realize the need for compassion. A. comforts B. appendages C. expediencies D. volumes E. valuables

10.

The Golmal party _____ tabling a serious no confidence motion in the council; nevertheless it expressed mild protests against the treasury benches. A. resorted to B. refrained from C. insisted on D. relied on E. stumbled upon

11.

The international news service _____ information _____ , so that events are reported all over the world shortly after they happen. A. records .......accurately B. falsifies ....... deliberately C. verifies ....... painstakingly D. disseminates ....... rapidly E. suppresses ....... thoroughly

12.

During the initial days, a name _____ against free software was common, as many assumed that such software would be whimsical and _____ . A. prejudice ....... ineffective B. propriety ....... applicable C. bias ....... user-friendly D. reaction ....... conducive E. antagonism ....... unsupported

31

Career Avenues

13.

While the financial system is engaged in _____ investments and thereby accelerating growth, developments in the system have raised concerns relating to the _____ of the system. A. mitigating ........ safety B. perpetuating .... logic C. garnering ..... proximity D. enhancing ....... propinquity E. augmenting ...... soundness

14.

The discovery of the radar has led to resolute efforts by the US to _____ other countries from stepping up their capabilities because of its desire to remain ahead. A. preclude B. summon C. proscribe D. rarify E. obviate

15.

The non-dramatic literature of the seventeenth century shows the opposing forces - on the one hand _____ the right of the individual, on the other, insistence on the _____ standards. A. assertion of ..... adherence to B. poignancy to ..... simplicity of C. manipulation of ..... recasting of D. emending of ..... benediction to E. repression of ..... continuance of

16.

The new literary movement, which we have been tracing, was the work of a small ______ of men, for the most part comparatively _____ . A. clique ..... recognized B. assay ..... famed C. compendium ..... isolated D. fraternity ..... idolized E. coterie ..... obscure SENTENCE COMPLETION DRILL 9 [CLUE - CONTRAST]

DIRECTIONS: Each of the following questions consists of a sentence that has either one or two blanks. The blanks indicate that a piece of the sentence is missing. Each sentence is followed by five answer choices that consist of words or phrases. Select the answer choice that completes the sentence best. 1.

Bernard Shaw’s _____ spirit was completely antithetical to the sentimental _____ of the Victorian era. A. apprehensive ....... intransigence B. dichotomous ...... holism C. doubtful ........ pessimism D. rebellious ....... non-conformity E. skeptical ........ optimism

2.

Today’s small, portable computers contrast markedly with the earliest electronic computers which were _____ . A. effective B. useful C. destructive D. over-used E. enormous

3.

That truth led him to proclaim that good ends can never be attained by _____ methods. A. malicious B. evil C. stealthy D. surreptitious E. cantankerous

4.

Modern feminist writers consider women’s _____ men’s domination one of their greatest themes and therefore achievements. A. revolt against B. conformity to C. denial of D. liberation of E. hostility to

5.

Although Freud showed a deep interest in religion and spirituality, he never acknowledged _____ as their source. A. sense perception B. mystical experience C. intuitive understanding D. cosmic phenomena E. hallucinations

6.

Proverbs, said Aldus Huxley, seems like _____ ; indeed truisms are _____ and often ancient. A. clichéd .... original B. platitude ..... trite C. axioms .... evanescent D. short-lived .... polemic E. embellishments .... personifications

32

Career Avenues

7.

Belief in the supernatural is an unfortunate psychological _____ fostered by blind belief in religious practices. A. delusion B. uneasiness C. construct D. conundrum E. pretension

8.

‘East goes West’ is the fervent personal expression of a/an _____ who longed to be accepted into American society. A. emigrant B. immigrant C. alien D. native E. serf

9.

The violent thunderstorm and sudden lightning after the oppressive heat are a/an _____ of one of nature’s most magical moments. A. forbear B. epithet C. antithesis D. aftermath E. celebration

10.

Philosophers are _____ and are rarely known to be practical in their approach to human problems. A. eclectic B. perfectionists C. idealists D. chauvinists E. humanists

11.

Pure knowledge is valuable for its own sake, because the human spirit cannot resign itself to _____ . A. contemplation B. experimentation C. abstraction D. ignorance E. affectation

12.

Immigrants are _____ as well as _____ , so they create jobs as well as take them. A. users ..... researcher B. consumers ..... producers C. buyers ..... sellers D. supporters ..... detractors E. proponents ..... critics

13.

Some species obtain nutrients from dead plant and animal matter, but many others are _____ , they steal nutrients from other living organisms. A. predators B. preys C. parasites D. pests E. neophytes

14.

Labour that leaves no room for education probably _____ human potential. A. consigns B. augments C. precipitates D. stunts

E. retorts

15.

Human progress and its reflection are depicted in the media; in other words, media is viewed as a _____ of the society. A. symbol B. insignia C. metaphor D. epistle E. mirror

16.

I would like to have the matter settled immediately because it keeps a man on _____ A. oars B. feet C. tenterhooks D. hold

E. ire

17.

The interest generated by the Soccer World Cup is _____ compared to the way cricket _____ the nation. A. milder ..... unnerves B. lukewarm ....... electrifies C. unusual ....... grips D. magnanimous ....... magnifies E. unwarranted ..... augurs

18.

Our government, no doubt, seems to survive on borrowed ideas, using _____ solutions. A. decadent B. uncanny C. second-hand D. moribund E. contrived

19.

The telephone symbolizes that awkward _____ ; while it intends to bring us together, it keeps us apart. A. allusion B. satire C. lampoon D. paradox E. icon

20.

Most of us suffer in life because we look at life in compartments, but if we look at things _____ everything will look much better and comfortable. A. fragmentarily B. holistically C. wishfully D. unarguably E. well-intentioned

33

Career Avenues

SENTENCE COMPLETION DRILL - 10 DIRECTIONS: Each of the following questions consists of a sentence that has either one or two blanks. The blank(s) indicate that a piece of the sentence is missing. Each sentence is followed by five answer choices that consist of words or phrases. Select the answer choice that completes the sentence best. 1.

While _____ residents are looking to the MoU as another instrument for city improvement, _____ are beginning to wonder if this is just another instance of mistaking passivity for action. A. skeptical ....... proponents B. suspicious ....... detractors C. pugnacious ....... truculent D. industrious ....... mettlesome E. optimistic ....... pessimists

2.

The report is thought to provide useful pointers to companies exploring click and mortar solutions to _____ the failure of dotcoms. A. counteract B. reinforce C. popularize D. deploy E. posit

3.

Nothing is permanent and forever, all phenomena are in _____ state of dynamic _____ from moment to moment. A. a stubborn ....... stasis B. an intransigent ....... flux C. pretentious ....... transformation D. altering ....... equanimity E. a serene ....... regression

4.

We need to _____ our political process in order to make sure that we can have an _____ democratic process. A. streamline ....... uninterrupted B. marshal ....... unrestrained C. eradicate ....... enticing D. program ....... enviable E. interpolate ....... iconic

5.

Research agencies couldn’t be _____ and expect to grow just by piggybacking on clients or _____ market share from each other. A. obsequious ..... clinching B. peremptory ..... touting C. truculent ..... gnawing D. complacent ..... clawing E. perilous ..... trading

6.

The exercise served _____ purposes; it demonstrated why one should not be taken in by miracles; but it also _____ the practical applications of scientific principles. A. supreme ..... negated B. duel ..... stressed C. various ..... overlooked D. vital ..... contested E. dual ..... underscored

7.

In the past fifteen months, the Chinese Government has tried to _____ the approaching crisis by _____ deficit financing to speed up investments. A. divert ..... clamouring for B. aggrandize ..... truncating C. avert ..... resorting to D. mollify ..... regularizing E. pacify ..... risking

8.

The dispute between the parties became so _____ that there was every likelihood of a free exchange of blows. A. complicated B. acrimonious C. bellicose D. aggressive E. typical

9.

Once he had _____ the difference between regional dialects, Abby found himself speaking the language _____ . A. rejected ..... considerably B. grasped ..... effortlessly C. mastered ..... implicitly D. forgotten ..... eloquently E. recognized ..... ambiguously

34

10. 11. 12. 13.

Career Avenues

His _____ directions mislead; we did not know which of the two roads to take. A. complicated B. foolish C. ambiguous D. just

E. pertinent

Architects travel to Greece and Italy to _____ the marvels of classic design. A. imitate B. treat C. recall D. study

E. exculpate

Automation threatens mankind with an increased number of _____ hours. A. meager B. useless C. active D. idle

E. trivial

Surprisingly, you are frightened by a concept that _____ from your own mind. A. accepted B. idealized C. externalized D. originated

E. detested

14.

Except for a few productive land areas, the prevailing methods of cutting the soil rapidly exhaust its _____ . A. loam B. content C. fertility D. substance E. barrenness

15.

Several Indian nuclear experts have argued that it is in the country’s _____ to conduct at least one hydrogen bomb test before the CTBT deadline in 2002. A. safety B. redemption C. grasp D. disaster E. meticulousness

16.

The bill in this effect is _____ the Lok Sabha, and the claims of making the entire country literate sound _____ . A. discussed in ..... logical B. clamoring ..... inadequate C. pending with ..... hollow D. passed by ..... desultory E. worrying ..... fulsome

17.

It is pointed out that around 2.5 million children are _____ elementary education every year since there are not sufficient schools to _____ them. A. deprived of ..... accommodate B. taught in ..... send C. desisted from ..... promote D. missing ..... recognize E. boycotting ..... cater

18.

By early 1999, the public reports on basic education in India _____ two myths about primary education. A. contradicted B. negated C. augmented D. exploded E. ruined

19.

Now the major _____ in the way of development is said to be _____ of resources. A. hurdle ..... hindrance B. threat ..... extension C. obstacle ..... paucity D. impediment ..... availability E. contradiction ..... non-availability

20.

First, the calculations seem to be _____ to avoid the _____ to eat ‘the forbidden food’. A. deflated ..... knack B. inflated ..... temptation C. estimated ..... inclination D. propounded ..... penchant E. assumed ..... penalty SENTENCE COMPLETION DRILL - 11

DIRECTIONS: Each of the following questions consists of a sentence that has either one or two blanks. The blanks indicate that a piece of the sentence is missing. Each sentence is followed by five answer choices that consist of words or phrases. Select the answer choice that completes the sentence best. 1.

The finance minister suggested that to contain the damages caused by the earthquake, resources could be _____ by cutting down on the _____ expenditure. A. adjusted ..... impending B. planned ..... efficacious C. offset ..... fecund D. retained ..... significant E. mobilized ..... unproductive

35

Career Avenues

2.

Even as city corporations try to use limited resources for the _____ task of urban renewal, citizens should also not _____ their responsibility toward the city. A. challenging ..... resurrect B. petty ..... sacrifice C. tedious ..... rescind D. daunting ..... abdicate E. veritable ..... negotiate

3.

Knowledge management is not merely the collection and _____ of all the information that exists in a company in one location. A. diversification B. storage C. appraisal D. collaboration E. termination

4.

Knowledge management is the science of embracing the disparate sources where knowledge resides whether they are explicit or _____ intellectual capital and unspoken thought processes. A. dynamic B. progressive C. tacit D. tangible E. transitory

5.

An effective knowledge management system will help _____ the knowledge wherever it resides and give it relevance and context by linking it to related data. A. cultivate B. outperform C. utilize D. negate E. diffuse

6.

Technology in today’s digital age is the _____ that can enable and streamline the capture and flow of information in a company. A. evaluation standard B. one-stop shop C. research tool D. undeniable facilitator E. amalgam

7.

Research on _____ origins of adult diseases has shown that children with low birth weight suffer from _____ degenerative disease in adult life such as osteoporosis or diabetes. A. inherited ..... incurable B. fetal ..... chronic C. hereditary ..... contagious D. familial ..... pathological E. paternal ..... neutral

8.

The new lot is fluent in _____ prevalent in this field such as suspension, domestic inquiry and dismissal. A. habits B. discourses C. communications D. idioms and vocabulary E. truisms

9.

Most people are _____ the philosophy that discipline is a behavioural change process and not a _____ . A. averse to ..... behavioural product B. oblivious of ..... punitive measure C. resistant to ..... welcome change D. unaware of ..... peace accord E. doubtful about ..... culmination

10.

Although many products are _____ in a burst of enthusiasm and good intentions, they often lose their way through _____ delays due to one reason or the other. A. kicked off ..... protracted B. relocated ..... peeling C. released ..... regulated D. carried forward ..... ongoing E. inaugurated ..... ancillary

11.

One of the preoccupations of the postcolonial mind is the _____ , the shame of having been seduced by the colonialist’s enticements. A. polemic responses B. subservience C. guilt complex D. relentless allegiance E. identity problem

36

Career Avenues

12.

There is a belief that mythology and therefore classical dance would be much easier to perceive for the common man than the _____ themes in contemporary dance. A. prominent B. abstract C. colourful D. redundant E. trite

13.

With the termination of talks went the one _____ of hope that the two _____ neighbors would be able to come closer and be amiable to each other. A. ray ..... contended B. glimmer ..... belligerent C. pool ..... traumatized D. boomerang ..... allying E. response ..... warning

14.

Man, with his _____ comfort and his passionate greed would eventually destroy natural resources. A. proximity to B. reliance on C. coherence to D. adherence to E. proclivity for

15.

Sea horses are _____ eaters and slurp their prey whole. A. voracious B. intransigent C. stupendous

D. gourmet

E. intensive

16.

Many instances of human rights abuse _____ from across the world have _____ the need for human rights education in its varying manifestations. A. observed ..... predetermined B. documented ..... reinforced C. glaring ..... assuaged D. reported ..... ignored E. observed ..... degraded

17.

There is a near total _____ on the fact that human rights education can help reduce human rights violations and contribute to building a free society. A. polarity B. preciousness C. consensus D. vacousness E. treatise

18.

In times of war, it is not permissible to think of your enemies as ordinary human beings: it is almost a _____ duty to think of them as mindless savages. A. patriotic B. perfidious C. royal D. commissioned E. permissible

19.

Despite the fact that _____ attitude has not been a major one on the modern art scene; it has rarely provided a foundation for a/an _____ interpretation of things in which one goes by what is verifiable. A. dogmatic ..... eclectic B. negative ..... universalistic C. holistic ..... humanistic D. realistic ..... aesthetic E. positivistic ..... empirical

20.

At a time when children’s stories are moral and _____ , Anderson revolutionized the genre by infusing it with _____ . A. doctrinaire ..... principles B. ethical ..... precepts C. didactic ..... humor D. altruistic..........purpose E. immortal..........motifs SENTENCE COMPLETION DRILL - 12

1.

Projecting rail accidents as natural phenomena is a very convenient way of _____ all concerned, of their responsibility, accountability and even _____ . A. acquitting ..... forthrightness B. upstaging ..... liability C. absolving ..... culpability D. reinstating ..... callousness E. capsizing ..... constructiveness

37

Career Avenues

2.

The role of consumption of raw foods in the development of cancer remain unclear though certain studies have _____ that their intake should be _____ . A. reinforced ..... continued B. approved ..... reprimanded C. disproved ..... contained D. refuted ..... controlled E. suggested ..... banned

3.

For a just society, the immediate need is to control deterioration of moral standards, in fact _____ of ethical values has eaten away the moral fabric of the society. A. degradation B. disabling C. corruption D. abeyance E. terrorizing

4.

The quality of energy spontaneously _____ , and the spontaneity of degradation drives the independent processes webbed around and within us, as though the _____ gear wheels of a sophisticated machine. A. spirals ..... simplistic B. augments ..... automatic C. degrades ..... interlocked D. permeates ..... suppressed E. extends ..... engaging

5.

The moot point of democracy is the avoidance of _____ , or, in other words, the _____ of unfreedom. A. anarchy ..... search B. polytheism ..... examination C. oligarchy ..... predominance D. plutocracy ..... denial E. dictatorship ..... rejection

6.

Clearly these proposals are totally _____ ; and, if agreed to, they will further _____ the imbalance. A. pluralistic ..... aggrandize B. one-sided ..... aggravate C. multifarious ..... alleviate D. lopsided ..... expurgate E. grounded ..... restrict

7.

More and more researchers are finding that what once were thought to be separate populations of the same species are, in fact, different _____ , each as genetically _____ as a horse from a cow. A. races ..... compatible B. adaptations ..... contravening C. inheritors ..... uniform D. lineages ..... distinct E. families ..... immutable

8.

To meet the changing needs of the customers the service provider has to _____ on an ongoing basis. A. replicate B. innovate C. proliferate D. deprecate E. compute

9.

When I compared Prigogine’s criteria for self-organising systems to Bateson’s criteria of mental processes, I found that the two sets of criteria are very _____ ; in fact, they seemed close to being _____ . A. contradictory ..... similar B. similar ..... identical C. similar ..... polarised D. acceptable ..... conducive E. marred ..... moribund

10.

Although Mounik admits that the _____ of his career is to make Blue Bunny a house hold name, he is not _____ about achieving his goals. A. worst part ..... concerned B. unspoken aim ..... bothered C. biggest challenge ..... unduly worried D. ultimate goal ..... prepared E. capitulating dream ..... vulnerable

11.

Creative minds see new angles to everyday things that most people would take for granted; indeed there is always a sense of _____ , of wonder at even the most mundane objects. A. pride B. anxiety C. surprise D. transtoriness E. curiosity

38

Career Avenues

12.

While an overall picture is slowly emerging now post-disaster, there is as yet no _____ of the economic losses caused by the cyclone. A. accurate assessment B. general account C. tentative evidence D. acute measure E. approximate analysis

13.

Some employees may find it difficult to present a/an _____ of their case in a logical and orderly sequence. A. planned proposal B. speculative base C. appropriate denial D. coherent account E. dispassionate report

14.

To the Indian observer, both are US power companies and thus _____ ; so the success of one and failure of another looks strange. A. quite dissimilar B. seemingly similar C. apparently polarized D. most expected E. visibly discrete

15.

A chaste mind never tends towards the _____ of the material world and helps in discovering the ways of the most needed _____ . A. meaninglessness ..... carnal satisfactions B. eternal aspect ..... human pursuit C. morality ..... ultimate satiations D. outer glamour ..... spiritual tranquility E. worldly passions ..... physical respite

16.

Generosity being the most pleasing aspect of a man’s personality, its best _____ is that whatever he possesses, he happily distributes it among the needy. A. contrast B. determination C. sanctity D. procrastination E. manifestation

17.

No type of test can remain _____ forever; a periodic review should be made and new ideas are tried to enhance the value of the test. A. sacrosanct B. flexible C. succinct D. enamoured E. syndromes

18.

Volumes of tabulated data, reports and planning guidelines are as good as _____ if they are not easy to find when required. A. non-existent B. proactive C. over-used D. cultivable E. naught

19.

Proponents insist that a/an _____ artistic language is being created, corresponding to the distinct social experience of women _____ male defined art. A. authentic ..... independent of B. particularistic ..... influenced by C. universalistic ..... relying on D. congruous ..... inspired by E. autonomous ..... liberated by

20.

We who have been labeled, stereo typed and _____ out of the very definition of art must be free to define art. A. expiated B. classified C. atoned D. gerrymandered E. famished

21.

Our experiments have led us to _____ that electrostatic change has _____ properties that prevents abrasions. A. visualize ..... anesthetic B. theorize ..... lubricating C. posit ..... centrifuging D. predict ..... electrifying E. hypothesize ..... physical

39

Career Avenues

22.

Those who deny the _____ of miracles say that miraculous events are impossible because they _____ the laws of nature. A. existence ..... violate B. occurrence ..... rescind C. redemption ..... break D. practice ..... govern E. possibility ..... encumber

23.

Society undergoes continual changes but this change is not _____ because for everything that is given in society something is taken. A. temerity B. amelioration C. reciprocal D. probity E. self-reliant

24.

Every man is at some time _____ ; language cannot paint him with colors but we know that it _____ and contains us. A. affable ..... perpetuates B. indefinable ..... circumscribes C. sensible ..... pervades D. spiritual ..... balks E. abates ..... enciphers

25.

In the context of erosion of quality in higher education offered by private enterprises, and the growing _____ with the performance of college teachers, there is an urgent need to _____ professional standards. A. disenchantment ..... enhance B. aspirations ..... mollify C. exception ..... vilify D. immobility ..... pursue E. magnitudes ..... repress

26.

Education in the current millennium would not _____ any gap between its content and the living experience of its people. A. truncate B. brook C. profess D. crack E. foresee ANSWER KEY AND EXPLANATIONS - SENTENCE COMPLETION DRILL 1

There are different words that could complete the sentence. One/two such option is/are given below. 1. interdependent / dependent 2. originality / novelty 3. self adulation / imperiousness 4. skepticism / opposition 5. optimism / hope 6. orator / rhetoric 7. apathetic / unconcerned 8. live wire / an enthusiast 9. wise / cognizant 10. independent / non-partisan 11. haphazard 12. hypocritical 13. ripe / apt 14. fanned / instigated SENTENCE COMPLETION DRILL 2 1. 2. 3. 4. 5. 6. 7. 8.

methods / consumer – friendly end product / goal implementation / execution aware / demanding multicultural intentionally / consciously/ deliberately precision simplicity / rhetoric

40

Career Avenues

SENTENCE COMPLETION DRILL 3 1. 2. 3. 4. 5. 6. 7.

interrupted mortality values / idealism pluralistic complementary / symbiotic / harmonious antagonism and betrayal floundering SENTENCE COMPLETION DRILL 4

1. 2. 3. 4. 5. 6. 7. 8. 9.

mediocrity / triviality exclusion / denial alleviate dissention / polarity / cleavage divisive / insular / chauvinistic potential solutions / remedial action didactic rawest characteristic / unsophisticated SENTENCE COMPLETION DRILL 5

1. 2. 3. 4. 5. 6. 7. 8. 9. 10. 11.

[C] [C] [B] [C] [D] [B] [C] [A] [C] [D] [E]

bringing in harmony and balance parallel idea - increase in health problem complements deterioration. The intensity expressed in the passage requires an equally intense expression. Inquiry related to court and quickness naturally resulting in availability. provoking wars is a result of aggravating (worsening) alienation. The contrast ideas - those who quest for peace, plans war. praiseworthy, indicates positive quality in extreme situation (adversity) Respect is the apt word; B, E are negative. One doesn’t ‘receive’ broadmindedness. ‘unless’ is the clue indicating contrast ideas The latter word should be ‘support,’ ‘organic food is crucial necessitates the word ‘support.’ Reducing a program into 30 – 60 seconds (encapsulate) parallel to shortened. SENTENCE COMPLETION DRILL 6

1. 2. 3. 4. 5. 6. 7. 8. 9. 10. 11. 12. 13. 14. 15.

[E] [B] [C] [C] [A] [D] [D] [A] [C] [A] [B] [C] [B] [D] [C]

Involving imaginary and formal style (aesthetic) Quaint and drabbed indicate a negative trait – oddities Picaresque is an adventure story. At the suggestion of his moods – indicate subjectivity. ‘Fatal’ refers to dreaded. Anonymity and modesty. Coexistence and interdependence are key words. Associated with deities is the clue word. The clue is from within and without. The clue word is to individual requirement. By considerations of affinity is the clue. Process of photosynthesis, pertains to botany. Rivalry is the clue word Extreme love implies ‘chauvinism’ Sense of helplessness (fatalism)

41

Career Avenues

SENTENCE COMPLETION DRILL 7 1-B

2-B

3-B

4-C

5-A

6-C

7-C

8-E

9-B

10-E

8-D

9-A

10-B

8-B 18-C

9-B 19-D

10-C 20-B

8-D 18-D

9-B 19-C

10-C 20-B

8-D 18-A

9-B 19-E

10-A 20-C

8-B 17-A

9-B 18-E

10-C 19-A

SENTENCE COMPLETION DRILL 8 1-A 11-D

2-B 12-A

3-C 13-E

4-B 14-C

5-A 15-A

6-C 16-E

7-A

SENTENCE COMPLETION DRILL 9 1-E 11-D

2-E 12-B

3-B 13-C

4-A 14-D

5-B 15-E

6-B 16-C

7-A 17-B

SENTENCE COMPLETION DRILL 10 1-E 11-D

2-A 12-D

3-B 13-D

4-A 14-C

5-D 15-A

6-E 16-C

7-C 17-A

SENTENCE COMPLETION DRILL 11 1-E 11-C

2-D 12-B

3-B 13-B

4-C 14-E

5-C 15-A

6-B 16-B

7-B 17-C

SENTENCE COMPLETION DRILL 12 1-C 11-C 20-D

2-A 11-E 21-B

3-A 12-A 22-A

4-C 13-D 23-B

5-E 14-B 24-C

6-B 15-D 25-A

T

7-D 16-E 26-B

42

Career Avenues

READING COMPREHENSION

43

Career Avenues

Ø Ø Ø Ø Ø Ø Ø Ø Ø Ø

Instructional objectives To familiarize you with the nature of comprehension testing in the CAT. To expose you to the type of passages tested To familiarize you with the type of questions given To enable you to acquire skills necessary for efficient reading To expose you to result-oriented strategies for better comprehension To provide you with adequate practice material to reinforce learnt strategies To enable you to evaluate and monitor your own performance To diagnose and identify areas that need remedial work. To provide guidance for additional reading to compensate for reading deficiencies To provide you with test taking strategies toward computer adaptive tests

The following pages contain Ø detailed description of the various comprehension question types Ø description of the skills tested Ø approaches to build up CAT reading skills Ø strategies for cracking each of the question types Ø illustrations for each question type Ø comprehension skill builders 4.1

DESCRIPTION OF A READING STIMULUS

In CAT you are presented with 4–5 passages followed by 4 – 6 questions based on their content – either stated in or implied by the passage. A CAT reading comprehension passage and question appear as follows: [We will be using this passage for discussion throughout] DIRECTIONS: Read the passage below and answer the question that follows. The behavioral school of psychologists believes that all learning is responses to stimuli. And that all learning should be assessed through definable responses. For behaviorists the concept of ‘size’ among children for example, is assessed by cuing the child to respond to questions pertaining to size; if a child is unable to respond to the stimulus, the child is assessed as not having developed the concept of size. However, for Piaget, this is a ‘mechanical’ view of the behaviorists. The concept of ‘size’, among children, Piaget says, is one dimension of an array of interrelated images (mental images): Covert responses can be expressed only with the image of all other concepts, say, length, height, weight etc. All these contribute toward a child’s response to say, the expression of size. Difficulties arise only if a child is unable to express a concept through a response to a single stimulus without the other images. In such a case, Piaget argues, that any dimension of concept formation can be assessed by providing clues on other related dimensions, in order to make the child respond covertly to achieve the fundamental requirement of assessment through response to stimulus, even if one has to present tangible examples of other conceptual dimensions. 1.

The author of the passage is primarily concerned with presenting A. Criticism of Piaget’s views on the conceptualization of behaviorism. B. Evidence to support Piaget’s claims about the problems inherent in behaviorism. C. An account of Piaget’s counter proposal to one of the traditional assumptions of behaviorism. D. An overview of behaviorism and its contributions to Piaget’s alternate understanding of behaviorism. E. A history of behaviorism and Piaget’s reservations about it.

The reading comprehension questions in the verbal sections tests your interpretive, applicative and inferential skills. The passages contain words ranging from 600 words to 1300 words. And these discuss topics from the pure sciences, social sciences and art/ literature. Since the reading passages include several different content areas, you

44

Career Avenues

will probably be generally familiar with some of the material. However, neither the passages nor the questions assume detailed knowledge of the topics discussed. 4.2

WHAT IS MEASURED

Reading comprehension questions measure your ability to understand, analyze and apply information and concepts presented in written form. All questions are to be answered on the basis of what is stated or implied in the reading material, and no specific knowledge of the material is required. Reading comprehension, therefore evaluates your ability to Ø Understand words and statements in the reading passages (questions of this type are not vocabulary questions. These questions test your understanding of and ability to use specialized terms as well as your understanding of the English language. You may also find that questions of this type ask about the overall meaning of the passage) Ø Understand the logical relationship between significant points and concepts in the reading passages, for instance, such questions may ask you to determine the strong and weak points of an argument or to evaluate the importance of arguments and ideas in a passage. Ø Draw inferences from facts and statements in the reading passages. (The inference questions will ask you to consider factual statements or information and on the basis of that information, reach a general conclusion) The following factors contribute to success in CAT passages FAMILIARITY WITH THE PASSAGE Previous knowledge of the content of a given reading selection invariably helps in efficient comprehension and in reading with speed. A technical test taker, therefore, familiarizes himself with varied contents and domains of knowledge. The single factor that contributes to your reading ability consequently is, wide reading. Read as much as you can. There’s nothing like being able to prepare for your reading comprehension test while having your morning cuppa! LANGUAGE COMPETENCE Proficiency in language most necessarily means better reading in terms of speed and comprehension. Shifts in meaning in a sentence are generally signaled by those structural words that carry profundity of ideas; you overlook them and the essence of the sentence is not grasped. So is the tone of a write up determined by the language intonations! If you’re a voracious reader you are fluent in reading and understanding written information. Some of you wouldn’t have been able to maintain your reading habits due to reasons such as the nature of academic course chosen, lack of time, and/or a host of other reasons. In order to compensate for it you have to spend more time now catching up with your reading. DOMAIN-SPECIFIC JARGON Contributes greatly to your performance in CAT reading passages. Those who have a flair for reading would find passages from any source readable - whether the passage is on art, history or science. Therefore we reiterate the need for your familiarizing with such literature. ABSTRACTIONS Often you’ll come across passages that would require slow reading when you’re hurrying through a test. The content may be highly abstract and may require repeated reading. You may not be able to cope with the expected accuracy. Such passages as philosophy, art and literature come under this category. CLARITY OF WRITING The general clarity of writing seen in well-organized articles may be missing in certain passages. Their inclusion most certainly is to add to the difficulty of reading the passages and answering those questions. Such passages may have to be read repeatedly to understand and then answer the questions that follow them.

45

4.3

Career Avenues

COMPREHENSION TOOLS

Essentially reading comprehension questions in any aptitude test, measure an individual’s ability to comprehend information given in the passages and to answer the questions that follow the passage, based on what is stated or implied in the passage. The passages that we see in the CAT verbal section are intended to measure your ability to read and comprehend the information so that the questions that follow are answered with maximum accuracy. The difficulties that test takers encounter are manifold: Ø Unfamiliar content Ø Uninteresting reading matter Ø Higher level questions such as inferential, application and evaluation questions Ø Difficult-to-eliminate choices Added to these is the stress and tension that goes with taking a test! In order to overcome these difficulties, you need to acquire, Ø strategies to overcome reading difficulties Ø practical tools to improve up on your current comprehension levels Ø productive tactics to crack the of the varied types of question types Ø methods of logic analysis of each of the question types and Ø strategies for answering the different question types Reading is a skill Reading ability is a cognitive disposition acquired over years of learning and practice- both conscious and unconscious. Reading ability depends, to a greater extent, on readability - the attribute of the reading material. You should not assume that your reading ability is poor by assessing the speed with which you have read, say, a passage on philosophy, in a given time. Even if you have previous reading in philosophy, you require more time to read a philosophy passage with fairly good comprehension, owing not only to the abstraction involved but also to the implicit meaning underlying such text than read a passage on a concrete readable text. At the same time you should realize that there are certain cognitive behaviours that are peculiar to efficient readers, viz. concentration, mental translation as one navigates through the passage, summation, connecting ideas, inferring, judging the material read etc. When do we consider a passage readable? Or why do test takers differ in their ability to read? There are differences between the reading skills required in an academic environment and those that are required on standardized tests. If you have a good sense of the passage structure and gist of paragraphs, you’ll have no problem navigating through the text. Practice active reading; what does an active reader do? Ø He thinks about what he is reading and assimilates denotations. Ø He goes beyond the denotations, searches for connotations (implicit ideas) Ø Translates ideas into his own language. Ø Poses questions to himself as he reads. Ø Skims through illustrative matter and scans through meaty points. Ø Looks for both matter discussed, and the speaker’s points of view. As you read across the passage, build a cognitive map of the information assimilated: while reading, you should keep asking these questions: - What is the topic the author is dealing with? - What is he attempting to establish? - How well does he succeed in this attempt?

46

Career Avenues

- What are his observations? - What shift does the author have in mind when moving on to this paragraph? - What bearing does this paragraph have on the author’s main idea? - What is the author trying to portray? Is he proposing a new idea? - Is he making any recommendations? - Challenging notions? Criticizing policies? And so on. Much of our reading is neither accurate nor thoughtful. When relaxed, we naturally skip and skim. Ordinarily, such reading neither deserves nor receives careful attention. But, often, we attempt to read closely reasoned and factpacked texts. Reading to understand involves attention, retention and awareness. The reading of genuinely important material must be painstakingly careful. Comprehension passages in tests need to be carefully read: I. to gain and understand accurate information and ideas. II. to recognize author’s organization of the content and style of writing. III. to interpret author’s intent and IV. to analyze and evaluate the textual matter. Reading effectively is reading with both comprehension and speed. An efficient reader reads thought units, not word by word. Your aim should be to reduce the number of fixations, and lengthen the eye span. Your reading rate will increase as you learn to do this efficiently and so will your comprehension. A skillful reader does not work with isolated units but with context - what precedes and follows the particular material being read. A good reader rarely loses time by having to refer to the beginning of a sentence or paragraph. Rather, the thought will have been carried through in one series of lengthened glances. The best way of learning to read with speed as well as comprehension is to “read with your mind assisted by your eyes”. Doing this will enhance comprehension by reducing the number of fixations and increasing concentration. Practice finding main thoughts in a passage and separating them from purely illustrative matter. Learn to find key words and phrases that summate the main ideas of the passage. These steps will greatly increase reading speed, although you must not forget that different kinds of material require different reading speeds. An efficient reader assimilates information that is read and translates it into his own language. Recall of information exactly as it is, is neither possible nor necessary. Instead, gather the gist of the information, form opinions and draw conclusions. Careful reading of any selection should lead to an understanding of the central theme and purpose as well as organization of the main thoughts. If you’re methodic, that is good for your academic reading. For aptitude test preparation, take the road not taken. You can’t approach an aptitude test in a casual, piecemeal way. If you want to maximize your likelihood of success, you have to take advantage of the unconventional approaches. The number of passages ranges from 2 to 4; the number of questions ranges from 6 to 10 and the length of the passages ranges from 70 - 250 words. The passages are either run-on matter or with indented paragraphs. Therefore, be sensitive to issues of topic, scope, structure and source of such passages in order to draw from the repertoire of your tactics to maximize your performance. In sum, read to locate those crucial ideas Ø to find out where answers to specific questions lie. Ø to get the gist of each paragraph. Ø to distinguish opinions or interpretations from factual assertions; this is an important skill in reading. Ø to attack the passage for author’s view. Ø when asked for meanings of words/phrases look for nearby context. Ø finally get the author’s purpose in writing the text, to help you answer inferential questions (if any) Know these facts: Ø Different questions require different strategies. Ø The reading comprehension questions are of varying difficulties.

47

Career Avenues

ANALYSIS OF CAT PASSAGES A. SOURCE OF THE PASSAGES Where are these CAT reading passages drawn from? Are they from college textbooks? No. These passages are drawn from: 1.

Pure Sciences : articles from biology, chemistry, physics, earth sciences or any other sub branches or related disciplines. or Ø evaluation of research hypotheses Ø discussion of recent findings Ø research reports drawn from science journals Ø new scientific observations Ø new developments in a specific science discipline Ø history of a discovery / events that led to a discovery

2.

Socio-political/cultural : A passage pertaining to social /economic / political / history It may be a discussion on: Ø achievements of Indians, since Independence Ø cultural heritage Ø an event from history Ø cultural aspects of our life Ø economic/ trade matters Ø contributions of famous personalities Ø discussion of cults / movements etc.

3.

Art / literature : A passage from humanities - related to art, literary criticism, or history of any of these. Ø discussion on / evaluation of art forms Ø book review Ø author review Ø comparative discussion of books / authors Ø trends / progress in art / literature Ø philosophical discourses / discussions Ø literary movements Ø philosophical articles Ø anthropological discussions

B. QUESTION TYPES The common comprehension questions are: I.

CONTENT BASED QUESTIONS

The questions that are based on the information stated directly in the passage (denotations) include these. Content-based questions are of two types: i main idea question ii. specific idea question (mostly line numbered or with paragraph references) II. STRUCTURE BASED QUESTIONS These questions ask you to analyze and evaluate i. the organization and logic of a passage ii. the author’s style of writing iii. how the paragraphs are arranged iv. how the author takes the discussion forward

48

Career Avenues

III. APPLICATION QUESTIONS These are questions that ask how information given in the passage can be applied in contexts outside the passage. These include i. working with hypothetical situations ii. recognizing scope of the text outside its context iii. evaluating analogous situations iv. the ideas the author would agree / disagree with IV.

INFERENCE QUESTIONS

These questions ask about ideas that are implied in the passage (connotations) i. meanings that are drawn from the passage ii. suggested ideas iii. inferences based on comparisons iv. inferences based on cause-effects v. drawing generalizations / conclusions V.

EVALUATION QUESTIONS

A higher-difficulty question type based on a passage involves evaluation questions. The question requires you to judge the information given in the passage, evaluate the authors arguments and/ or assess the scope and application of the information in the passage. These questions would also require you to identify, if any, the flaws in judgment, question the validity of a proposition and the like. VI.

ASSUMPTION QUESTIONS

These are questions in which you are required to identify the assumption that the author is making while stating something within a passage. VII.

LOGICAL CONCLUSION QUESTIONS

In this question type you are asked to identify a statement that would logically follow the passage. The answer is closely related to the content in that it summarises the ideas discussed in the passage. VIII. ATTITUDE / TONE QUESTIONS These questions require you to make a statement about the author, his attitudes, values, and principles as inferable from the passage or the tone of the passage (again an indirect reference to the author himself). IX.

SPECIALISATION OF THE AUTHOR

A not so common question; nevertheless, there could be questions in which you are asked to infer the specialization of the author. X.

IDENTIFYING THE SOURCE OF THE PASSAGE

An occasional question may be asked on the source from which the passage is drawn.

49

Career Avenues

Now let’s discuss each of these question types in detail and the strategies for answering for each of these. I. MAIN IDEA QUESTION Each reading comprehension passage in the CAT verbal section is a unified whole – that is, the individual statements and paragraphs support and develop one main idea or central point. Sometimes the central idea is told in the passage explicitly and sometimes it will be necessary for you to determine the central point from the overall organization of the passage. You may be required to recognize a correct restatement, or paraphrase, of the main idea of a passage, or to assign a title that summarizes, in a clause or sentence, the central idea of the passage or a particular paragraph. The central idea question is phrased in one of the following ways. Ø Ø Ø Ø Ø Ø Ø Ø Ø Ø Ø Ø

Which of the following best states the central idea of the passage? The author’s primary purpose / objective is to... Which of the following is the principal topic of the passage? The author’s main concern is... The central idea / theme / topic of the passage is... Which of the following best summarizes the passage as a whole? In the passage, the author is primarily interested in... Which of the following titles best summarizes the passage as a whole? The primary purpose of the second paragraph is which of the following? The last paragraph of the passage performs which of the following functions? A suitable title for the passage would be... Which of the following questions answers the central theme of the passage?

ANSWERING STRATEGIES Central idea questions are general questions; therefore, they will always have ‘general’ answers. This helps you to eliminate choices that are specific – choices that contain information pertaining to a specific paragraph alone. The wrong choices are partly true or are centered on any of the paragraphs. The main idea may be presented immediately in the very first sentence. Or it may be presented in the end of the first paragraph. Or the main idea may be a sum of the opening sentences of each paragraph. Just focus on the first and last sentence of each of the paragraphs and it is unlikely that you do not get the main idea. Now let’s answer a main idea question: PASSAGE-1 The behavioral school of psychologists believes that all learning is responses to stimuli. And that all learning should be assessed through definable responses. For behaviorists the concept of ‘size’ among children for example, is assessed by cuing the child to respond to questions pertaining to size; if a child is unable to respond to the stimulus, the child is assessed as not having developed the concept of size. However, for Piaget, this is a ‘mechanical’ view of the behaviorists. The concept of ‘size’, among children, Piaget says, is one dimension of an array of interrelated images (mental images): Covert responses can be expressed only with the image of all other concepts, say, length, height, weight etc. All these contribute toward a child’s response to say, the expression of size. Difficulties arise only if a child is unable to express a concept through a response to a single stimulus without the other images. In such a case, Piaget argues, that any dimension of concept formation can be assessed by providing clues on other related dimensions, in order to make the child respond covertly to achieve the fundamental requirement of assessment through response to stimulus, even if one has to present tangible examples of other conceptual dimensions.

50

1.

Career Avenues

The author of the passage is primarily concerned with presenting A. Criticism of Piaget’s views on the conceptualization of behaviorism. B. Evidence to support Piaget’s claims about the problems inherent in behaviorism. C. An account of Piaget’s counter proposal to one of the traditional assumptions of behaviorism. D. An overview of behaviorism and its contributions to Piaget’s alternate understanding of behaviorism. E. A history of behaviorism and Piaget’s reservations about it.

Analysis of the question: A. B. C. D. E.

Does the author criticize Piaget’s views? No. He doesn’t criticize either Piaget or Behaviorists. Is the author supporting Piaget’s claim? Not really. He doesn’t support Behaviorists either. Is the author presenting Piaget’s counter proposal to Behaviourist’s assumptions? Yes! He is. In the second paragraph the author presents Piaget’s criticism of Behaviourist’s assumptions and offers alternate solutions (remember the author himself doesn’t criticize Behaviorists) Is the passage about an overview of Behaviourism? No. Does Behaviourism contribute to Piaget’s understanding? Not really. Is the author presenting a history of Behaviourism? No; you can easily eliminate this choice.

Now look at the choices of this question: The author of the passage is primarily concerned with presenting A. Criticism of Piaget’s views on the conceptualization of behaviorism. B. Evidence to support Piaget’s claims about the problems inherent in behaviorism. C. An account of Piaget’s counter proposal to one of the traditional assumptions of behaviorism. D. An overview of behaviorism and its contributions to Piaget’s alternate understanding of behaviorism. E. A history of behaviorism and Piaget’s reservations about it. Analysis of the question: You’re asked to identify the primary purpose of the passage. Let’s look at the choices: A] Resolve a dispute. Is the author stating any dispute and offering solutions? No. B] Discuss controversial perspectives – similar to choice [A]. is there any controversy discussed in the passage? No. C] Is the author comparing and contrasting three approaches? No. The author only discusses three historiographical considerations (opening line of the passage). D] Set forth the author’s position on historiographical issues? Yes. This is the author’s purpose. E] Is the author questioning the validity of methods used by historians? No. Unrelated to the passage. You may find it interesting to know that it is possible to eliminate choices with the first word of each of the choices itself. [In a central idea question, all that matters in the choices is the first word, which summarizes the purpose of the entire passage.] Is the author presenting criticism? (Not his) Ø Evidence (the author gives no evidence) Ø Account of counterproposal? (Yes) Ø Overview of Behaviourism (No) Ø History (No) Let’s answer another question:

51

Career Avenues

PASSAGE-2 Three historiographical considerations played an important role in my decision to write a history of Black women in the United States. First, I was persuaded that Black people and White people in the United States clearly represent two different cultures, with different traditions and often times diametrically opposed past experiences. The same may hold true for the two sexes, whose roles, history and contributions seem on close examination to be almost different enough to constitute different cultures. Of course, all groups share in the common history. I do not believe that recognizing the existence of large cultural subdivisions means succumbing to separatism, cultural nationalism, and a narrow particularistic vision. There is a place for universalistic interpretations of that, which is common to all humanity, and there is a place for that which is particular to one special entity. We accept that there are both a world history and a history of France or of India, histories that are not the same, but that fit into similar categories. Our historical perceptions can only be enriched by accepting the fact that, since sex and race are frequently used (however unfairly) to assign certain roles and statuses to people, these people have had different historical experiences from those of the dominant group. Second, I became convinced that standard histories of the United States have tended to overlook the contributions and viewpoints of Black Americans and women. I hope and expect that the present generation of United States historians will rectify this mistake, recognizing that women and Blacks were there and that their special contributions to the building and shaping of American society were different from those of White men. In the meantime, a separate history of Black women is needed. Finally, I had to consider the role that a White historian can legitimately play in the writing of Black history. Certainly, historians who are members of the culture about which they write will bring a special quality to their material. Their understandings are apt to be different from those of an outsider. But scholars from outside a culture have frequently had a view that contrasts with the view of those closely involved in and bound by their own culture. These angles of vision are complementary in arriving at an accurate picture of the past. The interpretation of the Black past made by Blacks will probably be somewhat different from that made by Whites. This does not mean that Black history can or should be interpreted only by Blacks. It should be interpreted by both groups, so that, in the juxtaposition of different interpretations, in debate, and in the clash of opinions, a richer and fuller and more solidly based history will emerge. 1.

The primary purpose of the passage is to A. resolve a historiographical dispute. B. discuss a controversial historiographical perspective. C. compare and contrast three alternative historiographical approaches D. set forth the author’s positions on three historiographical issues. E. question the validity of three historiographical methods used by the other historians.

It will also help you to know, when and for what kind of passages these ‘key words’ are used. Take a look at some of them. A.

B. C. D.

Describe What do we generally describe? - A scientific phenomenon - An event / incident - A piece of history Explain / discuss / elucidate A concept, an idea – with definitions, illustrations, features, cause-effect relationships. Analyze An issue, argument, problem, hypothesis Question Assumptions, validity of a hypothesis, policies, new scientific propositions

52

Career Avenues

E.

Compare and contrast two books, two authors, two hypotheses, two schools of thought. (pointing out similarities and differences)

F.

Propose Suggest a new hypothesis, a new theory, a solution for a problem alternative methods.

G.

Argue Against a policy, for an idea, to establish one’s own idea, to disprove/ refute a claim.

H.

Criticize a book, an author, a policy, hypothesis, or a point of view.

I.

Evaluate an argument, validity of a hypothesis, a new theory, suggested solutions, proposed solutions / ideas.

II. SPECIFIC IDEA QUESTION These questions measure your ability to comprehend the supporting ideas and illustrative matter in a given passage. These assess your ability to differentiate ideas that are explicitly stated in the passage from ideas that are implied by the author. You may be asked about the facts cited in the passage, specific data presented by the author, cause-effect relationship between factual data, or about descriptions used to elaborate the main theme. While questions on the main idea ask you to determine the meaning of a passage as a whole, questions about specific ideas ask you to determine the meaning of individual sentences and paragraphs. The best way to find the answer to specific question is by using ‘lead words’ together with chronological order. The lead word is the most specific word or phrase in the question. It tells you what the question is about, which in turn tells you where to look in the passage. Lead words are often nouns or noun phrases. Once you locate the lead word in the given question stem, simply scan the passage in order to locate it. The answer to the question is near the location of the lead word. Use these hints to identify lead words: i. ii.

capital letters, illustrative words, quantified data, abbreviations, names are easy to spot. a question stem may have more than one lead word; trace the lead word that is less repeated in the passage. iii. if the question doesn’t have a lead word, check out clue words in the choices; verify with the passage of their relevance. Most of such questions have references to line numbers or paragraph references. The question stems are as follows. Ø Ø Ø Ø Ø Ø Ø Ø

The author uses the term ……………. (line 20) to mean The ‘common ground of understanding’, line 5-6 most probably refers to…. The author quotes sprat (in Para 2) primarily in order to…. According to the author caffeine differs from adenosine in which of the following ways? The author mentions the excavation in Cyprus to emphasize which of the following points? The author cites all of the following as reasons for …… EXCEPT According to the passage, researchers use surrogates for all of the following experiments EXCEPT According to the passage, which of the following is true/not true of stellar evolution?

ANSWERING STRATEGIES

53

Career Avenues

Most specific questions have what are called lead words or phrases in their stems. These are words or phrases that are easy to locate in the passage by skimming through it. Scroll long passages to bring the part of the passage that possibly has the answer to the center of the screen. This will enable you to compare each of the choices that are placed on the right side of the screen with the content on the left side (in the passage). With the answer choices right next to the matter in which the answer lies, matching the choices become convenient. Questions with line references are all the more easy to spot and analyze. For line referred questions, always read the preceding line(s) and/or the following line(s) as required by the question. Every passage is followed by specific questions; use them to your advantage. In some cases, certain lead words are scattered all over the passage. Then make use of the related idea in the question stem to narrow down to which attribute of the lead word is required by the question. Let’s answer a specific question: PASSAGE-1 The behavioral school of psychologists believes that all learning is responses to stimuli; and that all learning should be assessed through definable responses. For behaviorists the concept of ‘size’ among children for example, is assessed by cuing the child to respond to questions pertaining to size; if a child is unable to respond to the stimulus, the child is assessed as not having developed the concept of size. However, for Piaget, this is a ‘mechanical’ view of the behaviorists. The concept of ‘size’, among children, Piaget says, is one dimension of an array of interrelated images (mental images): Covert responses can be expressed only with the image of all other concepts, say, length, height, weight etc. All these contribute toward a child’s response to say, the expression of size. Difficulties arise only if a child is unable to express a concept through a response to a single stimulus without the other changes. In such a case, Piaget argues, that providing clues on other related dimensions, in order to make the child respond covertly to achieve the fundamental requirement of assessment through response to stimulus, even if one has to present tangible examples of other conceptual dimensions, can assess any dimension of concept formation. 1.

According to Piaget’s conception of concept formation, if a child is unable to respond to the stimulus to assess concept formation in him, which of the following would be true? A. The child is assessed as not developed the intended concept. B. Both the child’s response and the stimulus are considered incomplete to arrive at any conclusive judgment. C. New concepts would be taught to the child in order to make him form mental images. D. Clues on other related concepts would be provided, in order to make the child respond through association recall. E. The child forms an entirely new set of concepts.

Analysis of the question: A. B. C. D. E.

If a child is unable to respond to the stimulus to assess concept formation in him, does Piaget argue that the child is assessed as not developed the concept? No. This is, in fact, the claim of Behaviorists. Does Piaget claim that the child’s response as well as the stimulus is considered incomplete? No. Does Piaget propose to teach new concepts for formation of mental images? No. Beyond the scope of the passage. Does Piaget suggest that clues on other related concepts could be provided? Yes. In Para 2 he suggests exactly this. D is the best answer. Does Piaget claim that an entirely new set of concepts would be formed? Again, beyond the scope of the passage.

Now look at another question.

54

Career Avenues

PASSAGE-2 Three historiographical considerations played an important role in my decision to write a history of Black women in the United States. First, I was persuaded that Black people and White people in the United States clearly represent two different cultures, with different traditions and often times diametrically opposed past experiences. The same may hold true for the two sexes, whose roles, history and contributions seem on close examination to be almost different enough to constitute different cultures. Of course, all groups share in the common history. I do not believe that recognizing the existence of large cultural subdivisions means succumbing to separatism, cultural nationalism, and a narrow particularistic vision. There is a place for universalistic interpretations of that, which is common to all humanity, and there is a place for that which is particular to one special entity. We accept that there are both a world history and a history of France or of India, histories that are not the same, but that fit into similar categories. Our historical perceptions can only be enriched by accepting the fact that, since sex and race are frequently used (however unfairly) to assign certain roles and statuses to people, these people have had different historical experiences from those of the dominant group. Second, I became convinced that standard histories of the United States have tended to overlook the contributions and viewpoints of Black Americans and women. I hope and expect that the present generation of United States historians will rectify this mistake, recognizing that women and Blacks were there and that their special contributions to the building and shaping of American society were different from those of White men. In the meantime, a separate history of Black women is needed. Finally, I had to consider the role that a White historian can legitimately play in the writing of Black history. Certainly, historians who are members of the culture about which they write will bring a special quality to their material. Their understandings are apt to be different from those of an outsider. But scholars from outside a culture have frequently had a view that contrasts with the view of those closely involved in and bound by their own culture. These angles of vision are complementary in arriving at an accurate picture of the past. The interpretation of the Black past made by Blacks will probably be somewhat different from that made by Whites. This does not mean that Black history can or should be interpreted only by Blacks. It should be interpreted by both groups, so that, in the juxtaposition of different interpretations, in debate, and in the clash of opinions, a richer and fuller and more solidly based history will emerge. 1.

The author suggests in lines 20-23 that she does not consider the historiographical perspective described in lines 5-8 to be overly A. traditional B. particularistic C. universalistic D. individualistic E. patriotic

Analysis of the question: The question is, what she doesn’t consider…. You may directly refer to lines 20-23; the author says that there is room for universalistic interpretations, and that she doesn’t believe in succumbing to separatism. Therefore the answer is B. III. LOGICAL STRUCTURE QUESTION These questions require you to analyze the logical organization of the passage, how the author presents his thesis, his style of writing. They may require you to recognise how the passage is constructed: for instance, does the author begin with a general idea, and provide supporting ideas and illustrations in the subsequent paragraphs. Or does the author begin with illustrations and instances and conclude with a general statement. You may also be asked to assess the strengths and weaknesses of the author’s arguments. Your ability to evaluate the passage critically is assessed through this question type. These passages need not conform to any formal logic nor do you have to know formal ways of evaluating an argument. You need to be just alert and sharp and reason carefully. Logical organization questions are usually phrased as follows: Ø The author organizes the passage chiefly by... Ø Which of the following best describes the organization of the passage? Ø Which of the following best describes the organization of the author’s discussion of the role of….?

55

Career Avenues

ANSWERING STRATEGY The structure of a passage can be inferred from its indentations. Observe what each of the opening lines of the paragraphs discusses. These lines will give you a fair idea of the logical organization of the passage. Study what the purpose of each of the paragraphs is. Observe the opening sentences of each of the paragraphs of the passage below. PASSAGE During adolescence, the development of political ideology becomes apparent in the individual; ideology here is defined as the presence of roughly consistent attitudes, more or less organized in reference to a more encompassing, though perhaps tacit, set of general principles. As such, political ideology is dim or absent at the beginning of adolescence. Its acquisition by the adolescent, in even the most modest sense, required the acquisition of relatively sophisticated cognitive skills, the ability to manage abstractness, to synthesize and generalize, to imagine the future. A steady advance accompanies these in the ability to understand principles. The child’s acquisition of political knowledge also promotes the growth of political ideology during adolescence. By knowledge I mean more than the dreary “facts,” such as the composition of county government, that the child is exposed to in the conventional ninth-grade civics course. Nor do I mean only information on current political realities. These are facets of knowledge, but they are less critical than the adolescent’s absorption, often unwitting of a feeling for those many unspoken assumptions about the political system that comprise the common ground of understanding – for example, what the state can “appropriately” demand of its citizens, and vice-versa, or the “proper” relationship of government to subsidiary social institutions, such as the schools and churches. Thus, political knowledge is the awareness of social assumptions and relationships as well as of objective facts. Much of the naïveté that characterizes the younger adolescent’s grasp of politics stems not from an ignorance of “facts” but from an incomplete comprehension of the common conventions of the system, of what is and is not customarily done, and of how and why it is or is not done. Yet I do not want to overemphasize the significance of increased political knowledge in forming adolescent ideology. Over the years I have become progressively disenchanted about the centrality of such knowledge and have come to believe that much current work in political socialization, by relying too heavily on its apparent acquisition, has been misled about the tempo of political understanding in adolescence. Just as young children can count numbers in series without grasping the principle of ordination, young adolescents may have in their heads many random bits of political information without a secure understanding of those concepts that would give order and meaning to the information. Like magpies, children’s minds pick up bits and pieces of data. If you encourage them, they will drop these at your feet – Republicans and Democrats, the tripartite division of the federal system, perhaps even the capital of Massachusetts. But until the adolescent has grasped the integument function that concepts and principles provide the data remain fragmented, random, disordered. 1.

Which of the following statements best describes the organization of the author’s discussion of the role of political knowledge in the formation of political ideology during adolescence? A. He acknowledges its importance, but then modifies his initial assertion of that importance. B. He consistently resists the idea that it is important, using a series of examples to support his stand. C. He wavers in evaluating it and finally uses analogies to explain why he is indecisive. D. He begins by questioning conventional ideas about its importance, but finally concedes that they are correct. E. He carefully refrains from making an initial judgment about it, but later confirms its critical role.

Analysis of the choices: You are asked to identify how the author organizes the passage. A. Does the author acknowledge the importance of political ideology in the beginning, and then modify his stand? Yes. In para 3 he says that he doesn’t want to overemphasize it.

56

Career Avenues

B.

Does he consistently resist the importance of political ideology? Not exactly. In the first two paras he says it is important. C. Does he waver in evaluating it? Is he irresolute? No. He makes assertive statements. D. Does he question conventional ideas and then concede to their accuracy? Not really? You could eliminate using the latter part of the statement. E. Does he refrain from making a judgment? No. Similar to choice C. Answer is choice A. IV. APPLICATION QUESTIONS These questions assess your ability to analyze and identify the scope of the content; to apply the information in the passage to a context outside the passage. For this you should be able to discern the relationship between situations or ideas presented by the author and an analogous hypothetical situation outside the passage. And hence apply the principle discussed in the passage in the outside situation. You may also be asked to identify among the choices, a situation analogous to the one discussed in the passage. Moreover, the question may require you to identify a statement (outside the passage) that the author would agree / disagree. The answers to most application questions are not what is given in the passage, instead they are like what you read in the passage. Common question stems: Ø It can be inferred from the passage that the author is most likely to agree / disagree with which of the following? Ø The passage suggests that if the new theory were to be true, which of the following would’ve been the case? Ø The author’s suggestion, if productive, would be most applicable to a situation in which……. ANSWERING SRATEGY Application questions are difficult to answer. These questions require that you scan the information in detail, study implications and connotations of the ideas in a sentence. The answer is the scope of the information. The answer is not a statement mentioned explicitly in the passage. Instead an outside statement constitutes the answer, which is indirectly meant in the passage. The following illustration will help you understand the question and the answering strategy. PASSAGE-1 Three historiographical considerations played an important role in my decision to write a history of Black women in the United States. First, I was persuaded that Black people and White people in the United States clearly represent two different cultures, with different traditions and often times diametrically opposed past experiences. The same may hold true for the two sexes, whose roles, history and contributions seem on close examination to be almost different enough to constitute different cultures. Of course, all groups share in the common history. I do not believe that recognizing the existence of large cultural subdivisions means succumbing to separatism, cultural nationalism, and a narrow particularistic vision. There is a place for universalistic interpretations of that, which is common to all humanity, and there is a place for that which is particular to one special entity. We accept that there are both a world history and a history of France or of India, histories that are not the same, but that fit into similar categories. Our historical perceptions can only be enriched by accepting the fact that, since sex and race are frequently used (however unfairly) to assign certain roles and statuses to people, these people have had different historical experiences from those of the dominant group. Second, I became convinced that standard histories of the United States have tended to overlook the contributions and viewpoints of Black Americans and women. I hope and expect that the present generation of United States historians will rectify this mistake, recognizing that women and Blacks were there and that their special contributions to the building and shaping of American society were different from those of White men. In the meantime, a separate history of Black women is needed. Finally, I had to consider the role that a White historian can legitimately play in the writing of Black history. Certainly, historians who are members of the culture about which they write will bring a special quality to their

57

Career Avenues

material. Their understandings are apt to be different from those of an outsider. But scholars from outside a culture have frequently had a view that contrasts with the view of those closely involved in and bound by their own culture. These angles of vision are complementary in arriving at an accurate picture of the past. The interpretation of the Black past made by Blacks will probably be somewhat different from that made by Whites. This does not mean that Black history can or should be interpreted only by Blacks. It should be interpreted by both groups, so that, in the juxtaposition of different interpretations, in debate, and in the clash of opinions, a richer and fuller and more solidly based history will emerge. 1.

The author’s recommendation for achieving the best possible interpretation of the Black past indicates that, if the staff of a government commission were collecting information about a controversial issue, the author would be most likely to approve of which of the following procedures? A. The staff does historical research on the issue and presents the results to the commission. B. The staff interviews people on both sides of the issue and present their differing views to the commission. C. The staff interviews people on both sides of the issue, finds points of agreement in their views, and presents these points to the commission. D. The staff interviews people on both sides of the issue, decides which viewpoint is more widespread, and presents that viewpoint to the commission. E. The staff interviews only people who are known not to have strong opinions about the issue and presents their viewpoints to the commission.

Analysis of the question: To answer the above question you should’ve understood the scope of the last paragraph of the passage. Without understanding the implications of the last part of the passage, you won’t be able to choose a choice confidently. What does the last part of the passage imply? It implies that both Blacks as well as Whites should write about the achievements of Blacks, so that complementary visions and their juxtapositions would lead to a more objective, fuller interpretation. A similar statement is made in choice B. Choice B states that both views are presented to a third party for debate. Therefore B is the right choice. A. C. D. E.

mentions historical research, there is no room for debate, hence wrong. ‘find points of agreement’ by researchers themselves. The analysis could be subjective. deciding on widespread view is not a sound way of analysis. Statistical significance may not conform to ‘qualitative’ interpretation. eliminate this; highly subjective way of collecting data.

Let’s study another example: PASSAGE-2 During adolescence, the development of political ideology becomes apparent in the individual; ideology here is defined as the presence of roughly consistent attitudes, more or less organized in reference to a more encompassing, though perhaps tacit, set of general principles. As such, political ideology is dim or absent at the beginning of adolescence. Its acquisition by the adolescent, in even the most modest sense, require the acquisition of relatively sophisticated cognitive skills, the ability to manage abstractness, to synthesize and generalize, to imagine the future. These are accompanied by a steady advance in the ability to understand principles. The child’s acquisition of political knowledge also promotes the growth of political ideology during adolescence. By knowledge I mean more than the dreary “facts,” such as the composition of county government, that the child is exposed to in the conventional ninth-grade civics course. Nor do I mean only information on current political realities. These are facets of knowledge, but they are less critical than the adolescent’s absorption, often unwitting of a feeling for those many unspoken assumptions about the political system that comprise the common ground of

58

Career Avenues

understanding – for example, what the state can “appropriately” demand of its citizens, and vice-versa, or the “proper” relationship of government to subsidiary social institutions, such as the schools and churches. Thus, political knowledge is the awareness of social assumptions and relationships as well as of objective facts. Much of the naïveté that characterizes the younger adolescent’s grasp of politics stems not from an ignorance of “facts” but from an incomplete comprehension of the common conventions of the system, of what is and is not customarily done, and of how and why it is or is not done. Yet I do not want to overemphasize the significance of increased political knowledge in forming adolescent ideology. Over the years I have become progressively disenchanted about the centrality of such knowledge and have come to believe that much current work in political socialization, by relying too heavily on its apparent acquisition, has been misled about the tempo of political understanding in adolescence. Just as young children can count numbers in series without grasping the principle of ordination, young adolescents may have in their heads many random bits of political information without a secure understanding of those concepts that would give order and meaning to the information. Like magpies, children’s minds pick up bits and pieces of data. If you encourage them, they will drop these at your feet – Republicans and Democrats, the tripartite division of the federal system, perhaps even the capital of Massachusetts. But until the adolescent has grasped the integument function that concepts and principles provide the data remain fragmented, random, disordered. 1.

The passage suggests that, during early adolescence, a child would find which of the following most difficult to understand? A. A book chronicling the ways in which the presidential inauguration ceremony has changed over the years. B. An essay in which an incident in British history is used to explain the system of monarchic succession. C. A summary of the respective responsibilities of the legislative, executive and judicial branches of government. D. A debate in which the participants argue, respectively, that the federal government should or should not support private schools. E. An article detailing the specific religious groups that founded American colonies and the guiding principles of each one.

Analysis of the question: You can infer from the entire discussion in the passage that adolescents do not possess sophisticated cognitive skills, such as abstract thinking, ability to generalize or to imagine the future. (discussed especially in para 1) A. B. C. D. E.

Deals with factual data and adolescents do understand facts. System of monarchic succession; again factual in nature; adolescents understand this. Responsibilities of branches of government deals with facts. A debate in which participate argue on an idea. This requires skills of evaluating, imagining the future; children do not understand it. Specific religious groups – factual information.

The answer is choice D. V. INFERENCE QUESTIONS These questions ask about ideas not directly mentioned in the passage. Instead, they are connotation of the passage; they are strongly implied. You infer from what is stated in the passage. Authors make their points mostly indirectly. Questions based on those indirect points are inferential; they are, in fact, suggested. To answer these questions, you’ve to go just one step ahead of the passage; go beyond their literal meanings, to recognize a plausible cause or effect / consequence. When you read the passage, you have to scan the text for subtle implications.

59

Career Avenues

By stating x, the author may be making a statement about y. paraphrase of a given statement.

In most inference questions, the answer is a

The question stems are: Ø Which of the following could be most plausibly inferred from the passage? Ø It can be inferred from the passage that a Victorian writer would most probably have approved of…. Ø The passage suggests that the reactions of literary critics toward women writers would be... Ø It can be inferred from the passage that the author admires writers who… Ø If the statements in the passage are true, which of the following must also be true? Ø In the passage, the author anticipates which of the following as a possible objection to her argument? Study the inference questions that follow the passage below. PASSAGE-1 During adolescence, the development of political ideology becomes apparent in the individual; ideology here is defined as the presence of roughly consistent attitudes, more or less organized in reference to a more encompassing, though perhaps tacit, set of general principles. As such, political ideology is dim or absent at the beginning of adolescence. Its acquisition by the adolescent, in even the most modest sense, require the acquisition of relatively sophisticated cognitive skills, the ability to manage abstractness, to synthesize and generalize, to imagine the future. These are accompanied by a steady advance in the ability to understand principles. The child’s acquisition of political knowledge also promotes the growth of political ideology during adolescence. By knowledge I mean more than the dreary “facts,” such as the composition of county government, that the child is exposed to in the conventional ninth-grade civics course. Nor do I mean only information on current political realities. These are facets of knowledge, but they are less critical than the adolescent’s absorption, often unwitting of a feeling for those many unspoken assumptions about the political system that comprise the common ground of understanding – for example, what the state can “appropriately” demand of its citizens, and vice-versa, or the “proper” relationship of government to subsidiary social institutions, such as the schools and churches. Thus, political knowledge is the awareness of social assumptions and relationships as well as of objective facts. Much of the naïveté that characterizes the younger adolescent’s grasp of politics stems not from an ignorance of “facts” but from an incomplete comprehension of the common conventions of the system, of what is and is not customarily done, and of how and why it is or is not done. Yet I do not want to overemphasize the significance of increased political knowledge in forming adolescent ideology. Over the years I have become progressively disenchanted about the centrality of such knowledge and have come to believe that much current work in political socialization, by relying too heavily on its apparent acquisition, has been misled about the tempo of political understanding in adolescence. Just as young children can count numbers in series without grasping the principle of ordination, young adolescents may have in their heads many random bits of political information without a secure understanding of those concepts that would give order and meaning to the information. Like magpies, children’s minds pick up bits and pieces of data. If you encourage them, they will drop these at your feet – Republicans and Democrats, the tripartite division of the federal system, perhaps even the capital of Massachusetts. But until the adolescent has grasped the integument function that concepts and principles provide the data remain fragmented, random, disordered. 1.

It can be inferred from the passage that the author would be most likely to agree with which of the following statements about schools? A. They should present political information according to carefully planned, schematic arrangements. B. They themselves constitute part of a general sociopolitical system that adolescents are learning to understand. C. If they were to introduce political subject matter in the primary grades, students would understand current political realities at an earlier age. D. They are ineffectual to the degree that they disregard adolescent political naïveté. E. Because they are subsidiary to government, their contribution to the political understanding of adolescents must be limited.

60

Career Avenues

Analysis of the question: The answer to this question is a paraphrase of the statement already cited in the passage. Let’s see what statement the author makes about schools. In para 2, the author states that adolescents try to absorb proper relationship of government to subsidiary social institutions, such as schools and churches. You have to recognize a paraphrase of this statement among the choices. A careful analysis of the choices reveals that each of the choices except B discusses ideas unrelated to the above statement. Choice B is the best answer. Subsidiary institutions mean part of the parental system (here society). Let’s answer one more question of this type: PASSAGE-2 Three historiographical considerations played an important role in my decision to write a history of Black women in the United States. First, I was persuaded that Black people and White people in the United States clearly represent two different cultures, with different traditions and often times diametrically opposed past experiences. The same may hold true for the two sexes, whose roles, history and contributions seem on close examination to be almost different enough to constitute different cultures. Of course, all groups share in the common history. I do not believe that recognizing the existence of large cultural subdivisions means succumbing to separatism, cultural nationalism, and a narrow particularistic vision. There is a place for universalistic interpretations of that, which is common to all humanity, and there is a place for that which is particular to one special entity. We accept that there are both a world history and a history of France or of India histories that are not the same, but that fit into similar categories. Our historical perceptions can only be enriched by accepting the fact that, since sex and race are frequently used (however unfairly) to assign certain roles and statuses to people, these people have had different historical experiences from those of the dominant group. Second, I became convinced that standard histories of the United States have tended to overlook the contributions and viewpoints of Black Americans and women. I hope and expect that the present generation of United States historians will rectify this mistake, recognizing that women and Blacks were there and that their special contributions to the building and shaping of American society were different from those of White men. In the meantime, a separate history of Black women is needed. Finally, I had to consider the role that a White historian can legitimately play in the writing of Black history. Certainly, historians who are members of the culture about which they write will bring a special quality to their material. Their understandings are apt to be different from those of an outsider. But scholars from outside a culture have frequently had a view that contrasts with the view of those closely involved in and bound by their own culture. These angles of vision are complementary in arriving at an accurate picture of the past. The interpretation of the Black past made by Blacks will probably be somewhat different from that made by Whites. This does not mean that Black history can or should be interpreted only by Blacks. It should be interpreted by both groups, so that, in the juxtaposition of different interpretations, in debate, and in the clash of opinions, a richer and fuller and more solidly based history will emerge. 1.

The author implies that which of the following, if they existed, would be most likely to make a separate history of Black women in the United States unnecessary? A. Standard histories of the United States that include information about American women in their appendices. B. Comprehensive histories of the United States that include the contributions and viewpoints of women and Blacks. C. Universalistic histories of the United States that emphasize the common experiences of men and women. Whites and minorities. D. Collections of articles written by Blacks and Whites that contain similar interpretations of Black history in the United States. E. Collections of articles written by Black women that present their interpretations of their own history in the United States.

61

Career Avenues

Question analysis: We can convert and translate this question in our own words. What is one of the reasons why the author is writing this book? Let’s refer to the passage: In the second paragraph, the author presents part of her thesis and ends with a statement ‘a separate history of Black women is needed’. The preceding statements should provide the basis for the need. The author says that she’s convinced that ‘standard histories of the United States have ‘ignored’ the contributions of Black Americans and women; therefore the need for a separate history. Now which of the choices ‘redefines and restates this idea?’ The answer is choice B. If comprehensive histories of United States included the contributions and viewpoints of women and Blacks, there was no necessity to write a separate history. VI. EVALUATION QUESTIONS There are questions that require you to evaluate the information given in the passage, or the author’s arguments. It is a higher order question which tests your ability to analyze and evaluate ideas. You have to judge the validity of the arguments. The common question stems are: Ø The author’s argument that………… would have been more persuasive, if he did which of the following? Ø As described in the passage, the author’s argument against…… would be more compelling, if he provided which of the following as premises? Ø Which of the following additional information is required for the author’s argument to be accepted as valid? ANSWERING STRATEGY A close analysis and scrutiny of the information in the passage is a prerequisite to answer this question. Have an open mind and attack the passage and the author’s arguments in a logical way. For instance, if the author is presenting a hypothesis you should study the logical base of the hypothesis. And if he proposes a new theory or proposition, you must scrutinize the evidences to determine if those evidences are convincing enough. Now let’s answer a question: PASSAGE The behavioral school of psychologists believes that all learning is responses to stimuli. And that all learning should be assessed through definable responses. For behaviorists the concept of ‘size’ among children for example, is assessed by cuing the child to respond to questions pertaining to size; if a child is unable to respond to the stimulus, the child is assessed as not having developed the concept of size. However, for Piaget, this is a ‘mechanical’ view of the behaviorists. The concept of ‘size’, among children, Piaget says, is one dimension of an array of interrelated images (mental images) : Covert responses can be expressed only with the image of all other concepts, say, length, height, weight etc. All these contribute toward a child’s response to

62

Career Avenues

say, the expression of size. Difficulties arise only if a child is unable to express a concept through a response to a single stimulus without the other changes. In such a case, Piaget argues, that any dimension of concept formation can be assessed by providing clues on other related dimensions, in order to make the child respond covertly to achieve the fundamental requirement of assessment through response to stimulus, even if one has to present tangible examples of other conceptual dimensions. As described in the passage, Piaget’s specific argument against behaviorists would be most strengthened, if he did which of the following? Provided evidence that many concepts are actually not formed in children. Explained why a child should be assessed as not developed the concept, if he fails to respond to a conceptual stimulus. Challenged the mechanism by which concept formation takes place in a child, through image formation. Mentioned other critics of behaviorism and the substance of their approaches. Gave an example of how a child can be prompted to respond to a stimulus by providing clues related to the concept tested. Analysis of the question: The question requires you to evaluate both the content and the author’s specific argument. We have understood from the passage that Behaviorists reject the notion that a child can have a concept without being able to respond to a stimulus involving the concept. However Piaget refutes this claim arguing that it is not true that a child doesn’t possess a concept even if he is not able to respond to cues related to the stimulus. The author’s argument would have been valid and acceptable if he gave a concrete example. Therefore the answer is E. Choice [A] is not true, as Piaget doesn’t believe that a child is not able to form concepts. [B] is not Piaget’s argument. [C] is not true as the passage does not discuss how concepts are formed in a child. [D] is absurd as mentioning other critics of Behaviorists doesn’t improve the logic of Piaget’s argument. VII. ASSUMPTION QUESTIONS You may also come across questions that require you to identify a certain presupposition that the author is making while making a statement, observation or conclusion. Obviously the answer is not given in the passage. The common stems are: Ø The author is making which of the following assumptions while referring to…..? Ø The author assumes which of the following in his discussion on………? Ø Which of the following is an assumption on which the author makes his recommendation? ANSWERING STRATEGY To answer this question you must first ascertain whether the statement is an assumption of a specific statement or one on which the entire argument is based. This can be gathered from the question stem itself. If it is based on a specific statement, locate the statement in the passage and connect this statement with the assumptions in the choices as follows: Assumption (choice), therefore statement (in the passage) The correct answer choice will fit in to this connection. Now let’s answer an assumption question.

63

Career Avenues

PASSAGE The evolution of the universe has always baffled astronomers. It is expected that during their lifetime the stars loose their mass and die in giant explosions. However astronomers are surprised to see that such a phenomenon is not occurring. Only two out of twenty-five stars die in such explosions. A group of scientists from the United States have observed and studied some stars to study the causes that lead to their death. One such study on a star named Noga shows that stars loose their mass slowly in what is referred to as ‘shedding weights’. Evidence of such shedding weights was observed among many stars. By studying the circum stellar material surrounding Noga, scientists have deduced the rate at which Noga loses mass, and how finally the star becomes diminutive and die. The circumstellar material consisted of infrared radiations and ammonia molecules. 1.

It can be inferred from the passage that the author assumes which of the following in the discussion of the rate at which Noga loses mass? A. The circumstellar cloud surrounding Noga consists only of ammonia molecules. B. The circumstellar cloud surrounding Noga consists of material expelled from the star. C. The age of a star is equal to its circumstellar cloud. D. The rate at which Noga loses mass varies significantly from year to year. E. Stars with great mass lose mass at a greater rate.

Analysis of the question: Let’s follow the same strategy explained above. The statement in the passage states that scientists deduced the rate at which Noga loses mass using the circumstellar material surrounding Noga. A. B. C. D. E.

True that the circumstellar cloud consists of ammonia, but it doesn’t imply that only ammonia is found. This assumption is made, otherwise scientists wouldn’t be using such material to assess the rate at which Noga loses mass. Choice B, therefore given statement. The purpose is to calculate the rate of mass loss. Irrelevant to the statements. No such comparison is made in the passage.

The answer is B. VIII. LOGICAL CONCLUSION QUESTION This question requires you to supply a conclusion to the passage or identify a statement that would logically conclude the passage. The common stems are: Ø Which of the following most logically conclude the last paragraph of the passage? Ø Which of the following is an appropriate statement to begin the paragraph that immediately follow the last part of the passage? Ø Which of the following conclusions is best supported by the passage? Now let’s answer such a question: PASSAGE Medieval alchemists found in the end that they could not create gold. Modern geochemists have a similar problem. They find it hard to understand how natural gold deposits form. There is much hand waving about gold-rich fluids from deep in the earth and chemical precipitation, but the physics does not add up. The answer may be that what is happening is not geo-chemical at all, but biochemical. And a casual experiment by a bacteriologist may hold the key.

64

Career Avenues

Derek Lovley, of the University of Massachusetts, Amherst, has been studying metal-eating bacteria for two decades. These bacteria make their living by converting (or reducing) the dissolved ions of metallic elements from one electrical state to another. This reduction releases energy, which the bacteria extract for their own purposes. Unsurprisingly, such bacteria tend to prefer common metals such as iron and manganese for lunch, though some species are able to subsist on such exotica as uranium. A few months ago, though, as a bit of a lark, Dr. Lovley decided to put some of his bacteria into a solution of gold chloride. He was fully prepared for nothing to happen, as gold compounds are generally toxic to bacteria. Instead, the test tube containing the solution turned a beautiful shade of purple, the colour of metallic gold when it is dispersed very finely in water. Bacteria are already known to be involved in the formation of an iron ore called limonite, and Dr. Lovley has argued that they are also involved in the creation of certain ores of uranium. His jokey experiment, reported in the July issue of Applied and Environmental Microbiology, opens up the possibility that gold deposits, too, may have a bacterial origin. 1.

Which of the following statements most logically conclude the last paragraph of the passage? A. Microbes can act as agents that concentrates gold from sources such as volcanic springs into a form that people can mine. B. Gold deposits have a bacterial origin and these bacteria can create ores. C. All ores of common metals are formed by bacterial action over a long period of time. D. The experiments conducted by Lovely et al do not arrive at any conclusive evidences about the action of bacteria in fossil formation. E. Most bacteria prefer to feed on iron and manganese rather than on uranium.

Analysis of the question: The question requires you to draw a conclusion from the last part of the passage. Your answer should logically follow the last statements of the passage. The answer is A. It logically follows the last few statements. Choice (B) is what is already given in the preceding paragraph. Choice (C) is beyond the scope of the passage. Choice (D) contradicts and underestimates the experiments reported in the passage. Choice (E) is a repetition of the data given in the passage. IX. ATTITUDE / TONE QUESTIONS These questions ask about the language of a passage and about the ideas that may be expressed through the language. You may be asked to deduce the author’s attitude towards an idea, a fact, or a situation from the words he or she uses to describe it. You may also be asked to select a word that precisely describes the tone of the passage. Based on the tone of the passage, you’ll be able to answer other question types such as – what audience the passage was intended for, what type of publication it probably appeared in. Common question stems: Ø In discussing………. the author’s tone is Ø Which of the following best describes the author’s attitude toward... Ø The passage suggests that the attitude of Victorian writers to literary critics was most probably one of... Ø The author regards the idea that all creative activity transcends limits with... Ø The author’s treatment of the subject matter of the passage can best be described as... ANSWERING STRATEGY

65

Career Avenues

In order to answer attitude/tone/style questions you should scan the reading text carefully. Every passage should be read for two aspects: Ø the matter of the passage, and Ø the author’s view the author’s statements expressing his approval / disapproval of ideas, incidents etc, may be embedded in the run-on matter. You should be able to differentiate such emotional content from factual content. You may be asked to recognize the attitude of the author toward or his style of writing of content as a whole or to specific situations or facts. For attitude questions, the answer is most likely the synonym of a word that expresses attitude or one word for a group of words. Let’s answer a question on attitude: PASSAGE-1 Three historiographical considerations played an important role in my decision to write a history of Black women in the United States. First, I was persuaded that Black people and White people in the United States clearly represent two different cultures, with different traditions and often times diametrically opposed past experiences. The same may hold true for the two sexes, whose roles, history and contributions seem on close examination to be almost different enough to constitute different cultures. Of course, all groups share in the common history. I do not believe that recognizing the existence of large cultural subdivisions means succumbing to separatism, cultural nationalism, and a narrow particularistic vision. There is a place for universalistic interpretations of that, which is common to all humanity, and there is a place for that which is particular to one special entity. We accept that there are both a world history and a history of France or of India, histories that are not the same, but that fit into similar categories. Our historical perceptions can only be enriched by accepting the fact that, since sex and race are frequently used (however unfairly) to assign certain roles and statuses to people, these people have had different-historical experiences from those of the dominant group. Second, I became convinced that standard histories of the United States have tended to overlook the contributions and viewpoints of Black Americans and women. I hope and expect that the present generation of United States historians will rectify this mistake, recognizing that women and Blacks were there and that their special contributions to the building and shaping of American society were different from those of White men. In the meantime, a separate history of Black women is needed. Finally, I had to consider the role that a White historian can legitimately play in the writing of Black history. Certainly, historians who are members of the culture about which they write will bring a special quality to their material. Their understandings are apt to be different from those of an outsider. But scholars from outside a culture have frequently had a view that contrasts with the view of those closely involved in and bound by their own culture. These angles of vision are complementary in arriving at an accurate picture of the past. The interpretation of the Black past made by Blacks will probably be somewhat different from that made by Whites. This does not mean that Black history can or should be interpreted only by Blacks. It should be interpreted by both groups, so that, in the juxtaposition of different interpretations, in debate, and in the clash of opinions, a richer and fuller and more solidly based history will emerge. 7.

In discussing the likelihood that “the present generation of United States historians will rectify this mistake” (para 2), the author’s tone is A. arrogant B. sentimental C. optimistic D. ironic E. apologetic

Analysis of the question: This question is specific to a situation. In para 2 the author expresses optimism (choice B). The statement in the passage is ‘I hope and expect’ that the present generation of historians will rectify this mistake. Now let’s answer a question on style of writing:

66

Career Avenues

PASSAGE - 2 The behavioral school of psychologists believe that all learning is responses to stimuli. And that all learning should be assessed through definable responses. For behaviorists the concept of ‘size’ among children for example, is assessed by cuing the child to respond to questions pertaining to size; if a child is unable to respond to the stimulus, the child is assessed as not having developed the concept of size. However, for Piaget, this is a ‘mechanical’ view of the behaviorists. The concept of ‘size’, among children, Piaget says, is one dimension of an array of interrelated images (mental images) : Covert responses can be expressed only with the image of all other concepts, say, length, height, weight etc. All these contribute toward a child’s response to say, the expression of size. Difficulties arise only if a child is unable to express a concept through a response to a single stimulus without the other changes. In such a case, Piaget argues, that any dimension of concept formation can be assessed by providing clues on other related dimensions, in order to make the child respond covertly to achieve the fundamental requirement of assessment through response to stimulus, even if one has to present tangible examples of other conceptual dimensions. 8.

The author’s treatment of the subject matter can best be described as A. a biased exposition B. an ambivalent rebuttal C. a dispassionate presentation D. an enthusiastic endorsement E. a hypothetical proposal

Analysis of the question: The question asks you to describe the author’s treatment of the topic. You must have gathered that the author presents Behaviourists’ assumptions and Piaget’s counter proposal to those assumptions. The author merely presents two opposing points of view, and refrains from making any judgment. Thus it is an objective (dispassionate) presentation. Here are some of the oft-used attitude words. Objective assessment

: (dispassionate/ disinterested presentation) The author reacts to a piece of work not influenced by personal feelings or prejudice. Assessing a piece of work based on its own inherent reality than by the evaluators whims and fancies. Judgment done through the intrinsic criteria of the work itself.

Biased interpretation

: (subjective/ prejudiced) An author’s opinion or feeling that strongly favours one side of an argument, sometimes unfairly.

Skepticism

: This would imply criticism, doubt or questioning the logic of an argument. This expresses the author’s disagreement with a hypothesis, idea, proposition, finding of a study and the like.

Pointed disagreement

: Sharp criticism/ unrelenting criticism; the author disagrees with an idea completely. There will be explicit statements in the passage denoting this.

Qualified admiration

: Limited/ modified/ restricted endorsement. Here the author is quite specific in his approval; he admires the person/idea discussed for certain qualities and may be critical of certain other aspects.

Grudging respect/approval

: This is characterized by an unwillingness from the part of the author to concede completely to the idea; he expresses his opinion reluctantly.

67

Career Avenues

Optimism / hope

: The author would express hope and expectation in say, solving a problem, implementing a recommendation.

Enthusiastic endorsement

: The author wholeheartedly approves of the idea and/ or supports the findings, recommendations or propositions.

Eulogy

: Extreme praise for the author being reviewed or person or idea under discussion.

Satirical

: The author expresses scorn/ridicule/ derision; he laughs at the matter under discussion.

Ironical

: The author uses words to convey the opposite meaning of what he expresses. One thing is said and its opposite is implied.

X. SOURCE OF THE PASSAGE An occasional question in which you have to infer from the nature of the content matter, the source of the passage or publication it probably appeared in. A tricky question, no doubt. However, an intelligent reader, one who is familiar with varied texts will not find it difficult to crack. Given the research based background of most of the passages, the subtle differences as their sources add to the difficulty. The questions stems are: Ø The passage most likely appeared as... Ø By its nature and content it can be inferred that the passage was most probably part of... Ø The passage is most probably drawn from, which of the following sources? ANSWERING STRATEGY Is it possible for us to infer the source from which the passage is drawn? Of course, yes. The following passage and the question will elucidate this. Let’s look at a question: PASSAGE - 1 Three historiographical considerations played an important role in my decision to write a history of Black women in the United States. First, I was persuaded that Black people and White people in the United States clearly represent two different cultures, with different traditions and often times diametrically opposed past experiences. The same may hold true for the two sexes, whose roles, history and contributions seem on close examination to be almost different enough to constitute different cultures. Of course, all groups share in the common history. I do not believe that recognizing the existence of large cultural subdivisions means succumbing to separatism, cultural nationalism, and a narrow particularistic vision. There is a place for universalistic interpretations of that, which is common to all humanity, and there is a place for that which is particular to one special entity. We accept that there are both a world history and a history of France or of India, histories that are not the same, but that fit into similar categories. Our historical perceptions can only be enriched by accepting the fact that, since sex and race are frequently used (however unfairly) to assign certain roles and statuses to people, these people have had different historical experiences from those of the dominant group. Second, I became convinced that standard histories of the United States have tended to overlook the contributions and viewpoints of Black Americans and women. I hope and expect that the present generation of United States historians will rectify this mistake, recognizing that women and Blacks were there and that their special contributions to the building and shaping of American society were different from those of White men. In the meantime, a separate history of Black women is needed.

68

Career Avenues

Finally, I had to consider the role that a White historian can legitimately play in the writing of Black history. Certainly, historians who are members of the culture about which they write will bring a special quality to their material. Their understandings are apt to be different from those of an outsider. But scholars from outside a culture have frequently had a view that contrasts with the view of those closely involved in and bound by their own culture. These angles of vision are complementary in arriving at an accurate picture of the past. The interpretation of the Black past made by Blacks will probably be somewhat different from that made by Whites. This does not mean that Black history can or should be interpreted only by Blacks. It should be interpreted by both groups, so that, in the juxtaposition of different interpretations, in debate, and in the clash of opinions, a richer and fuller and more solidly based history will emerge. 1.

The passage most likely appeared as A. part of the preface to a scholarly book. B. a rebuttal to a previously published scholarly article. C. part of a book review in a scholarly journal. D. a condensed version of an article in a scholarly journal E. a complete entry in a encyclopedia of the social sciences.

Analysis of the question: You observe that the author begins with an admission of the three reasons (bases) why she is writing this book. And the subsequent paragraphs discuss each of these three reasons in detail. You can infer from this that the article is the ‘author’s own’. And most plausibly the reasons for writing a book are given in the preface (introduction) to a book (preface is always written by the author himself presenting his aim of writing the book). Therefore the answer is A. B. C. D. E.

A rebuttal (prove something as false) should be done somebody other than the author; therefore first person references (I, me myself) is not part of it. A book review is done by somebody other than the author; therefore third person references (he, him) should be part of it. An article is usually on topic; an article is unlikely to be on the authors reasons for writing a book. An encyclopedic entry has only factual information, definitions that are proven scientifically.

Here’re some of the common specific sources that you see in the choices of such questions. 1.

Foreword: A short introduction to a book, printed in the beginning and usually written by a person other than the author. How to spot it: Ø Second person reference (he/they); ‘they’ in case there is a co-author. Ø There won’t be any negative comments.

2.

Preface: As mentioned earlier, written by the author stating his aim. You will see first person reference (I/me) and not third person reference (he/she)

3.

Book review: Written by a person other than the author; denotes judgment of the book. Ø could’ve approval / criticism / admiration etc. Ø third person reference Ø title of the book mentioned (usually)

4.

An article in a science journal: Mostly research reports; research analysis, hypothesis, newly proposed theory. Ø discussion of research done by different scientists, deals with one of the pure sciences. Ø can be written by the researchers themselves, or by other experts.

69

Career Avenues

5.

Excerpts / Extracts from a book on …: A passage from a book. (Fiction or non-fiction). Ø either first person or second person reference is possible; therefore the choice would contain ‘excerpt from what’. Going by the source of the excerpt you’ will be able to choose the choice.

6.

Synopsis / Abstract of a research report: Summary of a research report will contain the hypotheses of the study, sample, analysis and findings (sometimes scope / field application also)

7.

Compendium: An article that discusses many items of information.

8.

A newspaper article: Reference to contemporariness; mostly social/political issues; current affairs / or responses to other articles.

9.

An article from a social science / art journal: New policy debates / socio- political issues, new findings in historical research, evaluation of art/ painting /films / contributions of personalities / economic reforms.

10.

Part of an essay: Points of view around on topic / issue; do not contain judgment of research / book.

COMPREHENSION TESTS In the following pages, you are provided with reading material for extensive practice. In each of the Tests you are presented with passages from the sources mentioned in the beginning of the reading section. The questions simulate CAT standard of difficulty and variety of questions discussed in the previous section. However, some passages are longer than the actual test passages. The objective is to expose you to possible expected changes in the CAT. Do one Test at a time and evaluate your performance. Closely watch your performance. If the performance in the abstract passages is not satisfactory, do remedial reading. Examine the type of questions you tend to make mistakes; review the strategies for those question types for which that errors persist.

70

Career Avenues

READING COMPREHENSION TESTS

71

Career Avenues

Test 1 DIRECTIONS: After each passage you will find a series of questions. Select the best answer choice for each of the questions. Answer the questions based on what is stated or implied in the passage. PASSAGE-1 (PURE SCIENCE) If a biographer wanted to write about a spectacular comeback from notoriety to respectability, and the tenuous natures of comebacks, a good subject might be nicotine. After denouncing nicotine for its highly addictive properties, which hook people on tobacco, scientists have begun to rehabilitate it, not only as a means of helping smokers to break their habit, but also as a potential treatment for a variety of other disorders, including stress, ulcerative colitis, Alzheimer’s, Parkinson’s etc. Nicotine replacement products developed to help people quit smoking used to consist only of chewing gum and skin patches, but now a nasal spray is available by prescription, as well as an inhale, which is really a fake cigarette made of plastic that a person sucks to get a dose of menthol-flavored nicotine into the mouth and throat, but not the lungs. Doctors are awaiting the approval of a nicotine tablet that patients are to dissolve under their tongues to help fight their craving. In all these products, the nicotine is extracted from tobacco leaves. But just as nicotine approaches respectability, a few researchers have begun to raise caution flags. Although all agree that replacement nicotine is safer than smoking, and worth some risk if it helps a person quit, a few are beginning to worry that some smokers are using nicotine products not just for the short periods the products were designed for, but for years. They fear this use will only increase if nicotine products proliferate and if more become available over the counter, as the patches and gum already are. And if nicotine or drugs chemically related to it do come into use to treat chronic diseases, the researchers say, there may be important side effects to take into consideration. 1.

The opening sentence of the passage implies which of the following? A. x is known to cure y; but x is found to cure z also. B. x is earlier known to have negative effects, but now seems to have positive effects. C. x, a known thing possesses both negative and positive effects. D. known properties of x, has the same effects as the unknown properties of y. E. x and y, used for positive effects are found to have negative side effects too.

2.

The author organizes the passage by A. introducing a new proposition, discussing the proposition in detail, and concluding with its applications. B. defining a hypothesis,discussing the implications of the hypothesis and suggesting methods to counter the hypothesis. C. pointing out the uses of a known material, explaining how it is used in a certain situation and then cautioning about possible negative consequences. D. describing a problematic situation, enumerating factors contributing to the situation, and puting forth possible solutions to confront the situations. E. pointing out a controversy, discussing both sides of the argument and summarizing the negative side of the argument.

3.

If the assertions in the last paragraph are true, which of the following can be inferred? A. There is no permanent cure for ailments that are inherited. B. Scientists fail to foresee the problematic consequences of a drug while developing it. C. The side effects of a drug manifests only after a period of use of the drug. D. A drug that cures an illness and causes other illnesses for which there is no immediate cure. E. A drug used to cure an ailment may have a reverse effect.

4.

Following the information in the passage, a rehabilitation counselor would recommend nicotine to break the habit of smoking with A. defiance B. caution C. bias D. concern E. consideration

72

Career Avenues

PASSAGE-2 (HUMANITIES) Simonton emphasized that emotional stress has two principal effects. It suppresses the body’s immune system and, at the same time, leads to hormonal imbalances that result in an increased production of abnormal cells. The production of malignant cells is enhanced precisely at a time when the body is least capable of destroying them. The basic philosophy of the Simonton approach affirms that the development of cancer involves a number of interdependent psychological and biological processes, that these processes can be recognized and understood. To do so, the Simontons help their patients to become aware of the wider context of their illness, identify the major stresses in their lives, and develop a positive attitude about the effectiveness of the treatment and the potency of the body’s defenses. Once feelings of hope and anticipation are generated, Simonton explains, the organism translates them into biological processes that begin to restore balance and to revitalize the immune system. As I listened to Carl Simonton, I realized with great excitement that he and Stephanie were developing a therapeutic approach, which could become exemplary for the entire holistic movement. It is a multidimensional approach involving various treatment strategies – conventional medical treatment, visualization, psychological counseling, and others – all of which are designed to initiate and support the organism’s innate psychosomatic process of healing. Their psychotherapy, which usually takes place in-group sessions, concentrates on the patients’ emotional problems but does not separate these from the larger patterns of their lives, and thus generally includes social, cultural, philosophical, and spiritual aspects. After the Simontons’ lectures it was clear to me that both of them would be ideal guides for my future explorations of health and healing, and I resolved to remain in contact with them as much as possible. 5.

The author of the passage is most likely to agree with which of the following? A. Recovery is essentially a biological process. B. The factors that contribute to the development of cancer cannot be fully construed. C. Emotional training of the patient helps in attaining biological recovery. D. Awareness of illness in one’s body is detrimental to the mental health of the patient. E. Psychotherapy is not done individually as a social setting contributes to early recovery.

6.

Which of the following can be inferred about the author of the passage? A. A psychotherapist B. A critic of Simonton approach C. A researcher in holistic healing D. A medical practitioner E. A biologist interested in mental health.

7.

The basic tenets of Simonton approach involve which of the following? I. Development of cancer is triggered by many psychosomatic factors. II. Human immune system is susceptible to stressful life events. III. Hormonal imbalance is caused by abnormal cells. A. III only B. II only C. I and III D. I and II

8.

E. I, II and III

Which of the following can be inferred from the passage regarding recovery of a patient from cancer? A. It is not a one-dimensional healing process. B. Visualization is integral to mental health. C. Once lost, it is not possible for the human body to bring back the immune system. D. The speed of recovery from cancer depends much on the type of medical treatment given. E. Psychotherapists find it difficult to induce hope and anticipation among Patients.

73

Career Avenues

PASSAGE-3 (ART) Jack Lemmon and Walter Matthau were the great comic pair of late 20th century Hollywood. Comic pairs are a tradition going back to Bud Abbott and Lou Costello (in B A and L C meet the mummy) followed by the inimitable duo of Laurel and Hardy (Chumps at Oxford, et al, with the latter inevitably sporting an aggrieved look on his face and telling the former “A fine mess you’ve got us into again!”). Lemmon and Matthau brought in the sophistication of a Hollywood which had gone beyond slapstick. A Hollywood which had grown up into a maturity where humour was no longer pieces of cake being thrown at funny faces but where life itself provided the laughs in the form of day to day situations humorised by a pair of tough oddballs. Witness the movie version of Neil Simon’s The Odd Couple where Lemmon played the fastidious neatnik to Matthau’s slob. Remember Neil Simon was the only playwright other than one Wil Shakespeare to have three of his plays running simultaneously on Broadway! The Lemmon and Matthau show was last seen in Grumpy Old Men. Lemmon also did enough on his own to be enrolled on Hollywood’s walk of fame, winning his first Oscar for best supporting actor in 1955 as the dithering ship’s officer Ensign Puliver who finally stands up for what he believes in, thanks to a mentor played by Henry Fonda in Mister Roberts. The best actor Oscar came 18 years later for Mr. Tiger where Lemmon portrayed a clothing manufacturer desperately trying to save his business. Lemmon is often remembered for his portrayal of one of the two Chicago musicians on the run from the mob in Billy Wilder’s Classic Some Like It Hot, which also featured Tony Curtis, Marilyn Monroe and Joe E Brown. He became a favourite with Wilder who once said “Happiness is working with Jack Lemmon”. His portrayal in The Great Race of an eccentric European prince who insists that everyone laughs when he laughs and stops when he stops was hilarious. There were other unforgettable cinematic moments in Days of Wine and Roses, The Front Page and The China Syndrome. However, it is the partnership with Matthau which created a genre of Hollywood humour at its oddball best. Ergo, it was almost inevitable that when Matthau left the stage called world, Lemmon would follow him less than a year later. 9.

The author is primarily concerned with presenting A. an account of the origin and development of slapstick comedies in Hollywood films. B. a comparative analysis of the old school and the modern school of film making in the Hollywood system. C. a tribute to two of the best comedians of Hollywood who have left an indelible mark on cinema. D. a critique of the excesses of technique over content in the Hollywood system of film making. E. contradictory epistemologies of defining comedy – the Shakespearian and the Simonian.

10.

The passage provides answers to all of the following questions EXCEPT A. who besides Lemmon and Matthau have elevated comedy to sublime heights? B. did Lemmon and Matthau follow the comical tradition of their predecessors in their portrayals? C. was William Shakespeare the only play Wright whose plays ran simultaneously on Broadway? D. who other than Neil Simon is credited with having his plays run simultaneously on Broadway with the exception of Shakespeare? E. what are some of the similarities between slapstick comedies and situational comedies?

11.

Which of the following is one of the contributions of Lemmon and Matthau to Hollywood comedies? A. Lemmon and Matthau freed Hollywood films from the grip of studio system. B. Lemmon and Matthau gave new dimension to humour, through situational comedies. C. Lemmon and Matthau used asides and dialogues in comic scenes instead of facial expressions. D. Lemmon and Matthau experimented with costumes to add humour to situations in a plot. E. Lemmon and Matthau brought in sophistication in the treatment of subject matter and went beyond screen plays. T

74

Career Avenues

TEST 2 DIRECTIONS: After each passage you will find a series of questions. Select the best answer choice for each of the questions. Answer the questions based on what is stated or implied in the passage. PASSAGE-1 A fifth trend that affects research is the globalization of science. Today’s research happens in many locations, with many partners. For example, IBM has built a medical device research facility in Japan so that they can tailor products to the local market. Microsoft has placed its R&D unit in Cambridge, England to have access to the research in computer science at a leading university with a European perspective. This is different from outsourcing, where the goal is to do business more cheaply. The globalization of R&D is a richer concept, where the goal is to enhance the quality and scope of research by being in multiple locations. A recent paper in the American Scientist stated that scientists in all countries, including developing countries, gain considerable benefit from collaborating with scientists around the world. Creating international partnerships and networks is one of the goals of the Millennium Science Institutes. International collaboration also benefits researchers in the developed world, who need the talented colleagues and new points of view found in other nations. In a broader sense, the developed world urgently needs to encourage faster development of small economics to help narrow the gap between nations. There are large challenges that await us as we enter the new Millennium. We need a higher level of interaction between disciplines, but there are still significant barriers to interdisciplinary research. In the mathematics community, for example, we have a tradition of relative isolation. We are not only isolated internally within our sub fields, but from other fields of science and certainly from nonacademic areas – especially the private sector. It’s important to build more bridges not only within institutions, but also between them. For example, the cultures of academic research and the private sector are very different, so that few mathematics students consider careers in industry. In the United States, some 80 percent of new doctoral mathematicians anticipate academic positions. And yet many promising opportunities are found in fields where industry is very active, such as bioinformatics and communications technology. Traditional structure of universities works against interdisciplinary collaboration. The existence of physically separated departments of “applied mathematics” and “pure mathematics” has perpetuated a narrow view about which kinds of mathematics can or should be applied. This view has limited the application of all of mathematics to important practical problems. In conclusion, it is essential for institutions, supporting agencies, and all of us to understand the importance as well as the difficulties of interdisciplinary research, and to recognize that sustained support is needed to develop excellence. This requires good balance in research portfolios, with special attention to long-term programs. 1.

An appropriate title for the passage would be A. Linear model to a dynamic model. B. Disciplinary to interdisciplinary research. C. Study of complex systems D. Globalization and the diffusion of knowledge. E. Knowledge management – a new paradigm.

2.

The globalization of research and development sets all of the following goals except A. quality enhancement B. scope of research C. local needs D. access to local knowledge E. cost effectiveness

3.

The author cites the instance of relative isolation in mathematics community primarily in order to A. point out factors that hinder interdisciplinary research. B. show the lack of communication between mathematicians in developing and underdeveloped countries. C. expostulate the difference between the original subject and subsidiaries. D. discuss the extend to which private sectors outsource information from governmental undertakings. E. propose new methods to bring together the different branches of mathematics.

75

Career Avenues

4.

The author is of the opinion that a broader view of mathematics would’ve contributed to which of the following? A. Lack of differentiation between the sub-fields of mathematics. B. 80% job placements in private sectors such as bio-informatics and communication technology. C. Finding solutions for practical problems where in mathematical models apply. D. To unify the diverse cultures of academic research and the private sector. E. Many PhDs in mathematics opting for careers in applied mathematics.

5.

Which of the following reflects the author’s view of knowledge creation and dissemination? A. Secularist B. Particularistic C. Eclectic D. Pragmatic E. Dogmatic PASSAGE-2

About 23 million years ago, a huge ice sheet spread over Antarctica, temporarily reversing a general trend of global warming and decreasing ice volume. Now a team of researchers has discovered that this climatic blip at the boundary between the Oligocene and Miocene epochs corresponded with a rare combination of events in the pattern of the Earth’s orbit around the Sun. Researchers show the transient glaciations and other climatic variations in the Earth’s orbit known as Milankovitch cycles. Although the concept of such relationships is not new, some of the results were surprising. The astrophysicist Milutin Milankovitch first proposed that cyclical variations in certain elements of Earth-Sun geometry can cause major changes in Earth’s climate. The main variables are eccentricity, obliquity, and precession. Eccentricity refers to the changing shape of Earth’s orbit around the Sun, which varies from nearly circular to elliptical over a cycle of about 100,000 years. Obliquity refers to the angle at which Earth’s axis is tilted with respect to the plane of its orbit, varying between 22.1 degrees and 24.5 degrees over a 41,000 year cycle. And precession is the gradual change in the direction Earth’s axis is pointing, which completes a cycle every 21,000 years. Because there are several components of orbital variability, each with lower frequency components of amplitude modulation, there is the potential for unusual interaction between them on long timescales of tens of millions of years. The result of this rare congruence was a period of about 200,000 years when there was unusually low variability in the planet’s climate, with reduced extremes of seasonal warmth and coldness. Earth’s orbit was nearly circular, so its distance from the Sun stayed about the same throughout the year. In addition, the tilt of Earth’s axis, which gives rise to the seasons, varied less than usual. 6.

The primary purpose of the passage is to A. Discuss temperature fluctuations in the Antarctica. B. Question the validity of the cyclical hypothesis. C. Explain how earth-sun geometry affects climate. D. Propose a new method to estimate the age of the earth. E. Summarize research findings in astrophysics.

7.

If the assertions in the passage are true, which of the following must also be true? A. The shape of the Earth’s orbit around the Sun doesn’t remain constant. B. The Earth’s orbit around the sun remained elliptical most of the time in the history of Earth. C. The shape of the Earth’s orbit varies with the changes in the Earth’s axis. D. Unusual interactions between the components of orbital variability occur in hundreds of millions of years. E. It is impossible to predict the actual causes that result in the tilting of the earth’s axis.

76

8.

9.

Career Avenues

According to the passage, the factors that contribute to climatic change is/are I. The changing shape of earth’s orbit. II. The angle at which the earth’s axis is tilted. III. Lack of change in the directions the Earth’s axis is pointing. A. II only B. I and III C. II and III D. I and II

E. I, II, III

It can be inferred from the passage that, an unusual interaction between components of orbital variability results in which of the following? A. Maximum variation in climate. B. Extreme warmth or coldness. C. The tilting of Earth’s axis varied more than usual. D. Unusually low change in climatic conditions. E. The Earth’s orbits’ becoming elliptical. T

77

Career Avenues

TEST 3 DIRECTIONS: After each passage you will find a series of questions. Select the best answer choice for each of the questions. Answer the questions based on what is stated or implied in the passage. PASSAGE-1 Researchers at the University of Texas at Austin have used a new form of laser technology called single molecule spectroscopy to make important contributions to understanding the motion of molecules in super-cooled liquids, a problem of interest to scientists for more than half a century. The article announces both the discovery of a successful method for observing the movement of individual molecules as well as new insights into molecular motion in a material just before it turns into a glass. When liquids freeze to become solid, the molecules arrange themselves into organized structures. Super cooled liquids, in contrast, are cooled so fast that they go below freezing before the molecules have a chance to organize. If you lower the temperature of a super cooled liquid, the molecules move slower and slower until eventually the material is so cold that all motion has stopped. Then you have a different kind of solid – you have a glass, an amorphous solid. If you look at the motions of molecules in super cooled liquid, you discover that they do not look like the motions of molecules in regular liquid. The researchers focused on the rotation of molecules just before this glass transition. In a normal liquid, all the molecules rotate at the same rate, which means only one time scale is involved. The researchers wanted to find out what happens to the rotation of molecules in a super cooled liquid and whether all molecules in a super cooled form were rotating in the same way at the same time, or whether each molecule was rotating at a different rate. The researchers made refinements in previously used laser technology to isolate and illuminate a very bright orange molecule called Rhoda mine 6G. Because the method allowed molecules to be observed individually, the researchers were able to avoid the jumbling effect that masks what is really happening. What they discovered is that the rotations of individual molecules are all very different from each other. Some of the molecules are moving really fast, and some are really slow. In super-cooled liquid, there are regions within the liquid that are different from one another, researchers said. 1.

The author of the passage perceives research and understanding of the motion of molecules in super-cooled liquids as A. speculative B. controversial C. inconclusive D. confirmatory E. deductive

2.

The molecules of a super-cooled liquid move slower and eventually stops, under which of the following conditions? A. When super-cooled liquid is frozen. B. When the temperature is increased. C. When the temperature is lowered. D. When molecules organize into structures. E. When the liquid is cooled fast.

3.

Which of the following can be inferred from the passage about our understanding of super-cooled liquids? A. Laser technology was not used earlier to study the motion of molecules in super-cooled liquids. B. Previous researchers failed to observe the “jumbling effect” that hid what was really happening. C. The new single molecule spectroscopy permits isolation and observation of molecules individually. D. In super-cooled liquids, cooling takes place after the molecules become organized structures. E. Motion of molecules in super-cooled liquids continues at a uniform rate even at extreme low temperatures.

78

4.

Career Avenues

Which of the following regarding rotation of molecules can be inferred from the passage? A. Uniform rate of rotation of molecule is not observed in normal liquids. B. There is no differentiation in the rotation of molecules in a super-cooled liquid. C. The rotation of molecules appeared different with different methods used for observation of molecular motion. D. Only one time scale is required when the molecules rotate at different rules. E. There is no uniformity in the rotation of molecules in a super-cooled liquid. PASSAGE-2

Lamarck was the first to propose a coherent theory of evolution, according to which all living beings have evolved from earlier, simpler forms under the pressure of their environment. Although the details of the Lamarckian theory had to be abandoned later on, it was nevertheless the important first step. Several decades later Charles Darwin presented an overwhelming mass of evidence in favor of biological evolution, establishing the phenomenon for scientists beyond any doubt. He also proposed an explanation, based on the concepts of chance variation – now known as random mutation – and natural selection, which were to remain the cornerstones of modern evolutionary thought. Darwin’s monumental Origin of Species synthesized the ideas of previous thinkers and has shaped all subsequent biological thought. Its role in the life sciences was similar to that of Newton’s Principia in physics and astronomy two centuries earlier. The discovery of evolution in biology forced scientists to abandon the Cartesian conception of the world as a machine that had emerged fully constructed from the hands of its Creator. Instead, the universe had to be pictured as an evolving and ever-changing system in which complex structures developed from simpler forms. While this new way of thinking was elaborated in the life sciences, evolutionary concepts also emerged in physics. However, whereas in biology evolution meant a movement toward increasing order and complexity, in physics it came to mean just the opposite – a movement toward increasing disorder. 5.

Which of the following is a definition of evolution propounded by Lamarck? A. Evolution is a process of genetic mutation. B. Evolution is a process of natural selection. C. Evolution results from chance variation. D. Evolution is a result of influence of external factors. E. Evolution is a process of conservation.

6.

It can be inferred from the passage that ‘Origina of Species’ A. is independent of and different from earlier theories. B. is congruent to ‘Principia’. C. accommodated previous theories. D. has changed all subsequent theories. E. was the first theoretical work on evolution.

7.

It can be inferred from the passage that scientists began turning away from Cartesian model because A. Physics was moving toward disorder. B. Cartesian followers thwarted the emergences of new concepts in physics. C. the unity found among biologists from Lamarck to Darwin was nearly absent among physicists. D. there was no cornerstone in physics just as it was there in biology. E. Cartesian model of the world was more of a mechanistic paradigm incapable of evolution.

8.

It can be inferred from the passage that, the author is critical of A. Lamarck’s theory of evolution. B. chance variation as explained by Darwin. C. Newton’s theory of physics. D. the emergence of post-Darwinian biologists and their claims. E. development in physics post-Cartesian times. T

79

Career Avenues

TEST 4 DIRECTIONS: After each passage you will find a series of questions. Select the best answer choice for each of the questions. Answer the questions based on what is stated or implied in the passage. PASSAGE-1 Among the experiences that psychotic people fail to integrate, those relating to their social environment seem to play a crucial role. Recent major advances in the understanding of schizophrenia have been based on the recognition that the disorder cannot be understood by focusing on the individual patients, but has to be perceived in the context of their relations to other people. Numerous studies of families of schizophrenics have shown that the person who is diagnosed as being psychotic is, almost without exception, part of a network of extremely disturbed patterns of communication within the family. The illness manifested in the ‘identified patient’ is a really a disorder of the entire family system. The central characteristic in the communication patterns of families of diagnosed schizophrenics was identified by Gregory Bateson as a ‘double bind’ situation. Bateson found that the behaviour labeled schizophrenic represents a special strategy, which a person invents in order to live in an unlivable situation. Such a person finds himself facing a situation within his family that seems to put him into an untenable position, a situation in which he ‘can’t win,’ no matter what he does. For example, the double bind may be set up for a child by contradictory verbal and nonverbal messages, either from one or from both parents, with both kinds of messages implying punishment or threats to the child’s emotional security. When these situations occur repeatedly, the double-bind structure may become a habitual expectation in the child’s mental life, and this is likely to generate schizophrenic experiences and behaviour. This does not mean that everybody becomes schizophrenic in such a situation. What exactly makes one person psychotic while another remains normal under the same external circumstances is a complex question, likely to involve biochemical and genetic factors that are not yet well understood. In particular the effects of nutrition on mental health need much further exploration. 1.

It can be inferred from the passage that schizophrenia is A. a manifestation of the disturbed communication among the members of the family, on an individual. B. a psychosomatic condition in which the integrity between body and mind ruptures and thereby a disturbance created. C. a condition in which the immediate social system fails to welcome and integrate an individual to its fold. D. a defense mechanism by which an individual rejects the communication pattern of one of the parents. E. a biological breakdown in which the various systems of the body loses balance, thereby affecting the individual’s mental health.

2.

Which of the following defines the ‘double bind’ situation as discussed in para 2? A. A situation resulting in split personality. B. A situation in which a patient is confronted with conflicting life goals. C. A situation in which an individual is faced with a contradictory situation with the family. D. A situation in which a person is in conflict with his goal and the goal of his family members. E. A situation in which an individual is faced with an approach-approach conflict.

3.

The author of the passage is most likely to agree with which of the following observations about psychosis? A. There has been no concerted effort to analyze and understand psychosis. B. The reasons why individuals become psychotic do not lie in simplistic questions. C. Schizophrenia is directly linked more to genetic factors than to environmental factors. D. Psychosis is a condition arising solely from an individual’s environment. E. These has not been any significant research or development in our awareness of schizophrenia.

4.

In his statement, “the effects of nutrition on mental health need further exploration,” the author’s tone is A. hopelessness B. pessimism C. indifference D. matter-of-fact E. bitterness

80

Career Avenues

PASSAGE-2 The seeds of a transformation in agriculture - which could ultimately be comparable in importance to the original Green Revolution - are about to be sown in several countries. This international project, which is being coordinated by a pioneering team at Britain’s university of Nottingham, has nitrogen fixation at its core. However, what is especially interesting is that this is not the ‘typical’ nitrogen fixation associated with cleavers or Soya beans but the very real prospect of mainstream, non-leguminous crops such as rice or wheat being able to thrive with little, or no, artificial nitrogen. The search for non-leguminous crops able to exploit the value of nitrogen fixing rhizobia is nothing new. Almost as soon as the phenomenon of symbiotic nitrogen fixation was discovered in legumes, just over a century ago, researchers were trying to understand why other plants couldn’t do the same. However, it was not until the late 1960’s that researchers with more sophisticated equipment began to target the subject - encouraged by the knowledge that the production of artificial nitrogen utilizes alarming quantities of natural resources. Several options were explored, including the heart-transplant type transfer of genes from nitrogen fixing bacteria directly into the cells of a non-legume. Another possibility was to encourage the bacteria which occur naturally around plant roots to become more efficient at nitrogen fixation. In theory these super-efficient bacteria would then ‘feed’ the roots of the rice or wheat, without invoking the traditional nodule forming relationship associated with nitrogen fixation in legumes. Under the glare of laboratory lights, both ideas had exciting potential. In practice, the chances of either becoming a commercial reality receded as time progressed. 5.

It can be inferred from the passage that one of the reasons why earlier scientists couldn’t explain the status of nitrogen fixation among non-leguminious crops was that A. scientists who were conducting experiments to study nitrogen fixation among non-leguminous crops confused cause with effect. B. The scientists’ knowledge that the production of artificial nitrogen utilizes alarming quantities of natural resources, was incomplete. C. In the first half of twentieth century researchers who sought to study why non-leguminous crops could not do nitrogen fixation, had to work with primitive equipment. D. Unlike leguminous crops such as cleavers or soya beans, non-leguminous crops were not common in earlier days. E. The staple diet of people in earlier days did not contain non-leguminous products.

6.

The emphasis on research on nitrogen fixation since 1960 can be attributed to which of the following? A. Nitrogen fixing plants are abundantly available. B. The energy harvested by symbiosis is usually large. C. Production of artificial nitrogen utilises large quantities of natural resources. D. Productivity of crops was on the decline. E. Previously it was felt that non-leguminous plants could maintain a symbiotic relationship with bacteria.

7.

Which of the following is/are possibilities explored by scientists in order to introduce nitrogen fixation in non-leguminous crops? I. Transplantation of genes from nitrogen fixing bacteria into the cells of non-leguminous plants. II. Encouraging bacteria naturally available around plant roots become efficient in fixation of nitrogen. III. Reducing nodule formation in the roots of non-leguminous plants. A. I and III only B. II only C. I only D. III only E. I and II only T

81

Career Avenues

TEST 5 DIRECTIONS: Each passage in this group is followed by questions based on its content. After reading a passage, choose the best answer to each question. Answer all questions following a passage on the basis of what is stated or implied in that passage. PASSAGE-1 I head for a tried and tested book that I know will serve to yank me out of whatever state I find myself in. Which is how I came to The Outsider by Albert Campus (in the most recent translation by Joseph Laredo) for the umpteenth time. If you think you are wrung out and pissed off with life in general, the state of mind of young Meusault is guaranteed to set you right in a jiffy. From the opening scene where the bored and nihilistic youth boards a bus to attend his mother’s funeral (where he does not cry, thereby damning him in the eyes of everyone present) to the final paragraphs in the book, where he lies exhausted in his cell, trying to summon up images of his own execution, The Outsider continues to be one of the greatest psychological inquiries into the kind of a man unable to comprehend the demands life makes on him. Looked at another way, Meusault typifies the uncompromising individual determined to live life on his own terms – a man who in so doing upsets everyone around him as cold, heartless and unfit for decent human society. About a decade after The Outsider was published, Camus was asked about Meursault who continued to puzzle and vex readers and critics. The writer had this to say in response: “A long time ago, I summed up The Outsider in a sentence which I realize is extremely paradoxical: “In our society any man who doesn’t cry at his mother’s funeral is liable to be condemned to death’. I simply meant that the hero of the book is condemned because he does not play the game. In this sense, he is an outsider to the society in which he lives, wandering on the fringe, on the outskirts of life, solitary and sensual. And for that reason, some readers have been tempted to regard him as a reject”. “But to get a more accurate picture of his character, or rather one which conforms more closely to his author’s intentions, you must ask yourself in what way Meursault doesn’t play the game. The answer is simple: he refuses to lie. Lying is not only saying what isn’t true. It is also, in fact especially, saying more than one feels. We all do it, every day, to make life simpler. But contrary to appearances, Meursault doesn’t want to make life simpler. He says what he is, he refuses to hide his feelings and society immediately feels threatened. For example, he is asked to say that he regrets his crime. In time-honoured fashion, he replies that he feels more annoyance about it than true regret. And it is this nuance that condemns him”. In his passion to be true to himself, Meursault offends and wounds people both figuratively and literally, of course. But the author refuses to apologise for the character he has created. In his view, “Meursault is not a reject, but a poor and naked man, in love with a sun which leaves no shadows. Far from lacking all sensibility, he is driven by a tenacious and therefore profound passion, the passion for an absolute and for truth. This truth is as yet a negative one, a truth born of living and feeling, but without which no triumph over the self or over the world will ever be possible”. For nearly 60 years now The Outsider has been the existentialist novel against which all others have been measured. It’s a tribute enough to its greatness that it has lasted so long, and to thousands of new readers everywhere, it is as fresh and as compelling as when it was first written. For me, time and again, it has proved to be a great pick-me-up. After a few hours in Meursault’s company one’s own dispiritedness pales into utter insignificance. 1.

Camus’ comment on his novel, “In our society any man who doesn’t cry at his mother’s funeral is liable to be condemned to death”, is considered a paradox by the author himself. The paradox is that A. the protagonist of the novel, Meursault is an impassive being who neither feels happy nor remorseful. B. the protagonist refuses to hide his feelings and expresses his true feelings, negative or positive. C. the protagonist of the novel ‘The Outsider’ is a reject, an antisocial being. D. the protagonist of the novel is tenacious and iconoclastic and doesn’t heed to the true feelings of other. E. the author didn’t create the character of his novel as a personification of his own ideals.

82

Career Avenues

2.

Which of the following summarizes the analyst’s view of ‘The Outsider’? A. The translated version of ‘The Outsider’ doesn’t have the character and flavour of the original. B. It is an endeavour to probe into the state of mind of a man who is overwhelmed by the pleasures life offers him. C. The novel is an inquiry into the hypocritical beliefs of ordinary people, depicted through its protagonist ‘Meursault’. D. The novel is a standard against which other works of similar nature are measured. E. The novel is an attempt to understand the social psyche of a community that is decadent and moribund.

3.

Which of the following words used in the passage best explains the significance of ‘The Outsider’? A. Compelling and fresh. B. Nihilistic and uncompromising. C. Demanding and paradoxical. D. Simple and dogmatic. E. Dispirited and triumphant.

4.

Which of the following is not true of the critic’s interpretation of Camus? A. Camus doesn’t express his intentions through the protagonist of his novel. B. Meursault could easily be comprehended by readers as a social reject. C. Camus’ character Meursault can easily degenerate into the hypocrisy of present day prudishness. D. An ordinary reader would find it difficult to interpret the character Meursault. E. Camus’ novels are read once and forgotten as it fails to create the magic usually seen in existentialistic writings. PASSAGE-2

Vitamin C, known to be a DNA protecting “antioxidant,” is a switch hitter, also capable of inducing the production of DNA damaging compounds. Mutations caused by these compounds have been found in a variety of tumors. Such mutations can be repaired, however, and Ian Blair of the Centre for Cancer Pharmacology, at the University of Pennsylvania, cautioned that the study shouldn’t be interpreted as a claim that vitamin C causes cancer. Nor does it question the wisdom of eating a balanced diet rich in fruits, vegetables, and whole grains, he said. The findings, which come from test-tube experiments (in vitro), may help explain why vitamin C has thus far shown little effectiveness at preventing cancer in clinical trials. According to the Science authors. “It’s possible that vitamin C isn’t working in cancer prevention studies because it’s causing as much damage as it’s preventing, but that’s really speculation at this point. What we can say is that vitamin C clearly doesn’t work when you expect it to, and now we’re in a position to see if that’s what’s happening in vivo, (or, in living cells)” Blair said. Some scientists have long recommended dietary supplements of vitamin C, particularly for treating and preventing cancer. But the supplement’s effectiveness has been hotly debated; with critics saying they either have no effect or that they may be harmful. “The logic being used (for vitamin C supplements) is that fruits, vegetables, etc. contain vitamin C; these foods prevent cancer; thus vitamin C prevents cancer,” Blair said. “But our message is that it’s the total diet that’s important, not just one antioxidant in isolation.” Vitamin C is known to do beneficial work in the body, including acting as an antioxidant that “disarms” free radicals. These highly reactive ions are produced by the breakdown of oxygen, which occurs constantly in cells. In addition to damaging DNA directly, free radicals can also act indirectly. They act by converting linoleic acid, the major polyunsaturated fatty acid in sunflower, and safflower cooking oils, as well as the major polyunsaturated fatty acid in human plasma, into another compound called a lipid hydroperoxide. When certain metal ions are present to act as catalysts, the lipid hydroperoxides degrade further, into DNA damaging agents called “genetoxins”. These compounds react with DNA, switching one base for another in mutations that have been found in human tumours. Scientists, including Blair and his colleagues, have suspected that vitamin C might also be capable of making lipid hydroperoxides degrade into genotoxins, in place of the transition metal ions. To investigate, the Science authors added vitamin C to solutions of lipid hydroperoxides in the lab. They used concentrations comparable to those found in the human body, assuming a person would take 200 milligrams a day.

83

Career Avenues

The vitamin was more than twice as efficient as transition metal ions at inducing the formation of genotoxins, including a particularly potent variety. The researchers’ next step is to see whether vitamin C produces significant amounts of genotoxins in intact cells, and whether they generate cancer-causing mutations. The study explains why vitamin C has thus far show little effectiveness in preventing cancer in clinical trials. 5.

Which of the following simplifies the logic of taking vitamin supplements? A. If A and B are useful for C and D; C and D are also useful for A and B. B. If A contains B and C contains A, then B contains A. C. If A contains B and A is useful for C, then B is useful for C. D. If A is not useful for B, then B cannot prevent A. E. If A is used for C, and B is used for C, then B is used for A.

6.

Which of the following can be inferred from te passage about the carcinogenic properties of vitamin C? A. Evidence shows that vitamin C can damage DNA in normal cells and bring in cancer causing changes. B. The clinical studies conducted confused cause with effect; vitamin C is only a catalyst in tumor formation. C. The studies so far are misleading because their findings disprove commonly accepted hypotheses. D. Vitamin C taken as supplements are more damaging than taken in the natural form. E. There is some evidence to prove it, but our understanding of the harmful effects of vitamin C is incomplete.

7.

Which of the following can be inferred from the passage as a measure to prevent cancer? A. Regular intake of supplements containing vitamin C. B. A planned diet that is completely devoid of polyunsaturated and lipid hydro peroxides. C. Regular intake of vitamin C-rich fruits such as citrus fruits and lemons. D. A balanced diet that is rich in fruits, vegetables, and pulses. E. 200 milligrams of vitamin C supplement per day.

8.

Which of the following is a finding of the study described in the passage? A. Genotoxins are catalysts that prevent the repair of damaged cells in the human body. B. Vitamin C is not very effective in preventing cancer. C. Vitamin C doesn’t have antioxidant properties when taken as supplement. D. In normal cells mutation caused by DNA damaging compounds is rare. E. Vitamin C does not act as catalyst in degrading hydro peroxides into genotoxins. PASSAGE-3

The tests that the U.S. government relies on to keep foods free from genetically modified contaminants can miss low levels, claims a controversial research paper by the lowa company, Genetic ID. The tests in question use antibodies to detect the presence of GM foods. But the companies making such tests claim that the report is an attempt by Genetic ID to promote the technique it uses. Genetic ID prepared samples of soybean containing between 0.1 and 10 per cent GM beans, and sent them to official testing labs. Of the samples with a GM content of less than 1 per cent, more than 30 percent were incorrectly identified as GM-free. The company says it also has unpublished evidence that there are similar problems with the detection of star link maize, a GM strain not approved for human consumption because of fears that it may trigger allergies. At least one shipment of maize destined for Japan – which prohibits even traces of unapproved GM foods – was cleared by the U.S. only to fail Japanese tests.

84

Career Avenues

Genetic ID’s researchers say that antibody tests should be supplemented by the technology the company uses, the polymerize chain reaction (PCR) method, which detects modified DNA directly. But critics disagree. Genetic ID’s report admits that the antibody tests are reliable if carried out properly, Layton says. And he points out that the labs that took part were told that the tests should be performed as they are “routinely used,” Labs do not generally test for levels less than 1 percent for soybeans. So if the labs usually test for a 5 per cent level – an acceptable level for soybean exports to Japan – it’s no surprise they failed to detect a 1 percent level. All the labs reliably detected samples with a 10 percent GM content. Don Kendall of the U.S. Department of Agriculture says the USDA’s quality assessment of its own labs showed that the tests are being used correctly. In response to Genetic ID’s report, however, it is investigating further. But Kendall doesn’t expect policy to be changed to include the use of PCR. 9.

A primary purpose of the passage is to A. refute a previously held practice. B. discuss a controversial issue. C. propose an alternative hypothesis. D. summarize existing research. E. question the validity of a hypothesis.

10.

If the assertions in the passage are true, which of the following must also be true? A. Polymerize chain reaction technique cannot be used in conjunction with antibody tests to detect the presence of GM foods. B. Antibody tests are unreliable in testing for the presence of GM contaminants in foods. C. There are no reliable methods to detect the presence of genetically modified contaminants in foods. D. The technique used by Genetic ID to detect food for GM contaminants is acceptable for Japan. E. Even low levels of genetically modified contaminants in the foods can be harmful.

11.

Which of the following can be inferred from the passage about Japan’s restrictions on genetically modified foods? A. Japan doesn’t use its own tests to detect the presence of GM foods in the foods imported to the country. B. Japan does not allow foods with a GM content below 5 percent level to be imported to the country. C. Japan permits foods with a GM content of above 5 percent to be imported to the country. D. Japan conducts its own tests to detect the presence of GM content in foods imported from other countries. E. Japan is willing to buy foods from the United States that are tested using the polymerize chain reaction method. T

85

Career Avenues

TEST 6 DIRECTIONS: After each passage you will find a series of questions. Select the best answer choice for each of the questions. Answer the questions based on what is stated or implied in the passage. PASSAGE-1 A fledgling reviewer may be cowered into reverence by the sheer size and ambitious scale of this book, ‘a sequel in time’ as the author puts it, to the equally monumental Those Days published in 1997. The two novels put together sprawl well over 1,300 pages. But if there is a case to be made that size actually matters, First Light provokes a mixed response. Most contemporary big novels can be successfully pruned down with profit to both reader and writer. And First Light is no exception. Terseness, a ruthless eye on economy are not among David’s virtues. Beauty in brevity is not his credo. But it must be remembered that the novel first appeared in a serialized form in a journal, much like the novels of Charles Dickens and other writers in 19th century England. Serialization has its inbuilt constraints, mainly the pressures of coping with looming deadlines, which may affect the quality of the work. There is the compulsion to drag an episode on till saturation point is reached with the audience and a panicky inventiveness may creep in to erode the credibility of the narrative. The effect is that of a soap opera on television in which the end of one episode is left tantalizingly poised before the hooked viewer. David is a garrulous and entertaining storyteller, and this genre, if it can be called that, suits him well. His doses are, by and large, palatable. The novel covers the period between roughly 1880 to 1910, picking up the threads from the previous three tumultuous decades covered in Those Days. David in characteristic fashion, intermingles real personalities with fictional players. David has a robust and energetic style and he handles the cat’s – cradle tangle of interlocking narratives with nimble fingers. Most of the stories centre around the high society of the times and though famine, epidemics and other calamities are often referred to, there is no real discomfiting exposure to them in the entire novel. Given its size and vaunted scope, this is a gaping absence. The stories appear to take place in rarefied world of famous people‘famous surgeon’, ‘famous writer’, ‘famous playwright’, ‘famous actress’, ‘famous scientist’. A few infamous characters, fictional or real, could have infused some badly needed grit to all that girth. It is hard to be overly critical of the translation of a book of this physical magnitude. More so when the translator has also done its predecessor, Those Days. Translators from the regional languages are generally poorly rewarded, inadequately recognised and too often condescendingly declared guilty of ‘not capturing the essence of the original’. Such hasty verdicts only throw a convenient veil on the poor quality of many of the originals. Such passages are not the translator’s fault who, after all, is only trying to capture the ‘essence of the original’, which was published, amazingly, as recently as in 1996 in book form. The serialized novel is a dated concept, and it shows in the translation. The punctuation is sometimes eccentric, the comma, on occasion, conspicuous by its absence. 1.

The passage suggests that the author admires writers A. whose fictions are less militant and revolutionary. B. whose writings have been characterized by stylistic innovation. C. whose writings are succinctly and economically developed. D. who express great optimism and hope than overt dogmatism. E. who treat conventional themes in a humorous and light fashion.

2.

The author’s attitude toward David’s novels can best be described as A. serious reservation. B. grudging respect. C. enthusiastic endorsement. D. qualified admiration. E. unresolved ambivalence.

86

3.

Career Avenues

The passage suggests that one reason why translations of novels are not recognized is that A. most books that are translated have in their original version, voluminous vignettes and narrations. B. the translations are usually done by vernacular authors who are not proficient in the language used by the translator. C. translators are viewed as unable to capture the beauty of the original in all its glory. D. poor quality of the translated version often keeps publishers away. E. fictional novels are hard to translate since the translator may not be comfortable with unrealistic portrayals.

4.

The aspects of David’s works most extensively discussed in the passage is A. their mystery and irony. B. their unrelenting moralism. C. their humorous overtures. D. daunting melancholy. E. volume and style. PASSAGE-2 Was Plato the world’s first sci-fi writer? Did he produce the story of Atlantis, the lost continent, entirely from imagination, or is there any truth in it? Even as the search for Atlantis goes on, one school of scientists thinks that there is nothing to it in the realm of fact. Others, of course, believe equally firmly that the continent did exist and that its remains will be found under the sea, west of Gibraltar, some day. Its discovery, they say, will throw light on an unknown chapter of human history and civilization. Some recent discoveries from under the seas have perhaps boosted their morale. The palace of Cleopatra has been found under water 20 miles off the coast of Egypt, though no artifacts have yet been salvaged. Hints of ancient structures under the oceans have also been received from elsewhere. Scientists are also investigating ships lying on the ocean floor, from some of which personal effects of their passengers have been retrieved. Myths and mysteries have always had a hold on our collective minds even in the age of science. Consider the numerous stories of UFO ‘sightings’ even though science has rubbished the idea of extra-terrestrials visiting the earth. Critics of the Atlantis story point out that there was no reference to it either before, or after, Plato. Also, that the account available is all too dramatic to be believable. A lush continent inhabited by beautiful people in an advanced state of civilization being suddenly gobbled up by the sea within a day is fiction material par excellence. One theory is that the entire continent cracked up due to volcanic eruptions. Another even more fantastic one suggests that an enormous magnet fixed on top of a palace by the sea had created a massive tidal wave, which caused the destruction intriguingly. Plato was not against propounding and perpetuating myths. In Republic he says, “I wonder if we could contrive... some magnificent myth that would in itself carry conviction to our whole community”. Was Atlantis the one myth he decided to contrive? It is difficult, perhaps impossible, to answer that. Meanwhile, it continues to baffle scientists simply by not being found, encouraging many to maintain that it will eventually be found. Such is the power of myths. And that power grows even more once the sea comes into it. Beginning with the Greeks, mankind has always looked with wonder and awe at the sea, infesting it with countless myths and mysteries. 5.

The author is primarily concerned with A. criticizing the unrealistic writings of Plato and their influence on human prejudices. B. giving an account of the origin and development of myth and legend in the earlier civilizations. C. discussing the overwhelming influence of myths and mysteries on human civilization. D. providing evidence for the hypothesis that volcanic eruptions are massive enough to engulf entire continents. E. questioning the scientific compatibility of UFO sightings.

6.

Based on the information in the passage, it can be inferred that the author would most probably disagree with which of the following statements? A. The Atlantis is a figment of Plato’s imagination and therefore, the theory that such a continent might have existed, is entirely unfounded. B. Over centuries, scientists are accused of propounding and perpetuating myths. C. Some modern scientists are hopeful that Atlantis would be found someday, even though, scientists have been unsuccessful so far. D. The belief in myths and mysteries is influenced by the lack of scientific temper among the common man. E. The stories of UFO sightings are baseless rumors, and it is impossible to ever attach any scientific evidence to it.

87

Career Avenues

7.

The author views recent efforts to unearth evidences of ancient civilizations from the sea floor A. pessimistically B. condescendingly C. objectively D. indifferently E. discouragingly

8.

The author cites the instance of the discovery of the palace of Cleopatra primarily in order to A. show that, in similar ways it may be possible to locate the Atlantis under the sea. B. criticize those scientists who propound and perpetuate unfounded stories to create confusion among the common man. C. argue that attempts to search for evidences based merely on hearsay are futile. D. reinforce the fact that Plato’s Atlantis is purely a mythical creation. T

88

Career Avenues

TEST 7 DIRECTIONS: After each passage you will find a series of questions. Select the best answer choice for each of the questions. Answer the questions based on what is stated or implied in the passage. PASSAGE - 1 Among issues of public policy, the one that has attracted the widest debate has been the respective role of the state and the market in economic development. This debate has once again assumed importance in the context of economic reforms and structural adjustment underway in many countries. Obviously this question does not have an ‘either-or’ answer. What needs to be determined is the optimal mix of state and market. The effectiveness of the state or market in economic intervention cannot be argued in a vacuum. Much depends on the nature of the state and the structure of markets. History is replete with instance of state terrorism. Even in the economic arena, the state has acted in many countries – even in democracies in the interests of the people in power rather than of people in general. Equally, the efficiency of markets depends on the market structure. In theory, much of the benefits of markets flow from competitive equilibrium or full or near full market clearing. In practice, this may not happen. The earliest proponent of free trade and the primacy of the market was Adam Smith, who made the celebrated statement that in pursuing his own advantage, each individual was “led by an invisible hand to promote an end which was not part of his intention”. Smith, nevertheless, said that the state should undertake the three main tasks of maintaining law and order, enforcing justice and undertaking public works which may not be in private interest. The idea of state minimalism, summarized in the statement “that government is best which governs the least”, acquired confirmation in welfare economics, where it was shown that if competitive market forces were allowed free play, an economy would obtain a state where it was impossible to improve a person’s well-being without affecting another person. The major criticism of state minimalism came from Keynes in the wake of the Great Depression. He stressed the role of the state in maintaining a level of effective demand, which was necessary to ensure full employment. He argued that market forces by themselves would not result in full employment. Besides, the state has a role to play in monitoring the actions of individuals and institutions. After all, in the absence of some superior authority, a section of the participants may contravene the rules. The importance of state intervention is emphasized by economists who regard the state as a ‘trustee of the poor’. In any economic system, there are three roles for the state – as a producer of goods and services, as a systemic regulator, and as a supplier of ‘public goods’ or ‘social goods’ like primary education and health. The first role finds expression in the system of planning, with public enterprises engaged in productive activities, if not in all, then at least in the critical areas called the ‘commanding heights’. The second role, of regulator, gives the state the superauthority to set the rules of the game. In fact, the quality of the economic performance of markets depends critically on the quality of public intervention through regulation. The third role – of facilitator is performed through the provision of physical and social infrastructure. Countries in general are moving away from the role of state as producer of goods and services. A major reason for the emergence of skepticism regarding the benefits of state intervention in this area has been the growing perception that government failures on account of political factors and bureaucratization may, in many cases, exceed market failures. The decreasing role of the state as producer and the increasing role of the market in this area enhances the role of the state as a regulator. Gerhard Schroeder summed up the position by remarking, “the state should not row but steer”. The analogy may not be perfect. But the substance is clear. The state can play a better role by setting the direction rather than by driving. Even in relation to the facilitator role, the state need not participate directly in investment in physical infrastructure. Private investment under a suitable regulatory authority may yet fill the need, even though the role of direct investment by the state in these areas will be dominant for quite some time in emerging economics. But in the area of social infrastructure, like health and education, the role of the state is clearly seen, even though intervention may take many forms.

89

Career Avenues

Thus, for example, a school may be fully funded by the government and run by the government; a school may be partly funded by the government but run by private management; both funding and provision may be done in the private sector but the government may support targeted children who cannot afford to pay for the private facility. The choices are many. But we must choose with care so that the benefits to society are enlarged. The question we have to address is one of how much state intervention, what kind and by what means. As a general rule, markets must be allowed to function where price signals clearly work in achieving efficiency. State intervention becomes necessary in areas where markets do not exist or where they cannot perform efficiently. Obviously, there are areas in which the state alone can function. The state has an absolute advantage in these areas. Here the objective should be to improve efficiency through better governance. 1.

Which of the following is/are the main concerns of the passage? I. Countries are moving away from the state as producer of goods and services. II. The state should intervene where markets cannot perform efficiently. III. The state must assume the role of market regulator and facilitator. A. I and II only B. II and III only C. I, II and III D. I and III only E. II only

2.

If the assertions in the passage are true, which of the following is a direct consequence of the influence of political forces on the market? A. Government’s failure to participate directly in investment in physical infrastructure for market enhancement. B. Any failure in the government owing to political factors would adversely affect the market. C. Government’s failure to maintain a level of effective demand, which in turn would ensure employment. D. The government’s going against private interest by enforcing justice through undemocratic means. E. Delay in planning, with public enterprises engaging in other unproductive activities.

3.

Keyne’s criticism of state minimalism can be attributed to which of the following assumptions? A. State intervention alone would not result in full employment. B. The state should not impose authority on the market to protect individuals. C. Intervention of the state would favour organized forces and bureaucrats rather than individuals. D. Market structure need not be a deciding factor in ensuring market efficiency. E. The necessity to ensure employment through state intervention in maintaining effective demand.

4.

The author recommends which of the following in the wake of a market’s inability to function productively? A. Restructure the market using economic reforms. B. The government should behave as the one that ‘governs the least’. C. Intervention of the Government. D. Privatizing of the concerned sector. E. Setting new rules to regulate demand. PASSAGE-2

Albert Einstein once said: “I never think of the future, it comes soon enough”. One aspect of the future that can never come soon enough is that of greater autonomy. In film-making, though, autonomy is frequently stifled by the unequal possession of capital and the voices of the many are simply marginalized. Alternatively, autonomy can be undermined by the predilections of the owners of capital, as in the studio system where formulas are devised to meet market-researched standards at the cost of creativity or experimentation. Most perniciously though, any act of investment constricts autonomy by creating a structural dilemma, the necessity of recouping investments – without which future production is endangered. The expensive nature of 35mm productions

90

Career Avenues

thus exerts an all too obvious pressure on film-makers, which drives commercial cinema towards the safety of numbers and tried-and-tested content. Unsurprisingly then, lowest common cultural denominators and universal themes often determine what is popular cinema. Digital video as a technology promises empowerment through an improved set of opportunities, which far outweigh its constraints. Autonomy in digital film-making (the making of digital format motion pictures) is derived from its relatively low capital-intensity requirements. This, in turn, reduces the break-even constraint. The broadcast quality bare essentials can be rented for upwards of $200 per day (though actual cost is entirely contingent on the intended nature/sophistication of the production). As a general rule though, the scale effects that dominate the production requirements of traditional formats are toned down enormously. An immediate consequence is the increased ability to take risks with form and content. The essential thrust of digital film making is liberal democratic as it increases the spectrum of output and also protects diversity by allowing for niche markets to be satiated. In occupational terms, the relative affordability of digital video also allows for the studio apprentice system to be replaced by greater hands on experience for first time film makers who believe that creativity can transcend experience. Another significant feature of digital vide is its relative immunity against the heavy handed tactics of parochial politics. It is hardly the case that India lacks brave political and social cinema. However, producing and protecting such produce become easier when it is in digital form. Digital video’s subversive capabilities are obviously double edged: The role of audio tapes, as a decentralized, lowcost medium, in furthering communal tension in the early 1990s has been well documented. It is a truism that decentralized mediums, which promote the freedom of speech, need not always promote what you want to hear. Nonetheless, high quality audio-visual dissent can strengthen human rights and provide unique perspectives: Palm sized digital cameras for example have been used for sensitive undercover investigations and war reporting. Presently, digital film-making is dogged by significant constraints. One is the capital intensive nature of the computer based editing and post production equipment where rentals can range from $20 to $ 200 per hour. A more awkward question is whether digital video will gain widespread acceptability amongst audiences despite its less than cinematic visuals. One complicated alternative is for a digital film that looks commercially viable to be transferred to 35mm for theatrical release. A more promising alternative lies in newer distribution channels that seek to broadcast over the Internet and broadband. Recent innovations in video compression technology, together with increased bandwidth availability on fiber-optic networks, promise to substantially increase the download speeds and visual quality of such streaming video. Research in the US also suggests it may eventually be possible for digital films to be downloaded from satellites to digital cinemas, thus allowing for an important part of the cinematic experience to be retained. Ultimately we can hope that the seemingly inexorable progress of technology will carry digital films towards greater sophistication and viability. Digital film-making should thus be seen as an incipient challenge to the unifying nature of global production for global markets. As such, it is an alternative to not the replacement of traditional techniques and content. Its produce will thus complement 16mm indie (independent) and guerrilla cinema that also challenge the formula system. A significant example of this techno-aesthetic approach is the Dogme 95 Manifesto a statement of intention adhered to by filmmaking away from the special effects and large budgets of Hollywood and back to storytelling and innovative camera work. Critically acclaimed works in this genre include Festen (which addresses sexual abuse within families), The Idiots (which addresses societal notions of deviancy), and two Oscar nominated documentaries, The Buena Vista Social Club and The Farm.

91

Career Avenues

5.

According to the author, which of the following is a direct consequence of the studio system on film making? A. Emphasis on experimentation. B. Lack of originality. C. Lack of research on market demands. D. Prevalence of 35 mm productions. E. Tried and tested formulas’ taking a backseat.

6.

Which of the following is an advantage of the use of digital video technology? A. Low cost in film production, which paves way for greater autonomy. B. The digital video movement will complement formulaic film making. C. Computer based editing will become capital intensive. D. Ability to take risks with form and matter will decrease dramatically. E. Provision for studio apprenticeship.

7.

The author is optimistic that innovations in video compression technology and bandwidth would contribute substantially to I. increase speed of downloading. II. curbing niche markets. III. enhance video quality. A. II only B. III only C. I and III only D. I and II only E. II and III only

8.

The author expresses optimism on which of the following possibilities in film production? A. Digital films will become increasingly popular as technology improves. B. Digital technology has the potential to replace traditional techniques of filmmaking. C. Traditional story-telling method will give way to special effects. D. Digital filmmaking will challenge and marginalize guerrilla cinema. E. Digital filmmaking can protect unity by satisfying niche markets. PASSAGE-3

The uniform equality of human beings as subjects of a state is perfectly consistent with the utmost inequality of the mass in the degree of its possessions, whether these take the form of physical or mental superiority over others, or of fortuitous external property and of particular rights (of which there may be many) with respect to others. Thus the welfare of the one depends very much on the will of the other (the poor depending on the rich), the one must obey the other (as the child its parents or the wife her husband), the one serves (the labourer) while the other pas, etc. Nevertheless, they are all equal as subjects before the law. For, one can coerce no lone else other than through the public law and its executor, the head of state, while everyone else can resist the others in the same way and to the same degree. No one, however, can lose this authority to coerce others and to have rights towards them except through committing a crime. And no one can voluntarily renounce his rights by a contract or legal transaction to the effect that he has no rights but only duties, for such a contract would deprive him of the right to make a contract, and would thus invalidate the one he had already made. From this idea of the equality of men as subjects in commonwealth, there emerges this further formula: every member of the commonwealth must be entitled to reach any degree of rank which a subject can earn through his talent, his industry and his good fortune. And his fellow subjects may not stand in his way by hereditary prerogatives or privileges of rank and thereby hold him and his descendants back indefinitely. 9.

The opening lines of the passage best exemplifies, which of the following? A. An analogy between seemingly parallel phenomena. B. A consequence of the contradiction existing within the rule of law. C. A paradox central to the concept of state and law. D. A dilemma arising out of the demands of the citizens and the duties of the state. E. An inherent weakness in the complementarities of legal vis-à-vis moral rights.

92

Career Avenues

10.

Which of the following, as expressed in the passage, would refute the argument that the welfare of one depends on the will of the other? I. The absence of a right for one to force another outside the law. II. The presence of equal right for one to resist oppression by another. III. The provision in the law for one to give up one’s right through a formal agreement. A. II only B. I and III only C. III only D. I and II only E. II and III only

11.

If a citizen were to, by agreement give up his rights, which of the following would be true? A. His son’s contract would be nullified because he has no right to make a contract. B. He will be deprived of his duties as a citizen under the definition of the state. C. He deprives himself of the privileges he would’ve enjoyed otherwise- hereditary or position induced. D. He renounces his duties of providing welfare to his dependants. E. He is liable to be recapitulated by coercive forces outside the public law. T

93

Career Avenues

TEST 8 DIRECTIONS: Each passage in this group is followed by questions based on its content. After reading a passage, choose the best answer to each question. Answer all questions following a passage on the basis of what is stated or implied in that passage. PASSAGE-1 The Hubble Space Telescope (HST) has completed its measurement of the Hubble constant, a measure of how fast the Universe is expanding and a cornerstone of our understanding of the age and size of the Universe. The final value of the constant seems to be just about spot-on. The finding marks the attainment of one of the HST’s primary goals. U.S. astronomer Edwin Hubble noticed in 1929 that the farther a galaxy is from the Earth, the faster it appears to be moving away. This idea of an expanding universe underpins the Big Bang theory, which says that the Universe began with an intense burst of energy and has been expanding ever since. Measuring the rate of the Universe’s expansion requires calculating the acceleration of objects both near and far from us, and measuring the difference. Before the HST, the best estimates of the Hubble constant put the figure at either 50 or 100 kilometers per second per mega parsec. This number describes Hubble’s discovery - an object’s speed increases with increasing distance from the observer. The lower of these values meant that some objects in the Universe would be older than the Universe itself. But most astronomers believed that the higher figure was too big. Measuring the Hubble constant was one of the HST’s primary aims because it can detect dim, distant objects that are not visible from the Earth, such as many so called ‘Cepheid variable’ stars. These stars ‘pulse’ between being dim and bright and their pulse rate is linked to their maximum brightness. Because this unique property of Cepheids makes their distance easier to confirm, they make ideal cosmic yardsticks with which to measure the Hubble constant. After surveying Cepheids in 18 galaxies at various distances using the HST, and combining their data with other measurements from other estimates of the constant, her team calculated the Hubble constant to be 72 kilometers per second per mega parsec. They agrees well with other recent measures of the size and shape of the Universe, such as those based on cosmic microwave background radiation. This approach has come up with an estimate of the expansion rate of the Universe by measuring the faint microwave afterglow of the Big Bang. 1.

The author considers Cepheid’s as the standards with which Hubble constant can be measured, for which of the following reasons? A. They are observable from the earth. B. They are brighter than any other star in the galaxy. C. They remain constant unlike other stars that constantly pulsate. D. Their distance is easier to confirm due to the correlation between pulse rate and brightness. E. They are found in about two dozen galaxies that exist closer to the earth.

2.

The information in the passage shows which of the following as a primary objective of the HST? A. To describe how long a measuring device might take to measure the expansion of the universe. B. To verify the relationship between distance of a galaxy from the earth and the rate at which it moves away. C. To investigate the birth of the universe and the mass of the sun when it was formed. D. To calculate the acceleration of stellar objects closer to the earth and farther from the sun. E. To estimate the age of the earth using the nature of the planets that revolve around the sun.

3.

Because the technique of measuring the rate at which the universe is expanding is based on Cepheids, if there turns out to be a problem with the approach, which of the following would be true? A. It could seriously weaken the HST’s estimate of the Hubble constant. B. It would refute the possibility of some objects in the universe to be older than the universe itself. C. It would be nearly impossible to study pulsating stars that are dim. D. It becomes difficult to estimate the speed with which stars farther from the earth are moving. E. It could boost the Big Bang hypothesis which otherwise would’ve been undermined by Hubble constant.

94

Career Avenues

4.

Which of the following is/are required to measure with precision, the expansion rate of the universe? I. Calculating the mass of the objects in the galaxy closer to the earth. II. Calculating the acceleration of objects near and far from the earth. III. Measuring the difference between accelerations of objects near and far from the earth. A. II only B. I only C. I and II only D. I and III only E. II and III only PASSAGE-2

The whole spectrum of knowledge from the faintest flicker of sensation in the lowliest of creatures to the highest peaks of wisdom and intuition in the enlightened sages – is revelation. All ideas and understandings, all thoughts and findings, are revelations. Which means all these and their antitheses are one and the same revelation – both monotheism and polytheism, dvaita and advaita, good and evil are revelations. Revelation is straggling and self contrasting on this end, though integral as its wellhead. The wellhead, the Cosmic Psyche (CP) or God or whatever you call it, is supremely singular and perfect ekamevadvitiyam, but its very singularity and perfection blesses it with an incurable disability – it cannot reveal itself except through a medium. Without a medium to reveal itself through, it is pure being which, as Hegel recognized, is equal to sheer nothing. Being is being only when it is known to be. To be known to be the CP, the CP has to take up a medium. But when it does, there comes another rub. A medium doesn’t let the stuff of revelation through intact as it is in its original form. It colours and qualifies revelation while passing through it. When the CP enters a medium, it enters a mould. Moulds are different and so the products also are different. Individual and collective psyches, the media for revelation, are the branches of the revelation tree and infinite opinions are its leakage. The singular virtue of science as against religion and philosophy is that it is not and can never be the beating of revelation into a hot sheet of self-love and paranoia of one or the other race or nations. Now you see why religion has to stand on belief – and science, on reason. A religion tries to sell its original race as the master race and its racism as the true way, while science tries to see reality behind appearance. Take out all racial national properties and proper nouns from a religion and see what remains. All you have in hand are one or two vague generalisms – love, brotherhood and the like – upon which that religion can’t establish its identity, let alone set out its validity or force. A thing is what makes its identity. Racism and nationalism are inborn in us and they are here to stay. They can’t be wished or forced away. But in order to ensure a civilized, peaceful and practical life on earth there has to be a proviso which is : Every race or nation, while living its racism or nationalism within its own racial or national constituency should grant the right of other races or nations to live their own racisms or nationalism within their own racial or national constituencies. This is a practical live and let live policy among races and this is secularism. Secularism is letting all the leaves on the revelation tree have their place in the world and bask in the sun. A transitional religion per se is a breach of secularism. It is this which contains the seeds of communal violence, fundamentalism and terrorism. A people is not just a crowd but a collective psyche. Where entire races and nations were converted, where the whole collective psyches were destroyed. That’s why conversion is genocide. An evangelical mission is an outrageous attack on the cosmic revelation tree (CRT). It is an attempt to strike the leaves of other races or nations off the CRT and flogging it to produce and present only the leaves of self-declared master race or nation. Where evangelism is on the prowl, there the native collective psyche is under threat to extermination and that is why evangelical missions are original provocations for hate-culture and violence. Our gods and religions are not pure revelations but revelation cooked to taste. A religion is essentially the identification of its original race and the demonstration of rival races. To reach out to pure revelation we have to demolish the media of revelation, our senses, mind and intellect, our very instruments for sensation and knowledge and grow one with the CP through concentration and meditation.

95

Career Avenues

Revelation is always progressive; it is never final. It is final only at the death of phenomenal existence. Only a religion that can keep its identity and idea even after being shorn and racial properties and proper nouns can stake claim as a universal religion. Since we are nowhere close to this, the least we can do is to live out our own identities within the bounds of our racial or national constituency, letting others do so within theirs. This is what Krishna asks of us in the Bhagavad Gita. This is secularism. 5.

Which of the following, in the author’s perception, is the incurable disability of the cosmic psyche? A. It manifest itself even in the lowliest of creatures. B. It is supremely singular and not all-encompassing. C. It can manifest itself only through a medium. D. It depends upon rational thinking for its revelation. E. It can show apparent signals in a vacuum.

6.

Which of the following is the author’s interpretation of the juxtaposition of science and religion? A. Science and religion are two perspectives of the same argument. B. Religion stands on faith whereas science stands on reason. C. Religion is individualistic where as science is collective. D. Religion stems from ‘being’ and science from ‘becoming’. E. Religion can relate diverse phenomena into meaningful syntheses whereas science is based on apparent contradictions.

7.

To bolster his own argument, the author cites Hegel in which of the following contexts? A. To propound that ‘being,’ if not revealed through a medium, is equal to nothing. B. To posit the singularity and perfection of the concept of the all-pervasive God. C. To argue that a medium does not alter the stuff of revelation when it passes through it. D. To explain the singularity of the product of the Cosmic Psyche’s passing through the medium. E. To claim that being or becoming is only hypothetical.

8.

The author makes which of the following statements about the nature of science? A. Science is a product of paranoid thinking. B. Intuitive analysis is integral to science. C. Science is generally secular thinking. D. Science attempts to see reality behind appearances. E. Nature attempts to unravel natural conspiracies between the real and unreal.

9.

Which of the following is not true in the context of the passage? A. A medium manipulates a revelation that passes through it. B. Collective psyches can serve as a medium for the passage of cosmic psyche. C. It is possible to get rid of racism and nationalism from the human psyche. D. Revelation is a continuous phenomenon. E. It is not possible to cleanse human psyche of racist and separatist feelings.

10.

Which of the following is the reason why the author rejects conversion of any kind? A. The author strongly defends a transitional religion. B. Conversion results in the destruction of the collective psyche. C. Evangelical missions are instrumental to secularism. D. Conversion demolishes our mind and intellect. E. It results in the creation of cynicism in the impressionable minds.

11.

The author cites which of the following as the criterion for a religion to be regarded universal? A. A religion’s racial properties. B. A religion’s national properties. C. A religion with generalisms of love and brotherhood. D. A religion that is inseparable from the racial consciousness. E. A religion that has an identify without its racial properties. T

96

Career Avenues

TEST 9 DIRECTIONS: Each passage in this group is followed by questions based on its content. After reading a passage, choose the best answer to each question. Answer all questions following a passage on the basis of what is stated or implied in that passage. PASSAGE - 1 Many perceptive marketers began suspicious of three basic assumptions they had made, in their efforts to be logical, concerning the predictable behavior of human beings, especially customers. First, they decided, you can’t assure that people know what they want. A major ketchup maker kept getting complaints about its bottle, so it made a survey. Most of the people interviewed said they would prefer another type the company was considering. When the company went to the expense of bringing out this other bottle in test markets, even people who had favoured it in interviews overwhelmingly rejected it in favour of the old bottle. In a survey of male beer drinkers the men expressed strong preference for a ‘nice dry beer’. When they were then asked how a beer could be dry they were stumped. Those who were able to offer any answer at all revealed widely different notions. Second, some marketers concluded, you can’t assume people will tell you the truth about their wants and dislikes even if they know them. What you are more likely to get, they decided, are answers that will protect the informants in their steadfast endeavor to appear to the world as really sensible, intelligent, rational beings. One management consulting firm has concluded that accepting the word of a customer as to what he wants is ‘the least reliable index the manufacturer can have on what he ought to do to win customers’. The Advertising Research Foundation took magazines to task for asking people what magazines they read frequently, and naively accepting the answers given as valid. The people, it contended, are likely to admit reading only magazines of high prestige value. One investigator suggests that if you seriously accepted people’s answers you might assume that Atlantic Monthly is America’s most-read magazine and some of the confession magazines the least read; whereas actually the confession magazines in question may have twenty times the readership of Atlantic Monthly. Finally, the marketers decided it is dangerous to assume that people can be trusted to behave in a rational way. The Colour Research Institute had what it felt was startling encounter with this proneness to irrationality when it tested package designs for a new detergent. It was testing to see if a woman is influenced more than she realizes, in her opinion of a product, by the package. It gave the housewives three different boxes filled with detergent and requested that they try them all out for a few weeks and then report which was the best for delicate clothing. The wives were given the impression that they had been given three different types of detergent. Actually only the boxes were different; the detergents inside were identical. The researchers were proved true what they had suspected. 1.

The difficulty facing marketers was that prospective customers were A. not honest B. inarticulate C. forthcoming and sensitive D. hesitant E. indifferent

2.

Supported in the article by evidence from the consumers of ketchup and ‘nice dry beer’ many marketers concluded that it could not be assumed that A. customers knew exactly what they wanted. B. customers did not change their preferences overnight. C. most customers were influenced by quality and style. D. most customers were highly susceptible to persuasive advertising. E. Most customers were irrational in their brand loyalty.

3.

Some marketers concluded from the evidence of readers of magazines, that people A. purchased more of what they thought had prestige value. B. did not always tell the truth. C. were surprisingly truthful. D. said whatever they imagined would please.

97

4.

5.

Career Avenues

E. were ambivalent in their opinions. We can infer from the instance of testing of package designs for a new detergent that A. customers can be influenced by a scientific approach to design. B. package design is not as important as it was thought to be. C. design influences some people much more than others. D. attitudes to package design are unpredictable. E. virtually all women went by instinct when it came to decision on purchases. One of the conclusions drawn in the passage is that it was dangerous to assume that A. motives are simple. B. motives do not conflict. C. people do not have unconscious motives. D. people can be trusted to behave in a rational way. E. people cannot be taken for granted when it comes to decision making. PASSAGE-2

There really is no such thing as Art. There are only artists. Once these were men who took coloured earth and roughed out the forms of a bison on the wall of a cave; today they buy their paints and design posters for the Underground; they did many things in between. There is no harm in calling all these activities Art as long as we keep in mind that such a word may mean very different things in different times and places, and as long as we realize that Art with a capital A has no existence. For Art with a capital A has come to be something of a bogey and a fetish. You may crush an artist by telling him that what he has just done may be quite good in its own way, only it is not ‘Art’. And you may confound anyone enjoying a picture by declaring that what he liked in it was not the Art but something different. Actually I do not think that there are any wrong reasons for liking a statue or a picture. Someone may like a landscape painting because it reminds him of home, or a portrait because it reminds him of a friend. There is nothing wrong with that. All of us, when we see a painting, are bound to be reminded of hundred-and-one things, which influence our likes and dislikes. As long as these memories help us to enjoy what we see, we need not worry. It is only when some irrelevant memory makes us prejudiced, when we instinctively turn away from a magnificent picture of an alpine scene because we dislike climbing, that we should search our mind for the reason of the aversion which spoils a pleasure we might otherwise have had. These are wrong reasons for disliking a work of art. 6.

It can be inferred from the passage that Art with a capital A A. means the same thing in different times and places. B. is art devoid of individual likes and dislikes. C. does not really exist. D. must be distinguished from commercial art. E. has ceased to be a fashion statement.

7.

Which of the following can be inferred about the author’s perception from the passage? A. There are no wrong reasons for liking a work of art. B. We must not bring our memories to the enjoyment of a work of art, but see it as it is. C. There are no wrong reasons for disliking a work of art. D. When responding to a work of art, we must rely on our instinctive feeling. E. The author admires art work only when it reminds him of any incidence in his life.

8.

The author is most likely to agree with which of the following? A. Artists have a knack of distorting things. B. Art appreciation is a highly subjective process. C. What inspires the artist’s vision is asymmetry in concrete things. D. determining the beauty of a picture is an objective pursuit. E. Artists prefer representing reality as it is.

98

Career Avenues

TEST 10 DIRECTIONS: Each passage in this group is followed by questions based on its content. After reading a passage, choose the best answer to each question. Answer all questions following a passage on the basis of what is stated or implied in that passage. PASSAGE - 1 In the case of any person whose judgment is really deserving of confidence, how has it become so? He has kept his mind open to criticism of his opinions and conduct. It has been his practice to listen to all that could be said against him; to profit by as much of it as was just, and expound to himself, and upon occasion to others, the fallacy of what was fallacious. He has felt that the only way in which a human being can make some approach to knowing the whole of a subject is by hearing what can be said about it by persons of every variety of opinion, and studying all models in which it can be looked at by every character of mind. No wise man ever acquired his wisdom in any mode but this; nor is it in the nature of human intellect to become wise in any other manner. 1.

The title below that best suits the above passage is: A. Criticism of ill-formed opinion B. How to form an opinion C. Attaining wisdom through listening D. The opinion of the wise E. The nature of human intellect

2.

Which of the following best expresses the author’s conception of an intelligent person? A. He should be slow in his assessment of objective reality. B. He is eclectic in his approach to acquisition of information. C. He can be certain that his conduct is sound. D. He believes that an open mind guarantees wisdom. E. He shuts his mind to the notions of the uneducated. PASSAGE-2

Knowing belongs to the realm of the rational; therefore most people tend to take too intellectualistic a view of learning; that the act of learning in based on irrational processes is overlooked. Because of a widespread, but mistaken belief that education is nothing more than the provision of ‘facts’ to be stored, the learner is thought of as an ‘empty vessel, waiting to be filled’ (Thomas Carlyle). The teaching function is conceived solely in terms of imparting information. The teacher is a doler out of ‘facts’ in quantities appropriate to the ‘capacity’ of the pupil. Learning is equated with the outcome ‘knowing’, which appertains to our rational nature. It is certainly true that an essential and important part of the educator’s task is the imparting of relevant information. But examine what remains from our school and university days, we find not a collection of discrete facts, but rather an attitude to life. Only those who will become teachers need concern themselves with hoarding ‘facts’ as the squirrel hoards nuts. Most of those who have written about education distinguish between education and instruction. To mention only one example: Herbart says that ‘Instruction forms the circle of thought: Education the character. The last is nothing without the first. Herein is contained the whole sum of my pedagogical doctrine.’ To complete Herbart’s thought we must add that the first (instruction and the circle of thought) is nothing without the last (education and character). Instruction is not the same as education. It is one of the instruments by means of which people become educated. Herbart says in another place: ‘The one task and the whole task of education may be summed up in the concept of morality.’ 3.

According to the passage, most people tend to ignore the fact that A. cognitive capabilities vary from individual to individual. B. learning as a process involves irrationality. C. learning is an intuitive and tacit process. D. learning is concerned only with receiving relevant information.

99

4.

5.

Career Avenues

E. Teaching is a one way activity. In the author’s analysis, what is carried forward from our school and university days is A. acquired knowledge. B. disinterested and uncritical attitudes. C. an attitude to life. D. the ability to reason. E. the ability to adjust to changes. Which of the following can be inferred from the passage about the relationship between instruction and education? A. There is no difference between good education and good instruction. B. What we need is education, not instruction. C. We need more instruction and less so-called education. D. Instruction is a pedagogical approach to education. E. Instruction is a broad all encompassing process whereas education is more narrow. PASSAGE-3

It was because he was dissatisfied with his ages, that the Romanticist looked back to former ages. Even when the desire to reach back in history had faded, the malaise remained. We may laugh, in retrospect, at the maladie d’un enfant du siecle, but it was a malady, which was universal. Byron regarded himself as a blighted being; but so did almost every other young man of sensibility in the early thirties. It is even more curious that women did so too. It was as if, after the orgies of the post-Revolutionary period, everyone in the world had awakened with a headache. Frank paganism and robust health seemed to have vanished together. It is startling to note the number of girls who ‘went into a decline’ and died before they had reached womanhood. Some have suggested that the extraordinary prevalence of consumption during the Restoration period was due to the very inadequate clothing of the previous generation; but in the face of modern medical opinion it is hard to believe that anybody was ever much worse for wearing little. No, the invalidism of the Romantics was largely a matter of mentality. It was nonetheless frequently mortal, and this is perhaps the most astonishing thing about it. Tight lacing may be considered either as a cause or merely as a symptom of the prevailing tendency. Women began to suffer from perpetual migraine, to look pale and faint upon sofas at the slightest provocation. To be fat was almost a crime, and even to look healthy was something approaching a solecism. An ideal fragility was the prevailing mode, and to attain it women were willing to suffer martyrdom. To eat heartily was a mark of grossness, and to such an extreme was this idea carried that many women found it necessary to make a meal at home before going out to dine. It will be remembered that Byron at one time confined himself to a diet of potatoes sprinkled with vinegar. ‘How long will his lordship persist in his present diet?’ Asked one of his friends. ‘Just as long as you continue to notice it’ returned another; and there was no doubt an element of ostentation in all such privations. 6.

Almost every man of sensibility in the early thirties regarded himself as A. an eccentric. B. a hero. C. a frustrated being. D. a poet. E. a non-conformist.

7.

The extraordinary prevalence of consumption among women was in actuality A. a lack of fresh air and proper food. B. a mental attitude. C. a consequence of tight lacing. D. a rebellion against set rules. E. triggered by the policies of the establishment.

8.

The prevailing mode among women was A. a vulnerably delicate appearance. C. whimsicality. E. a snobbish look.

B. moral fervour. D. poetic feeling.

100

Career Avenues

PASSAGE - 4 As a way of life, there can be no doubt of Methodism’s appeal; it contained so much that was capable of satisfying the deepest needs of human nature. In the exercise of religion there was no emotional restraint. Sobbing, weeping, laughter, hysteria were commonplaces of Methodist fervour – a lack of restraint which seems to us almost pathological. But there was an edge to life in the eighteenth century, which is hard for us to recapture. In every class there is the same taut neurotic quality – fantastic gambling and drinking, the riots, brutality and violence, and everywhere and always a constant sense of death. At no point did the Anglican or Dissenting churches of the day touch this inner tragedy of man, which was the emotional core of Methodism. But Methodism gave far more than emotional release; it brought a sense of purpose and a field for the exercise of both will and power. To men and women who were just climbing out of utter poverty by the dint of their own thrifty endeavour this concentration of will and purpose was particularly appealing. The oligarchical and rigid nature of local institutions meant that there was little scope for ambitious men and women with a social conscience. All doors were closed to them, including, of course, those of the established Church, but Wesley provided an organization in which they could fulfill their need for power and their sense of duty. Unfortunately, Methodism appealed to other less socially valuable sides of human nature. There was nothing intellectual about Methodism; the rational attitude, the most fashionable intellectual attitude of the day, was absolutely absent. Wesley believed in witches, in the corporeal existence of the devil, and in possession by devils. He made decisions by opening his Bible at random and obeying whatever command he might discover from the first words which met his eye. Wesley’s superstitions were those of his uneducated audiences. He produced a little book on physic, which was on sale at all meeting houses. It is an absurd, fantastic compilation of uncritical folklore. 9.

It can be inferred from the passage that Methodism owed its appeal to A. the restraint in the opportunities for emotional release. B. providing a sense of purpose and a platform for the exercise of will and power. C. an almost pathological lack of intuition and meaningfulness. D. the respite it gave from the fear of mortality. E. the apparent sense of security that it gives.

10.

As a way of life Methodism appealed especially to men and women who A. were just climbing out of their indigence by their own frugality and efforts. B. believed passionately in all the socially valuable sides of human nature. C. were opposed to the established Church. D. were appalled by the excesses and violence in class struggle. E. were discredited by the establishment.

11.

Which of the following is a characteristic of Methodism? A. Rationality swamped by emotionalism. B. Reason subordinated to faith. C. It allowed as much scope as possible within the limits of current orthodoxy. D. A total absence of rationality. E. Intellectualization of emotions. T

101

Career Avenues

TEST 11 DIRECTIONS: After each passage you will find a series of questions. Select the best answer choice for each of the questions. Answer the questions based on what is stated or implied in the passage. PASSAGE-1 To stop science would create more problems than solutions. Aside from military considerations, it would be disastrous to freeze culture at its present high point. The highly technical civilization of the 20th century is like an airplane in flight, supported by its forward motion. It cannot stop without falling. If all the world’s inhabitants, for instance, learn to use natural resources as fast as Americans do now, many necessary substances will be exhausted. Scientists confidently count on improvements, including atomic energy, to provide ample substitutes. Present techniques won’t do it. Where will man’s curve of scientific progress take him ultimately? The surprises since 1900 have made scientists humble. They know that as science grows, it only penetrates deeper into mystery. Human knowledge may be visualized as an expanding sphere whose volume grows larger as its diameter increases. But the area of the sphere’s surface, its frontier with the unknown, increases as the square of the diameter. Beyond that frontier - nobody can know, until the frontier advances. 1.

2.

The title below that best suits the passage is: A. Technical civilization C. Stagnation of progress E. Scientific temper

B. New frontiers in science D. The 20th Century

Scientists feel that improvements which can provide substitutes are required to A. prevent useless loss of life. B. keep culture advancing. C. advance scientific knowledge. D. prevent exhaustion of basic resources. E. exploit unconventional sources. PASSAGE-2

The state of the primitive society at the moment of their first contact with the outer world makes a profound impression on the intruders because there is an extreme contrast between the two colliding ways of life – between the mental attitude of those who have hitherto been sheltered from challenges by an easy environment and mental attitude of people who have been challenged and have responded victoriously. This impression works so powerfully upon the intruders’ emotions and imaginations that it issues in mythology. The classic Greek example of the myth is the fable of the lotus-eater. The companions of odysseys fell in with the people of the land and when they had tasted the lotus fruit, they no longer desired to return home, but had only one thought, to remain where they were, feeding on lotus and thinking no more of their journey back to Greece. They were saved by odysseys, who dragged them to the ships and bade the rest of their companions come aboard with all speed, lest any man should eat the lotus and so forget about the voyage home. This mirrors a philosophic truth, which is substantially the same in the minds of the Greek, and the western observer. Alike they see that there is corresponding contrast between the mental attitude which has been induced in the primitive by his easy circumstances and the mental attitude which has been induced in themselves by strenuous life; they see that the primitive cannot and will not ever join them in running the race of civilization so long as an easy environment continues to shield him from the necessity; and finally, they see that they themselves, if they succumb to this insidious environment, will cease to run the race that is set before them. Ease is the enemy of civilization; the greater the ease of environment, the weaker the stimulus that environment offers to men.

102

Career Avenues

3.

It can be inferred from the passage that primitive societies make a tremendous impact on intruders because A. of the unambiguous parallels that exist between the different strata of society. B. it is a novelty to many. C. of the paradoxical aspects of those who led an unchallenged life and of those who faced challenges. D. of the subtle emotionality that differentiated one race from the other. E. of the powerful impact of the environment on the ways by which humans resolve their conflicts.

4.

The author’s attitude towards the introduction of the civilised world into primitive society is likely to be A. sympathy B. apathy C. encouragement D. antagonism E. indignation

5.

The author is most likely to agree with which of the following as a precondition for the growth of civilization? A. The right attitude toward accepting the transitoriness of natural environment. B. Strong environmental stimuli that pose challenges for survival. C. Emotional conformity with near and dear ones. D. A tacit acceptance of the uncertainty of the immediate environment. E. A strong belief in fatalism and inevitability. PASSAGE - 3

Man is constantly changing for the better to satisfy his never satisfied wants, both the inner and outer environment, both his physical and social worlds. During the recent stages of human history, modifications of his environment have been vast and particularly rapid. It is sometimes claimed that these large environmental changes have furthered some of man’s need at the expense of others, that human nature is not at home in the world of civilization which it has made for itself. One form of this criticism has found a well known expression in the contention that the modern environment balks some of our ‘’instinctive dispositions’’, that man is born with a set of dispositions related to the world of tropical or sub tropical wood and cave which he inhabited during millions of years of slow evolution and that these dispositions being suddenly dissociated from the stimuli of such surroundings, function uneasily and capriciously. According to this view, man is biologically adapted to the life of the fields and the woods, not to the life imposed by the city, the office, the factory. This claim that modern technology and social life necessarily frustrate hereditary dispositions has recently been widely popularized. There is, however, no evidence that the members of the simpler societies possessing relatively undeveloped systems of technology and social organization are more content with their lots or better adapted to the conditions of their lives than ourselves; the myth of the ‘’happy savage’’ has been shattered by modern anthropology. Still it is true that in modifying our environment to satisfy more fully some of our desires we may make it less favourable to the satisfaction of others. The more complex the life, the more complex must the environment be and the more complex the adjustment. The process of adjustments between modern man and his total environment is intricate and varied. 6.

Modern environment balks some of our “instinctive dispositions” is evident in A. man’s easy adjustment to modern life. B. modern technology and social life frustrating hereditary dispositions. C. man’s modifications of environment being vast and particularly rapid. D. simpler societies possessing undeveloped systems of technology. E. social organizations more satisfied with their life condition. PASSAGE-4

Much as an electrical lamp transforms energy into heat and light, the visual apparatus of a human being acts as a transformer of light into sight. Light projected from a source or reflected by an object enters the cornea and lens of the eyeball. The energy is transmitted to the retina of the eye whose rods and cones are activated. The stimuli are transferred by nerve cells to the optic nerve and then to the brain. Man is a binocular animal, and the impressions form the shape, colour, size, position, and motion of the things he sees.

103

Career Avenues

Photometry is the science of measuring light. The illuminating engineer and designer employ photometric data constantly in their work. In all fields of application of light and lighting they predicate their choice of equipment, lamps, wall finishes, colours of light and backgrounds, and other factors affecting the luminous and environmental pattern to be secured, in great part from data supplied originally by a photometric laboratory. Today, extensive tables and charts of photometric data are used widely, constituting the basis for many details of design. In most discussions of colour, a distinction is made between white and coloured objects. White is the colour name most usually applied to a material that diffusely transmits a high percentage of all the hues of light. Colours that have no hue are termed neutral or achromatic colours. They include white, off-white, and all shades of gray, down to black. All coloured objects selectively absorb certain wavelengths of light and reflect or transmit others in varying degrees. Inorganic materials, chiefly metals such as copper and brass, reflect light from their surfaces. Hence we have the term “surface” or “metallic” colours, as contrasted with “body” or pigment” colours. In the former, the light reflected from the surface is often tinted. Most paints, on the other hand, have body or pigment colours. In these, light is reflected from the surface without much colour change, but the body material absorbs some colours and reflects others; hence, the diffused reflection from the body of the material is coloured but often appears to be overlaid and diluted with a “white” reflection from the glossy surface of the paint film. In paints and enamels, the pigment particles, which are usually opaque, are suspended in a vehicle such as oil or plastic. The particles of a dye, on the other hand, are considerably finer and may be described as colouring matter in solution. The dye particles are more often transparent or translucent. 7.

Which of the following questions is NOT answered in the passage? A. What is phytometry? B. How does the visual apparatus in human beings work? C. What are colours with no hues termed? D. What is the difference between surface and body colours? E. What type of paints wear out soon?

8.

In form and style, the passage can best be described as A. an objective presentation. B. a biased exposition. C. a tentative hypothesis. D. an ambivalent portrayal. E. a flawed synopsis. T

104

Career Avenues

TEST 12 DIRECTIONS: Each passage in this group is followed by questions based on its content. After reading a passage, choose the best answer to each question. Answer all questions following a passage on the basis of what is stated or implied in that passage. PASSAGE-1 The fundamental objective of Magic realism is perhaps the single most devious, most innocently alluring trap for the adventurous writer from our part of the world. A dollop of “native” exotica, a large measure of popular culture, a handful of “scenes” of the nukkad variety, a generous helping of pure whimsy (the more sacrilegious the better), the merest silver of a plot and, voila, there you have it. Magic realism at its post-Rushdie best. In 1981, when Salman Rushdie unleashed his ‘Midnight’s Children’ upon an unsuspecting world, he opened the floodgates to a whole new way of telling stories; stories that were part autobiographical, part metaphorical, stories that dove in and out from the real to the imagined, often stories within stories that presented a vivid, multi-coloured, multi-textured tapestry that was neither pure history nor pure fantasy, but somewhere in between. It was this delightful coming together of the absurd and the actual, of history and the imaginative retelling of historical facts, of the autobiographical and the universal that critics were quick to dub magic realism. Rushdie single-handedly showed the world that great novels could be crafted from ethnic stories, especially stories from one’s childhood, using a completely “native” idiom and expression and a distinctly idiosyncratic use of the English language. Over the years, Rushdie’s near-iconic style and linguistic audacity have been hijacked by a wagonload of wannabes with one eye on the western media and the other on the pantheon of literary agents who can bag them their multimillion dollar contracts, while keeping their fingers firmly crossed about lucrative film deals with foreign television channels feeding the diaspora’s insatiable hunger for trivia from the homeland. The two books under review fall, quite lamentably, in this category. For both Uzma Aslam Khan’s ‘The Story of Noble Rot’ and Musharraf Farooqi’s ‘Passion in the Time of Termites’ can be viewed as having all the right ingredients in place, but somehow quite lacking in zest. The unbridled flights of fancy, the brilliant portrayals of local colour and custom, the almost compulsive, nearphotographic recall of detail that has become practically de rigeur in most post-colonial writing coming out from the Asian sub-continent are all there. But there is also a restlessness, a false note too many. It seems as though these two debut authors – both from Pakistan, both with experiences of having lived and studied in the West – are not fully at ease with themselves or with the stories they have to tell. They use the English language with dexterous ease but it is more a tool to astonish and dazzle the reader than to convey any real strength and imagination. Both Khan and Farooqi exhibit a certain desperate desire to impress. The ruling mantra is overkill. The more absurd, bizarre and farfetched, the better. Plot, characterisation, narrative – all are sacrificed at the altar of grandiloquence. Of the two, ‘The Story of Noble Rot’ perhaps demands a greater ability for “complete suspension of disbelief” from the reader. The title itself hangs on a very slender thread of credibility: “The sweet taste of the wine comes from the muscadelle grape, and the grayish mould that it attracts. The mould is lovingly called pourriture noble, noble rot.” It is this sweet – tasting but morally degenerating wine that brings the wealthy Mrs. Masood in contact with Malika, a poor carpenter’s wife. The first slip of forbidden wine marks the beginning of a relationship that inexorably changes the lives of the two women and all those who are drawn into Malika’s complex web of plans and dreams. There is the urbane Mr. Masood, factory owner and nouveau riche extraordinaire; Momin the child labourer with hennaed, deformed hands who works in his factory and dreams of fish and birds; the Pathan gardener with the sun dappled eyes; Mr. Saeed, the withdrawn scholarly widower who employs Malika to look after his hopelessly anglicised children; Chaudry the carpenter who can carve magic out of a piece of wood; Saima with her volatile swings of affection and fund of fabulous stories; the beauteous Laila in her impeccably starched and embroidered salwar khameezes; and a couple of wine – guzzling Frenchmen who gleefully clink away their glasses ‘to partnership’. The presiding deity of this lush and intensely imaged landscape is the churail (witch), Soomila, named after a legendary Sindhi princess.

105

Career Avenues

With so much material at her disposal, one presumes Uzma Aslam Khan set out to tell a story but it got lost some where in a maze of literary contretemps and stylistic peccadilloes. To wit: “The bride listened to the oxymoron of Sirkash’s footsteps as he padded through the desert with heavy lightness and hushed intensity.” And, “she was the fox in the chicken coop, the child in an adult theatre, Rushdie in Khomeini’s bed.” Musharraf Farooqi, the more gifted and sure-footed storyteller of the two, shows a similar tendency to dazzle and overwhelm. ‘Passion in the Time of Termites’ is the story of a town called Purana Shehr that has been besieged by an epidemic of termites. Farooqi has an unerring eye for detail and an often pungent way of making observations: “Nowhere else was despair found in greater number, running on two feet, than in court.” It is perhaps for this reason that he has chosen to leaven his story with a remarkable number of “scenes”: “At the photo studio”, “At the Dyer’s”, “The Letter writers at Work”, “The Knife sharpener at Work”, and several such drolly amusing and finely crafted vignettes. But this is where the tedium sets in. Farooqi is on to a good thing and does not know where to let go. So there is more of local colour by way of desi kushti (style of wrestling), katibs (calligraphers), pigeon breeders, kite fliers, cock fighters, fortune reading parrots, ad nauseum. Khan and Farooqi suffer from the worst excesses of much of the new genre of writing in English coming out from the Asian subcontinent. Masala has come to replace savour and characters have been taken over by caricatures. 1.

Which of the following best describes the term ‘magic realism’ mentioned in the passage? A. A story telling method in which the author unleashes a world of imaginations in front of the reader. B. A story telling method in which there are sub-stories inside a story, each one of which follows the realistic portrayal of its protagonist. C. A story telling method in which the real life conflicts and dilemmas of the author are portrayed very realistically, without embellishments. D. A story telling method that is a harmonious blend of real life incidents and fictional elements, of historical events and their imaginative narrations. E. The weaving of a story with the use of metaphors and allusions of common incidents.

2.

The author refers to Rushdie’s ‘The Midnight’s Children’ for which of the following purposes? A. To show that before Rushdie, elements of metaphor and native idioms were unknown to Asian writers. B. To point out how Rushdie introduced novel ways of telling stories and creative ways of using language. C. To express his resentment toward Asian writers in English for distorting historical facts to suit their agendas. D. To express apprehension about the modern day writer’s avoidance of content and form in their writings. E. To contrast such a writing style with the one in the discussion - especially in their use of metanyms and ironies.

3.

Which of the following can be inferred from the passage about post – Rushdie Asian writers in English? A. Many writers who followed Rushdie’s school of story telling failed to make an impact. B. Post-Rushdie writers successfully explained away the apparent contradictions in the use of pure history vis-à-vis pure fantasy. C. Critics were quick to relegate post-Rushdie writers into a slot that was neither into the category of fiction writers nor into a category of realists. D. Almost all writers who copied Rushdie’s inimical style used dazzling style and language, ridden with native jargon but ignored substance of writing. E. Many writers first wrote stories in the vernacular languages and then translated them to English themselves.

106

Career Avenues

4.

The author’s primary purpose in the passage is to A. compare and contrast two novels that are written in different settings but whose protagonists share common symbolic characteristics. B. critique post-Rushdie writers for their having clung on to the ornamental view of writing instead of following the modern view of story telling. C. review two books that followed Rushdie’s style of story telling but failed to weave the magic that Rushdie did successfully. D. summarise the works of post-Rushdie Asian writers that were written primarily to attract western television channels. E. discuss trends in English writing in contemporary times and to lament their meaninglessness.

5.

Which of the following, in the author’s analysis, is/are the feature’s of ‘The Story of Noble Rote’ and ‘Passion in the time of Termites’? I. The depiction of ethnic elements II. Uneasiness with English Language III. Unrestrained use of imagination A. II only B. I and II C. I only D. I and III E. I, II and III

6.

Which of the following best summarises the author’s attitude toward Aslam Khan’s and Farooqui’s books? A. unrelenting criticism B. subtle admiration C. surprising neutrality D. objective eulogy E. unmitigated ambivalence

7.

It can be inferred from the passage that the author would admire writers who A. can succinctly and piercingly present human dilemmas, with their characteristic styles. B. draw from iconic styles of writing such as use of metaphors and native idioms. C. weave stories around meaningful plots, characterizations and narrative styles. D. tell stories from their heart than from their rational thoughts, relying more on form than content. E. write stories in prolific, trenchant languages for one’s own creative satisfaction rather than for an audience.

8.

Which of the following ways does the author contrast the two novels under review? A. Farooqi’s novel fails to bring in detailed narrations whereas Uzma Khan uses detailed characterization unerringly. B. ‘In Passion in the Time of Termites’, the author displays pungent style and a nearly perfect attention to detail, which is missing in the other novel. C. Both novels make extensive use of real life experiences, however Uzma Khan excels in making use of childhood events to develop the subplots. D. Neither of the authors depicts events clearly so that the pure narratives can leaven the fantasy of the readers. E. One novel is more rustic and rural and the other is woven in a more urban setting.

9.

The author is of the opinion that ‘Passion in the Time of Termites’ would’ve clicked if A. the author knew how to take his story further from the dazzling and overwhelming rhetoric and observations to a meaningful climax. B. the author used English language with flawless ease to achieve idiosyncratic novelties just as Rushdie did. C. the author did not tamper with ethnic realities that led to moral degenerations and erosion of values among the characters. D. the author lived in the Asian region and closely studied the ethnic and multicultural traditions of its people, for better portrayal. E. the author himself translated and revised his original book written in a vernacular medium.

107

Career Avenues

10.

Which of the following is a weakness that the author finds in ‘The Story of Noble Rot”? A. Lack of imagination. B. Oft repeated ironic disjunctions. C. Excess use of irony and metaphor. D. A strong desire to impress the reader. E. A detached and disinterested writing style.

11.

Which of the following can be inferred from the passage about the new generation of Asian writers in English? A. Their novels are primarily written for commercial benefits. B. They live on foreign land and write about Asian cultures in a detached way. C. They are driven by Rushdie school of thought, and therefore fails to make an impact. D. They all have moved away from the more predictable realistic writings to completely fictional styles. E. They are unaware of the meter and style used by their original counterparts. T

108

Career Avenues

TEST 13 DIRECTIONS: After each passage you will find a series of questions. Select the best answer choice for each of the questions. Answer the questions based on what is stated or implied in the passage. PASSAGE-1 The fundamental objectives of sociology are the same as those of science generally - discovery and explanation. To discover the essential data of social behaviour and the connections among the data is the first objective of sociology. To explain the data and the connections is the second and larger objective. Science makes its advances in terms of both of these objectives. Sometimes it is the discovery of a new element or set of elements that marks a major breakthrough in the history of scientific discipline. Closely related to such discovery is the discovery of relationships of data that had never been noted before. All of this is, as we know, of immense importance in science. But the drama of discovery, in this sense, can sometimes lead us to overlook the greater importance of explanation of what is revealed by the data. Sometimes decades, even centuries, pass before known connections and relationships are actually explained. Discovery and explanation are the two great interpenetrating realms of science. The order of reality that interests the scientists is the empirical order, that is, the order of data and phenomena revealed to us through observation or experience. To be precise or explicit about what is, and is not, revealed by observation is not always easy, to be sure. And often it is necessary for our natural powers of observation to be supplemented by the most intricate of mechanical aids for a given object to become “empirical” in the sense just used. That the electron is not as immediately visible as is the mountain range does not mean, obviously, that it is any less empirical. That social behaviour does not lend itself to as quick and accurate a description as, say chemical behaviour of gases and compounds does not mean that social roles, statuses, and attitudes are any less empirical than molecules and tissues. What is empirical and observable today may have been non-existent in scientific consciousness a decade ago. Moreover, the empirical is often data inferred from direct observation. All of this is clear enough and we should make no pretence that there are not often shadow areas between the empirical and the nonempirical. Nevertheless, the first point to make about any science, physical or social, is that its world of data is the empirical world. A very large amount of scientific energy goes merely into the world of expanding the frontiers, through discovery, of the known, observable, empirical world. 1.

According to the passage, scientists are not interested in theological explanations because A. scientists tend to deny the existence of God. B. theology cannot explain the fundamental nature of the universe, i.e. change. C. theological explanations cannot be subjected to empirical testing. D. theology fails to accommodate the complexities of human behaviour. E. scientists are concerned primarily with data gathering.

2.

The primary purpose of the passage is to A. show that explanation is more important than discovery. B. prove that sociology is a science. C. explain the major objectives of sociology. D. discuss scientific method. E. describe social behaviour.

3.

Which of the following statements best agrees with the author’s position? A. Science is the formulation of unverified hypothesis. B. Explanation is inferred from data. C. Causation is a basis for explanation. D. Generalisation is a pre requisite for explanation. E. Empiricism is the science of discovery.

109

4.

Career Avenues

The author’s main point in the first paragraph may best be described by which of the following statements? A. Science and sociology are interdisciplinary. B. The first objective of sociology is discovery. C. Discovery without explanation is meaningless. D. Both discovery and explanation are fundamental to building a science. E. It takes a long time before relationships of data are discovered. PASSAGE-2

Today, the educational value of appealing to the eyes as well as to the ear has been fully recognized, and the lack of suitable materials, which has in the past been a handicap to visual education, is being made good. The exhibition is one important medium for visual education that has gained greatly in popularity in recent years. Exhibitions may range in subject from the purely practical – such as those sponsored by some authorities for health and housecraft demonstrations – to collections of artistic and historical interest like those circulated by the Victoria and Albert Museum. A natural focus for this particular form of education through the eye is the local museum. Museums and galleries are, indeed, playing an increasingly important part in the educational life of their localities; and possibly their greatest contribution can be made in the field of adult education. If they are valuable as a base for the display of material supplied from outside sources, they have a more important function in fostering local distinctions and traditions of all kinds. This they can do by their permanent collections, by special exhibitions shown on their own premises, or by traveling exhibitions circulated to other centers in the locality – for example, to colleges of further education or to community centers. An outstanding value of the exhibition method is that it does not provide instruction only, but by the display of works of craftsmanship and art it gives training in appreciation, a sense of quality and individual character. Another most important medium for visual education is the cine-film and at the present time the supply of specially produced educational films is being rapidly increased. The film, like the exhibition, is well adapted for the demonstration of practical activities and skills of all kinds, and to the communication of cultural ideas and values. But it has a further distinctive contribution of particular importance in education. Owing to the exceptionally strong emotional impact of the cinema, films dealing with controversial topics – as, for example the whole range of modern social questions – can be relied upon to provoke discussions and arouse interest in current problems. 5.

A suitable title for the passage would be A. Cine films: The educational angle. B. Museums and Galleries. C. Visual Education: Importance and Approaches. D. Films and craftsmanship. E. Emotional cinema.

6.

The film strip is referred to as “humbler” possibly because A. it was invented by Mr. Humbler. B. it consists of a series of static pictures while a film has continuous movement. C. it is not as advanced a visual form as the cine film. D. it is cheap and can be easily handled. E. it comprises a series of dynamic pictures while the film is static.

7.

The passage can best be described as A. indoctrinating B. argumentative D. biased E. informative

C. critical

PASSAGE-3 It is said that more stress is placed on achievements in athletics than in academics in our public schools. In fact in some schools games are compulsory. They maintain that it is tyranny to compel boys with no athletic bent to spend hours of misery on a cricket or football field. They should be left to themselves, when they would occupy their time more usefully and enjoyably on some profitable creative hobby. The drawback ofthis argument lies in the facile assumption that every non-athlete has a profitable hobby. This is not true; even if it were, model engineering or

110

Career Avenues

stamp collecting is no substitute for being out in the fresh air, exercising the muscles, and having contact with other human beings. I would compel all boys to play games, but I would protect them fiercely from the here-say that it was their duty to be good at games. One should guard, too, against the youthful idolizing of athletes which tends to upset a boy’s sense of values, and may do actual harm to the objects of this hero worship. The schoolboy athlete does suffer enormously through being adorned at an early age with a fake halo of artificial light. From Preparatory School to University his career is a triumphal procession. Then he becomes a legend for the future, one of the greatest products of the school which is proud to call him her son, although she may have taught him nothing except to play football – which he could do already. Not until he hangs his boots up and goes into the open market of the world does he realize his true value – or lack of it. It would be better for everyone if this tinsel pageantry was stripped from games at an early stage. The community must be taught to lay emphasis on the essential triviality of talents which are merely physical, unless we are willing that our ideals should be those of the jungle. The almost muscle-bound weight-lifter may seem a pattern of manhood in contrast with a weedy classical scholar, but he would cut a poor figure, after all, beside a gorilla. It is for other than physical values that man is the paragon of the animals, and those are the values that really matter to education. 8.

Public Schools are criticized for their A. equal emphasis on sports and academics. B. unequal emphasis on sports and academics. C. inability to develop profitable hobbies. D. greater stress on athletics than on academics. E. clinging on to conventional mode of functioning.

9.

Idolising of young athletes may harm the athletes because A. they may not live up to expectations in the professional field. B. their physical prowess is inadequate. C. mere physical prowess cannot ensure success in the open market at a later date. D. a lot of competition awaits on the field. E. failures would affect the mental as well as physical health of the individual.

10.

Man is the paragon of the animals because A. of his superior intellect. B. of better muscle power. C. of both superior intellect and strength. D. he is emotionally superior. E. he is fit to survive. T

111

Career Avenues

TEST 14 DIRECTIONS: Each passage in this group is followed by questions based on its content. After reading a passage, choose the best answer to each question. Answer all questions following a passage on the basis of what is stated or implied in that passage. PASSAGE-1 When we use a word in speech and writing, its most obvious purpose is to point to some thing or relation or property. This is the word’s “meaning”. We see a small four-footed animal on the road and call it a “dog”, indicating that it is a member of the class of four-footed animals we call dogs. The word “dog,” as we have used it there, has a plain, straightforward, “objective” meaning. We have in no way gone beyond the requirements of exact scientific description. Suppose, however, that we had called that same animal ”mongrel”. The matter is more complicated. We have used a word which objectively means the same as dog of mixed breed but which also arouses in our hearers an emotional attitude of disapproval toward that particular dog. A word, therefore, can not only indicate an object, but can also suggest an emotional attitude toward it. Such suggestions of an emotional attitude do go beyond exact and scientific discussions because our approval and disapproval are individual – they belong to us and not to the objects we approve or disapprove of. An animal, which to the mind of its master is a faithful noble dog of mixed ancestry, may be a “mongrel” to his neighbour whose chickens are chased by it. Once we are on the outlook for this difference between “objective” and “emotional” meanings, we shall notice that words, which carry more or less strong suggestions of emotional attitude, are very common and are ordinarily used in the discussions of such controversial questions as those of politics, morals and religion. This is one reason why such controversies cannot yet be settled. There is a well-known saying that the word “firm” can be defined as follows “I am firm, thou art obstinate, he is pigheaded”. That is a simple illustration of what is meant. “Firm,” “obstinate”, and “pigheaded” all have the same objective meaning – that is following one’s own course of action and refusing to be influenced by other people’s opinions. They have, however, different emotional meaning of strong approval, “obstinate” of mild disapproval, “pigheaded” of strong disapproval. 1.

The author’s point in the first three paragraphs is that A. there is no real difference between calling a dog a mongrel or calling it a dog of mixed breed. B. “a dog of mixed breed” is an emotional term. C. “mongrel” is an objective term. D. words may suggest emotional attitudes as well as objective meanings. E. “sticks and stones can break my bones but names will never hurt me.”

2.

The author maintains that A. in discussing scientific subjects, emotional words are often used to make meanings clearer. B. in discussing controversial questions, objective terms are generally used to help clarify meanings. C. in discussing scientific subjects, objective terms are generally used to help clarify meanings. D. in discussing controversial questions, emotional terms are used very often. E. the use of emotional terms has little or no influence in political or religious discussions.

PASSAGE-2 To learn means “to gain knowledge through experience”; but one of the meanings of “experience” is “to perceive directly with the senses”, a meaning that appears initially in the definition of know. But knowledge is defined, among other things, as learning (erudition) and as familiarity or understanding gained through experience, and learning is defined as acquired knowledge. So we come a full circle. Consider the two further terms memory and remembering. Memory is the faculty of retaining and recalling past experiences, or the ability to remember and remembering is defined as recalling an experience to mind or thinking of it again. These clearly form an interconnected cluster of concepts. More than that, in our everyday dealings, memory

112

Career Avenues

(or remembering) is one of the primary ways by which we know things and by which we support knowledge-claims. The status accorded in the law courts to testimony from first hand witnesses attests to the evidential power-topersuade of direct memories: “How do I know that John stole the money? Because I remember seeing him with his hand in the till.” The existence of such memories constitutes a prima facie case for the knowledge-claim – unless other considerations enter to cause doubts. In fact, one of the earliest uses of the psychological study of memory was to undermine its validity for supporting knowledge-claims. These studies showed that many memories of remote events reported in testimony were inaccurate, distorted and subjectively biased. These mistakes were especially likely in memories of emotionally laden crimes, fights or disasters. One of the most engaging issues within the theory of knowledge is the question of how concepts and knowledge arise, and what is the relation between experience and the organization of the mind. Two opposing positions on this matter are empiricism and rationalism. These have been constant combatants within the intellectual arena for centuries, and strong forms of them are still recognizable today in “scientific” psychology. Empiricism is the view that experience is the only source of knowledge. Special emphasis is given to sensory experience, although some knowledge is derived from intellectual reflections regarding relations among experiences. Our ideas are derived from sense impressions, either as direct copies of sensory impressions or combinations of several simple or complex ideas. The sensory impression of an object (say, an orange) is decomposable into sensory qualities – sensations corresponding to its colour, smell, size, texture, taste and so on. These sensory qualities become connected (or “associated”) in the mind because they occur closely together in time or in space as we interact with the object. The idea of an orange is complex, but reducible to inter-associations among simpler, more primitive ideas. Further “knowledge” acquired about oranges can be expressed by associating this complex of ideas to the other relevant ideas – for example, that oranges are fruits and are edible. The features of empiricism are (1) sensationalism, the hypothesis that all knowledge is derived through sensory experience; (2) reductionism, the thesis that all complex ideas are built up out of a basic stock of simple ideas, and that complex ideas are in turn reducible to these simple ideas; (3) associationism, the thesis that ideas or mental elements are connected through the operation of association of experiences that occur closely together in time (contiguity), and (4) mechanism, the thesis that the mind is like a machine built from simple elements with no mysterious components. Empiricism involves two basic learning mechanisms: (1) internal representations of simple ideas (“memory images”) which originate by simply copying their corresponding sense impressions into the memory store; and (2) complex ideas are formed by connecting together in memory simple ideas that are experienced contiguously; they are connected by an associative bond. The memory that event A was followed immediately by event B is recorded in memory as an association from idea ‘a’ to idea ‘b’. This is in effect copying into memory the fact of the cooccurrence of mental contents ‘a’ and ‘b’. Such associations can record temporal or causal sequences of events, such as striking a match – lighting the match – heat – fire. Activating or reviving these associative sequences from memory is the presumed method by which the mind moves in thought from one idea to another. This method accounts for the order of succession in a chain of ideas during idle thinking or goal-directed thinking. To illustrate goal-directed thinking, suppose that the final event in a chain becomes a goal (“I want to eat ice cream”). Then thinking of that goal will call to mind an immediate precursor of it from the past (“Buy some at the shop”), and that thought in turn will bring to mind what must first occur in order to cause it (“Get money and go to the shop”), and so on. Thus, a goal-directed chain of ideas may unwind backward from effect to cause until it arrives at some action that can be performed now to initiate the thought-out sequence. Associative chains that reflect causal sequences can be used in two basic ways : they may run forwards from ‘a’ to ‘b’ to ‘c’ to predict, anticipate, or expect future events from the present event or action; and they may run backwards from ‘c’ to ‘b’ to ‘a’ to explain why events happened or to plan how to bring ‘c’ about. Predicting, explaining, and planning are fundamental skills by which we deal with the world, and the associative theory suggests ways to do these things. Empiricists included in their theory of mind the notion of “reflection”, whereby the mind supposedly can call up from memory, several ideas, compare them, and arrive at some conclusion which would be recorded as another association. The idea of reflection was needed to explain how we gain knowledge by abstraction, inference and deduction. By “abstracting” the common, critical properties out from the varying, accidental, nonessential properties, we form a general concept of a type of thing from experience together with a set of its widely varying examples. In deduction, we bring into conscious reflection a logical consequence of other things we know. Thus, if we know that Bill is taller than John, and John is taller than Pete, then upon reflection the mind can deduce (and

113

Career Avenues

store in memory) that Bill is taller than Pete. According to the empiricist doctrine, reflection is the only mechanism the mind has available to free itself from being a totally passive recorder of sequences of sensory impressions. 3.

According to the passage, learning can be defined as A. sensory reaction to stimuli. B. a response to external cues. C. recalling past experiences. D. retaining past experiences. E. acquisition of knowledge.

4.

“These clearly form an interconnected cluster of concepts”( para 2) The statement most plausibly implies that A. memory and remembering are related to acquisition of knowledge. B. one is able to recall only events that are associated with the present. C. experience and perception are interrelated concepts of learning. D. knowledge in everyday dealings is connected to one another. E. learning and teaching are two sides of the same coin.

5.

“The psychological studies conducted earlier showed that many memories were inaccurate”. This finding was used in A. memories of emotionally-laden crimes and disasters. B. first-hand witness evidence in law courts. C. prima facie case for knowledge claim. D. undermining the validity for supporting knowledge claims. E. refuting the inter-dependence of stimulus and response.

6.

According to the passage, empiricism and rationalism are A. complementary schools of thought on the acquisition of knowledge. B. contradictory epistemologies of how concepts are acquired. C. opposing viewpoints on the mental processes of memory and learning. D. theories that define acquisition of knowledge as the product of only sensory experience. E. parallel ideologies followed by educationalists.

7.

According to empiricists, learning takes place when A. sensory qualities become connected in the mind to form sensory impressions. B. associated sequences from memory are activated and derived. C. the mind reflects several ideas already acquired and arrives at some conclusion. D. through reflection, the mind frees itself from being a totally passive recorder. E. visual or auditory stimuli are absorbed in to the brain.

8.

According to the passage, abstract learning is A. learning through intellectual reflections without any use of sensory experience. B. bringing in a logical sequence of things we know and to deduce from it. C. forming a general concept of a type of thing with a set of its widely varying examples. D. the process of rejecting accidental, non-essential properties of an idea. E. learning without concept formation.

9.

According to the passage A. goal-directed thinking is unidirectional. B. idle thinking can occur in forward and backward directions. C. goal-directed thinking may happen backwards from effect to cause. D. goal-directed thinking is synonymous with abstract learning. E. all thinking styles are dynamic.

10.

Associative chains that reflect causal sequences can be used I. to explain why events happened. II. to predict future events from the present ones. III. to plan a certain future event. A. II only B. II and III C. I and II D. I, II and III E. III only

114

Career Avenues

TEST 15 DIRECTIONS: After each passage you will find a series of questions. Select the best answer choice for each of the questions. Answer the questions based on what is stated or implied in the passage. PASSAGE-1 Critical accounts of travel writing pertaining to the East have focussed attention on the ethnocentric and Orientalist assumptions inherent in many of these travelogues. Invoking this tradition, Purabi Panwar directs attention to three authors whose fictional works have been much discussed but whose travel writing has been characterised as marginalia. Or, as in the case of V.S. Naipaul’s travelogues, righteous and self-righteous indignation has obviated the possibility of any balanced assessment. Panwar traces the trajectory from the works of Kipling, writing at a time when it seemed the sun would never set on the British Empire; to E.M. Forster, when the glorious days of British supremacy were already over; to Naipaul, who writes about post-Independence India clinging on to the tattered legacy of the Empire. The “works” mentioned in the title of the book include newspaper articles, diary entries and descriptive accounts in journals as well as fictional and non-fictional writings about India. Discursive and extra-discursive links are established between them. Consider for example, Forster’s review of Kipling’s travel essays entitled “The Boy Who Never Grew Up” which appeared in The Daily Herald. When we are told that while on a visit to Simla, Naipaul remembers his favourite author, Kipling’s fondness for the place, it appears that the sequence of events linking Kipling, Forster and Naipaul has come a full circle. Another instance of discursive linkages is the meticulous research into what these authors were reading around the time they travelled in India and how it may have coloured their narrational stance. It is observed that they attempted to understand India through a plethora of sources: the works and translations of Orientalists like William Jones; subjective accounts of parts of India by the colonial officers, James Tod and William Sleeman and works which represent the distillation of the “essence” of India such as The Mahabharata, The Bhagvad Gita or the Kama Sutra. The extra-discursive links can be seen in their reactions to the places they visited. Kipling’s and Forster’s impressions of the Taj Mahal are probably symptomatic of the different periods of colonial rule. Naipaul’s glorification of the natural beauty of Kashmir is similar to Kipling’s aesthetic appreciation of the Taj. That this obfuscates Kipling’s and Naipaul’s cognisance of the ground realities – colonial oppression in the former’s times and the turmoil in Kashmir in the latter’s – is indicated in the analysis. The ambivalent and inconsistent standpoints of these authors on issues like religion, race relations, nationalist and anti-nationalist politics are also examined. Panwar observes how Kipling’s positive description of the Babu in his travel writing contrasts with the portrait of the Anglophile buffoon Hurree Babu in Kim. In a similar vein, Forster’s views on race relations, although marking an advance on Kipling’s jingoistic imperialism, were not as progressive as his professions of liberalism would lead one to expect. There is, as Panwar notes, hardly any reference to the nationalist movement in India in Forster’s diaries and letters. This was probably because he was unwilling to see Indian resistance towards the British as an expression of nationalist ideology Forster attributed to “the repressive measures of the English in India”. Naipaul turns his gaze on the separatist movements construed as anti-nationalist in post-independence India. His post-modernist stance on these separatist and fundamentalist movements and their ideologues – the Kashmir liberationists, the Khalistanis, the Dalit Panthers and the Shiv Sainiks – somehow evades an assessment of the dominant and dangerous roles they have come to play in recent times. Reading the book, one comes across many references to people, places and works, which may be common parlance for specialists in the field but not for the general reader. Keeping this in mind, detailed notes have been provided. Those pertaining to the chapters on Kipling and Forster read like a veritable who’s who of British India. These explanations, comparable in a sense to hypertext links on the World Wide Web, can be used to undertake related and

115

Career Avenues

independent fields of enquiry in the social life, literary tastes and the representation of political events in British India. It is the chapter on Naipaul which best illustrates the problems of the post-colonial revolutionary enterprise, indicated by the subtitle. An outsider-insider perspective like Naipaul’s and its credibility are questions much debated in post-colonial theory and practice. The issue is not, perhaps cannot be, resolved without recourse to facile generalisations about the geographical and cultural “location” of these writers attempting a critique of the multifacetedness comprising “India(s)”. No simplistic answers are possible. Indeed, in the spirit of open-ended enquiry, none are attempted in this informative and insightful account of the representation of India in the works of Kipling, Forster and Naipaul. 1.

The author of the review is mainly concerned with A. enumerating the similarities in the fictional works of Kipling, Forster and Naipaul. B. criticising Naipaul’s writing on post-Independent India and his clinging on to the tattered legacy. C. reviewing three authors - Kipling, Forster and Naipaul – whose travel writings have been considered marginalia. D. questioning the ambivalent and inconsistent standpoints of Forster, Naipaul and Kipling on nationalist politics. E. setting forth three literary considerations in writing a travelogue.

2.

Purabi Panwar, in the author’s analysis, drew upon which of the following works I. Newspaper articles II. Diary jottings III. Non-fictional writings about India A. II and III B. I only C. I and II D. II only E. I, II and III

3.

The author states that Naipaul’s travel writings were not assessed objectively because A. of righteous and self-righteous indignation. B. of lack of righteous indignation. C. of Naipaul’s open support for the British Empire. D. the period of British supremacy had ended when those travelogues were published. E. of natural resistance to new interpretations.

4.

Purabi Panwar is justified in bringing the three authors under one umbrella as A. these writers did not follow the ethnocentric or orientalist paradigm. B. there are discursive links among the travel writings of the authors under review. C. they have been visibly influenced by their reading in Indian philosophy which have coloured their narrations. D. they belonged to the school of thought, which openly showed its resistance to the colonial rule. E. the three authors have similar cultural and literary traditions.

5.

Panwar attributes a lack of reference to the nationalist movement in Forster’s diaries to A. Forster’s aggressive views on racial relations. B. Forster’s partisan views on racial relations. C. Forster’s blind faith in anti-nationalist politics. D. Forster’s unwillingness to see India’s resistance to imperialism as an expression of ideology. E. Forstor’s compliance with India’s resistance to divisive forces.

6.

The author perceives Naipaul’s stance on separatist movements in India A. as lacking consistency resulting from his ambivalent attitude toward religion. B. as ignoring an objective assessment of the pernicious roles these movements played. C. with apprehension resulting from Naipaul’s tacit acceptance of anti-nationalist politics. D. as questionable in the wake of his controversial post-colonial revaluations. E. as weakened by disinterested assessment.

of the three authors?

116

Career Avenues

PASSAGE-2 Audrey Thomas is a well-known Canadian fiction writer with six collections of short stories and eight novels to her credit. Her writing spans nearly three decades, starting with a collection of short stories, Ten Green Bottles (1976) to her recent novel Coming Down From Wa (1995). Dismembering / remembering Fictions of Audrey Thomas by Krishna Sarbadhikary reads her work from the feminist angle and tries to deconstruct the formal, linguistic and structural aspects of the texts, looking at the novelist’s innovative use of form, language and narrative voice. This is the first full length book on the novelist though, over the years, her works have inspired a number of book reviews and articles. Sarbadhikary makes it clear in the introduction that she does not intend to study Thomas’ works chronologically. Rather, she tries to bring out “the evolution of the female subject from self-divided, conflictual identity, unable to resolve contradictions of her experiences to a mature woman artist”, in the first three chapters, to use her own words. These three chapters take up Thomas’ long fiction in three different clusters from different thematic angles. Thomas’ concern with identity is taken up in the first chapter and the author examines the attempt of Thomas and her female narrator (are the two voices different?) to “… dislodge male-defined female identity” in a process of “re - naming” and “re - membering” the self, as perceived in the trilogy, Songs My Mother Taught Me, Mrs. Blood and Blown Figures. The second chapter discusses Intertidal Life And Graven Images, focusing on the Freudian family plot, featuring triangular relationship of the child with its parents, followed by her attempt to reconstruct a new version of the mother-daughter plot. In the third chapter, an attempt is made to move the subject further ahead “… in the process of establishing the self and aspires towards a progressively maturing artistic vision aligned to the material”. Africa is the setting in many of Thomas’ stories and novels, a country where she has lived for several years and to which she returned twice. The last visit in the nineties produced her latest novel Coming Down From Wa. Sarbadhikary finds this fascination with Africa problematic specially because of its “autobiographical consciousness” which strengthened the position of her female protagonists. Sarbadhikary attempts to “… dismember the autobiographical consciousness of the author and her protagonists” in order to examine and “re – member” from her perspective “how certain choices of narrative, form and language have been made, in order to inscribe that colonized other”. Sarbadhikary finds a remarkable similarity between the “desire for quest” (not very clear) and for narrative control in Thomas’ characters and Conrad’s heroes, which explains this chapter heading. She tries to bring this out through a parallel textual analysis. As a “Third World reader” (if one may use the term without denigrating oneself), she attempts to pinpoint Thomas’ limitations because of her specific position. She points out the difficulties of identification prompted by Thomas in terms of gender, “which would magically transcend race, class, or ethnic barriers” because of the particular form of narrative adopted by her. “Africa as a place associated with blood, miscarriages and lost children has been the repetitive structural image from Mrs. Blood to Coming Down From Wa”. As a full length work of Audrey Thomas, Dismembering / Re – Membering … treads new paths and brings one closer to one of the well known feminist writers in Canada. Sarbadhikary is to be commended for her work, and the bibliography at the end of the book should be useful to any scholar working in that area. However, this reviewer has a few suggestions to make. First, an index at the end would have made the book more useful to scholars for whom it is primarily meant. Second, though the sources of quotations are generally in parentheses, some explicatory notes would have been in order. Last, since a book like this is often read by the general reader, a comparatist outlook involving other Indian / African writers with something similar, may have enabled an added dimension.

117

Career Avenues

7.

The passage can be best described as A. A critical review of the compendium of writings of Audrey Thomas. B. A preface to the collection of writings of Audrey Thomas. C. A review of the analysis of Audrey Thomas’ fiction. D. A full length book on Canadian feminist writers. E. A rebuttal to a previous interpretation of a literary work.

8.

The author of the passages concedes which of the following as attributes of Audrey Thomas’ fictions? A. Creative use of style and language. B. Extensive use of imagery. C. Their mystery and irony. D. Their chronology and sequence. E. Overemphasis on style that matter.

9.

It can be inferred from the passage that Sarbadhikary’s study of Thomas’ fictions is mostly confined to A. Bringing out the theme of female identity. B. Exploring formal and linguistic aspects of Thomas’ writing. C. Analysing gender – inequality prevalent in Thomas’ short stories. D. Viewing Thomas’ fictions from various thematic angles, mostly ignored by the author herself. E. Thomas’ portrayal of the unequal relationship between sexes.

10.

It can be inferred from the passage that Sarbadhikary would most probably agree with which of the following features of Thomas’ fiction? A. The author and the protagonist of the novel are in conflict with each other’s identity. B. The author speaks through the protagonist of the novel. C. The protagonist in Thomas’ fiction is submissive whereas the author herself tries to voice for gender equality. D. There is no chronological or cumulative progression of themes in Thomas’ short stories. E. The author fails to identify with the sensibilities of the characters.

11.

It can be inferred from the passage that Sarbadhikary doesn’t believe that A. Audrey Thomas is not influenced by her real life experiences in her conception of female protagonists. B. Audrey’s works of fiction can be construed from more than one thematic angles. C. there are no meaningful links between and among the fictional works of Audrey Thomas. D. there are many similarities between Audrey’s themes and any other writer’s. E. Audrey was influences by her genre of writers.

12.

In Sarbadhikary’s review, Thomas’ use of specific narrative technique has resulted in which of the following consequences? A. Difficulty in gender identification. B. Repetition of images. C. Denigration of the human self. D. Devaluing the ‘desire for quest’. E. Her writing becoming more daring and militant.

13.

The author would most probably admire writers with which of the following writing skills? A. Writers who use narrative styles involving symbolism and imagery. B. Writers who would give detailed explanations of citings used in the article. C. Writers who are capable of putting ideas in an economical page or two, in a trenchant and piercing manner. D. Authors who compare the style of writing of one writer with others’. E. Writers who make use of technical devices in creating subplots. T

118

Career Avenues

TEST 16 DIRECTIONS: After each passage you will find a series of questions. Select the best answer choice for each of the questions. Answer the questions based on what is stated or implied in the passage. PASSAGE-1 Religious scriptures, like any other texts, can be interpreted in remarkably diverse ways to suit different, often radically contrasting, social and political agendas. If we in India have long been plagued by arch-reactionary forms of religion that are used by vested interests to promote social conflicts and bolster the interests of ruling elites, religion has had another face to it (although these days far less visible) as a means for articulating protest against injustice and inequality. It is this latter, largely submerged, tradition that ‘liberation theology’ seeks to recover and articulate. This book provides a general overview of what liberation theology is all about. Although the contributions are all written from within a Christian perspective, it provides valuable insights, lessons as well as inspiration for people from other faith traditions who are concerned about how their religions can be used to provide resources in the struggle for a more just and humane social order. In his opening essay, Christopher Rowland delineates the main characteristics of liberation theology as it has developed from the late 1960s onwards. In contrast to traditional Western Christian theology, liberation theology, a product largely of theologians from the so called ‘Third World’ sees Jesus as a radical crusader for social justice, as a revolutionary almost in the Marxian mould. It understands him as having exercised what is called ‘the option for the poor’, fighting along with the poor against the rich and oppressors generally. For liberation theologians, the notion of sin, which is so central to Christianity, is seen as not just personal and individual, but as more encompassing ‘sinful’ social and political structures that give rise to poverty and social injustice. Salvation, accordingly, is seen not as a pie in the sky, castles and rivers of milk and honey in heaven. Rather, as Rowland tells us, it is also seen in distinctly this worldly terms, as ushering in the Kingdom of God on earth, establishing a society free from injustice and inequity. In other words, liberation theology has a dual task before it – to liberate the poor as well as to liberate theology itself, making it a tool for social justice. The major contributors to the development of liberation theology have been from South America. One of the pioneers and well-known liberation theologians is Brazilian Gustavo Gutierrez, whose article in this volume traces the origins and main thrust of the movement in Latin America, showing how it has had to struggle against the Church establishment as well as ruling elites, which have, he says, been in league with each other for centuries. Barring the Philippines, no country in Asia has a Christian minority, but this has not prevented Asian Christians from seeking to fashion liberative theologies moulded to suit their own contexts of poverty and religious pluralism. In an incisive essay, the Sri Lankan Catholic writer Bastiaan Wielenga provides an overview of writings and efforts of Sri Lankan, Korean and Indian Christian scholars in developing new expressions of Christianity that can help both in the struggle for justice and for improving relations between people of different faiths. If poverty and injustice and the struggle against them were the concerns of the early liberation theologians, later writings on the subject have looked at other forms of oppression and have sought to develop new forms of Christian expression to deal with these as well. Thus, black liberation theology has, as Edward Antonio writes, witnessed a remarkable development in recent years, both in the USA and in Africa. Black liberation theologians bitterly castigate the mainstream churches for either explicitly or subtly sanctifying racist exploitation. For them, Jesus is black, not in the sense of actual physical colour but as siding with the black masses in their struggle against racism and white domination. Likewise, feminist Christians have developed theologies that question the patriarchal traditions so ingrained in traditional forms of Christianity, seeing Jesus as a partner in their struggle for gender justice and equality, as Mary Grey tells us in her well – argued essay. Liberation theology is being put into actual practice in Latin America through small groups, called base ecclesiastical communities, Andrew Dawson writes. Small groups of people reflect together on passages of the Bible

119

Career Avenues

and seek to relate these to their own existential situations, gaining inspiration for their struggle against oppression as well as conservative elements within the Church. Not unexpectedly, the Vatican has hardly taken kindly to the challenge of liberation theology. Peter Hebblewaite, in an excellent piece, shows how through a policy of calculated ambiguity and outright opposition, the Vatican has sought to dampen the radicalism of the movement, going so far as to defrock or silence vocal priests. While the Vatican’s reluctance to embrace liberation theology is perfectly understandable, given its long history or connivance with ruling powers, liberation theology has other challenges that it must contend with if it is to survive and prosper. The collapse of the Soviet Union, Valpy Fitzgerald writes in his essay, calls for new ways of thinking about alternate economic models by liberation theologians, for clearly socialism like any other economic system, has its share of limitations to which earlier advocates of liberation theology devoted but gave little attention to. More attention also needs to be paid to a political philosophy that takes into account the rapidly changing global political conditions of today, Oliver O’ Donovan says in his contribution, for the often simplistic political analyses of the earlier generation of liberation theologians can hardly meet the challenges of globalisation. Yet, as all the writers whose essays are included in this volume insist, liberation theology still has a decisive role to play in the struggle for social justice as well as in the campaign for emancipating religion from layers of dogma and conservatism that have reduced it, for the most part, into a tool for oppression. 1.

The Vatican appears to be hostile to liberation theology for which of the following reasons? A. Liberation theologians try to provoke enemity among the different factions of Christianity. B. Liberation theologians have only poverty and injustice as their concern. C. Vaticans have a long history of colluding with the ruling parties against which theologians are uniting. D. Liberation theologians have in many occasions coloured the texts of the Bible for their own ends. E. Majority of people favoured the Vatican’s supremacy over church.

2.

Which of the following is a pragmatic application of liberation theology as depicted in the passage? A. Through ecclesiastical communities, people interpret passages in the Bible to gain inspiration to fight against oppression. B. Theology can be interpreted in various ways to suit different political agendas. C. Theological pursuits would help prevent political parties from using racism as a weapon against the under-privileged. D. Liberation theology sees Jesus as a radical crusader for social justice just as Marx is for the labour force. E. Liberation theologies displayed an undivided national will.

3.

For liberation theologians, ‘sin’ connotes A. a personal misgiving C. a political evil E. fatalism

B. a social menace D. poverty

4.

Salvation, according to liberation theologians, would involve A. freedom from worldly, material life. B. an egalitarian and just society. C. using theology as a tool for justice. D. a life free of sin. E. freeing from the binding of carnal pleasures.

5.

Which of the following can be inferred from the passage? A. Pluralism is a social situation of the East and Middle East. B. Liberation theology is an offshoot of Christianity, which caters to only Christian Faith. C. Liberation Theologies provide inspiration for those who seek justice against oppression of any kind. D. Patriarchal traditions are observed by all religious faiths. E. The author is apprehensive of the unconventional principles of theologies.

120

Career Avenues

PASSAGE-2 In trying to understand the mystery of life, men and women have followed many different approaches. Among them, there are the ways of the scientist and mystic, but there are many more; the ways of poets, children, clowns, shamans, to name but a few. These ways have resulted in different descriptions of the world, both verbal and nonverbal, which emphasize different aspects. All are valid and useful in the context in which they arose. All of them, however, are only descriptions, or representations, of reality and are therefore limited. None can give a complete picture of the world. The mechanistic world view of classical physics is useful for the description of the kind of physical phenomena we encounter in our everyday life and thus appropriate for dealing with our daily environment, and it has also proved extremely successful as a basis for technology. It is inadequate, however for the description of physical phenomena in the submicroscopic realm. Opposed to the mechanistic conception of the world is the view of the mystics which may be epitomized by the word ‘organic’, as it regards all phenomena in the universe as integral parts of an inseparable harmonious whole. This world view emerges in the mystical traditions from meditative states of consciousness. In their description of the world, the mystics use concepts which are derived from these non-ordinary experiences and are, in general, inappropriate for a scientific description of macroscopic phenomena. The organic world view is not advantageous for constructing machines, nor for coping with the technical problems in an overpopulated world. In everyday life, then, both the mechanistic and the organic views of the universe are valid and useful; the one for science and technology, the other for a balanced and fulfilled spiritual life. Beyond the dimensions of our everyday environment, however, the mechanistic concepts lose their validity and have to be replaced by organic concepts which are very similar to those used by the mystics. This is the essential experience of modern physics which has been the subject of our discussion. Physics in the twentieth century has shown that the concepts of the organic worldview, although of little value for science and technology on the human scale, become extremely useful at the atomic and subatomic level. The organic view, therefore, seems to be more fundamental than the mechanistic. Classical physics, which is based on the latter, can be derived from quantum theory, which implies the former, whereas the reverse is not possible. This seems to give a first indication why we might accept the worldviews of modern physics and Eastern mysticism to be similar. Both emerge when one enquires into the essential nature of things – into the deeper realms of matter in physics; into the deeper realms of consciousness in mysticism – when one discovers a different reality behind the superficial mechanistic appearance of everyday life. The parallels between the views of physicists and mystics become even more plausible when we recall the other similarities which exist in spite of their different approaches. To begin with, their method is thoroughly empirical. Physicists derive their knowledge from experiments; mystics from meditative insights. 6.

According to the author, representations of reality A. are not all – encompassing C. are illusions E. are ephemeral and evanescent

B. are unlimited D. are intuitive understandings of nature

7.

According to the author: A. Non-verbal descriptions of the world are invalid. B. Approaches to understanding life are not uniform. C. All realms of physical phenomena can be described through mechanistic world view. D. Mechanistic and mystic views are parallel, complementary interpretations. E. Life cannot be understood in all its various manifestations.

8.

Which of the following best explains the ‘organic’ view of the world? A. It fills the void between mechanistic world view and mystical view of the world. B. It considers diverse phenomena of the world as part of a congruent unified whole. C. The organic world view emerges from without, that is from the ecological component of the world. D. The organic view emerges from the day to day observations and experiences of man in general. E. Life as it is seen in its physical existence is in a transitional stage.

121

Career Avenues

9.

Based on the information in the passage, if there is widespread famine in a subcontinent, which of the following philosophical traditions can offer solutions to overcome the situation? A. The organic world view, as it offers solutions to solve the economic and ecological problems faced in our day to day lives. B. The mystical world view, as problems faced by mankind are a result of the disharmony between nature and man as a consequence of his continuous exploitation of nature. C. Physical theories derived from intuitive experiences as direct experiences, many a time, fail to connect diverse phenomena together which would offer solutions to man’s problems. D. The mechanistic world view, as it is advantageous in dealing with our environment, and in using science and technology to create wealth. E. Eco-social theories that offer solutions through understanding of existence and survival.

10.

Which of the following can be inferred from the passage, as the difference between classical physics and modern physics? A. Modern physics makes use of organic world view to construe phenomena at the subatomic level where as classical physics doesn’t. B. Modern physics cannot help us understand phenomena beyond the dimensions of everyday life. C. Modern physics is an offshoot of classical physics, and classical physics is more fundamental. D. Modern physics can be derived from classical physics which in other words is quantum theory. E. Modern physics derives the existence of intellect whereas classical physics does not.

11.

Which of the following can be inferred from the passage? A. Classical physics cannot be derived from quantum theory. B. Modern physics incorporates the organic view from which mechanistic world view is derived. C. Both mechanistic and mystic worldview can be derived from quantum theory, but quantum theory cannot be derived from classical physics. D. The worldviews of classical physics and Eastern mysticism are parallel in many ways. E. The author does not subscribe to either modern or classical physics.

12.

It can be inferred from the passage that A. Classical physics and Eastern mysticism are empirical pursuits, therefore parallel. B. Modern physics derives knowledge from meditative reflections and deny the significance of experimentation in its methodology. C. The methodology of mysticism and modern physics are similar, where as their contributions are different. D. Modern physics and mysticism follow different approaches, the former adopts experimentation and the latter intuitive knowledge. E. The world cannot be understood as a united whole without understanding each of its components. T

122

Career Avenues

TEST 17 DIRECTIONS: After each passage you will find a series of questions. Select the best answer choice for each of the questions. Answer the questions based on what is stated or implied in the passage. PASSAGE-1 There are about six million red blood cells in every milliliter of an adult’s blood. The body produces red blood cells at an average rate of 9,000 millions per hour. Every day, 200 billion red blood cells die. Though the above facts look incredible they are proven facts about our blood. Blood may be defined as the fluid circulating in the arteries, veins and capillaries of vertebrate animals. Blood, the red fluid, is pumped around the body by the heart. Blood is made up of cells and a liquid. The liquid portion of the blood is known as plasma in which the cells are suspended. There are three types of cells in the blood: Erythrocytes (red blood cells), which contain hemoglobin, the oxygen carrying red pigments; leukocytes (white blood cells), which are responsible for the body’s defense mechanism; and platelets, the small particles involved in blood clotting. Erythrocytes, the oxygen-carrying cells of the blood have the dominating presence. In fact, 99 percent of the cells are erythrocytes. The cells have high surface-to-volume ratio. This enables them to rapidly diffuse oxygen and carbon dioxide to and from the interior of the cells. The erythrocytes are produced in the soft interior of bones called bone marrow. Erythrocytes are immature cells. These cells lack nucleus and organelles. They cannot reproduce themselves and their life span is also short. The average life span of an erythrocyte is approximately 120 days. The total volume of circulating erythrocytes in the human body for this reason. But unfortunately, many misconceptions come in the way of voluntary donation of blood. A healthy individual can safely donate blood once in three months without an remains fairly constant. The chief characteristic of erythrocytes is the presence of a protein, hemoglobin, which binds oxygen taken in by the lungs. In a hemoglobin molecule, there are four sub-units. Each of the four sub-units consists of an organic molecule known as heme attached to a polypeptide. The four identical polypeptides join together to form an entity known as globins. Each heme portion contains an iron atom (Fe). It is the iron atom that binds oxygen. Leukocytes are classified according to their structure and affinity for the various dyes. There are five types of leukocytes viz. neutrophils, eosinophils, basophils, monocytes and lymphocytes. The first three types are collectively known as polymorphonuclear granulocytes. Neutrophils have no preference for any particular type of dye. Eosinophils derive their name from the fact that they take up the red dye, eosine. Basophils have an affinity for basic dyes. Granulocytes are the most numerous among the white blood cells. They have a multilobed nucleus and they contain a large number of cytoplasmic granules. Neutrophils are capable of engulfing bacteria and other microorganisms. The cytoplasmic granules of the neutrophils are packs of potent enzymes capable of digesting many types of cellular materials. Like neutrophils, eosinophils are also highly mobile and capable of engulfing and destroying microorganisms. Monocytes are the largest cells of the blood. They have a relatively large nucleus. The cytoplasm of a monocyte contains a large number of fine granules. They are capable of degrading infectious agents as well as red blood cells. Lymphocytes are part of the immune response to foreign substances in the body. Lymphocytes protect the body from foreign microbes. Most lymphocytes are small and contain scanty cytoplasm. The nucleus of a lymphocyte occupies most of the cells. Some are larger and have more cytoplasm material. Plasma is the liquid portion of the blood. It is made of a large number of organic and inorganic substances like proteins, hormones, nutrients, lipids, minerals such as iron and metabolic wastes dissolved in water. Platelets are colourless cell fragments that contain numerous granules. Platelets, which are much smaller than erythrocytes, are produced when the cytoplasm of large bone marrow cells (megakarcytes) become pinched off and

123

Career Avenues

enter the circulation. It may be noted that platelets are not true cells, but membrane bound cell fragments. There are normally 15,000 to 400,000 platelets per cubic milliliter of blood. Absence of platelets causes severe bleeding. Blood is classified into certain groups on the basis of the presence or absence of certain proteins (antibody) on the surface of the red blood cells. Two main antigens are designated A & B. The major grouping is the ABO system, which was the first human blood system to be discovered in 1901 by Austrian born American scientist Karl Landsteiner (1868-1943). This system gives rise to four blood groups - having only A (A), having only B (B), having both (AB), and having neither (O). Each of these groups may or may not contain the rhesus (Rh) factor, a blood protein present in the red blood cells in a majority of the population. The presence (Rh positive) or absence (Rh negative) of Rh factor forms the basis for the Rh blood group system. There are other minor blood group systems. Correct identification of blood groups is a must in transfusion. Incompatible blood groups of donor and recipient of blood will result in blood clotting with possible death of the recipient. Blood clotting is the process by which excessive bleeding is prevented after injury. It involves a number of chemical reactions between certain soluble proteins or clotting factors in the blood leading to the formation of fibrous protein known as fibrin. Fibrin is a thread-like protein that weaves a network that entraps erythrocytes, platelets and other materials to create blood clotting. Blood clotting is a very complex process that requires a number of protein and other factors including vitamin K present in the plasma. The presence of platelets is essential for the process of blood clotting to start. On many occasions (accident, surgery, critical conditions), the requirement of this life-saving fluid is high. The noble procedure of voluntary blood donation has been systematised just y adverse effects. The only precaution that needs to be taken is to ensure the use of sterilised needles. 1.

2.

3.

4.

5.

Which of the following is true in the context of the passage? I. Red blood cells control the body’s defense mechanism. II. Erythrocytes are dominant. III. White blood cells do not contain hemoglobin. A. I and II B. II only D. I and III E. III only Which of the following is a feature of erythrocytes? A. It contain a nucleus. C. It constitute only 50% of the cells. E. It doesn’t contain plasma. Which of the following describes neutrophils? A. They are prone to micro-organisms. C. They are not mobile. E. They cause bloodclotting.

C. II and III

B. It does not have organelles. D. It does not circulate.

B. They can engulf bacteria. D. They have preference to certain dyes.

Which of the following is/are true of lymphocytes? I. They can defend the body. II. They contain excessive cytoplasm. III. They are prone to bacterial action. A. I only B. II only C. I and II D. I and III E. III only

Blood is classified based on which of the following? A. On the presence of platelets in the cells. B. On the number of red blood cells. C. On the presence or absence of antibodies on the surface of erythrocytes. D. On its degree of clotting. E. Only on the Rh factor. PASSAGE-2 Words are little symbolic units of meaning and can be as contradictory as sub-atomic particles. This may well be nature’s quixotic way of laughing at our desperate need to explain everything. It gives us a full stop, but watches helplessly, as we expand it into three dots and continue to search.

124

Career Avenues

Although the measurement of the velocity of sub-atomic particles precludes the measurement of their position and vice versa, it hasn’t stopped nuclear physicists from trying, from searching, from attempting to pin down, to explain. And it is important. In a book on quantum physics called In search of Schrodinger’s Cat John Gribbin says something very fascinating. If a mythical god with a magical pair of infinitesimally small pliers started the task of removing one atom from a molecule of hydrogen (if I remember correctly) every second from the time of the Big Bang ...today, it would take another million years for him or her to complete the task. But it is still important to try. Why? If everything is so small and the now proved quantum world is essentially indefinable, why do we go on trying to define it? Because, we must. It is as important to be rigorous and empirical as to accept the indefinable. Lest we forget, it is through absorption in the act of definition that we first encountered the indefinable. And it is still found there more easily than anywhere else. But for the effort to define, how would we find the indefinable? But for the setting of limits, would the notion of the limitless have ever arisen? Didn’t William Blake once remind us that we never know what is enough; unless we have known what is more than enough? So, when we analyze words, they are paradoxical, as anything selfreferential is. Whether it is the language of mathematics or the language of words, self-reference engenders paradox. But one wonders why this is a cause for concern for some people, who would prefer no shades of gray. Paradox is delightful. It is a rich and fertile ground that nourishes and nurtures what we want to communicate, which often has a nasty habit of falling in between any two given words available to describe it. Words are very close to what Planck called “quanta” though they are not literally packets of meaning; they are the paradoxical verbal equivalent of “receptacles of meaning”. Little drawers if you like, in which we can insert fresh meanings that expand, limit or even contradict the accepted meaning of the word or phrase. When we say “I’ll believe you” for instance, we mean the exact opposite. As, indeed, when we say something is “bad” in Black American language, because it means “good”. The original meaning of the word is like a reference point on a matrix. Good, if we use its definition as a working hypothesis. But very dangerous, if we take it as a full and final, irrevocable statement of what it sets out to describe. Why, one may ask, give the word a meaning at all, if accepting it is suspect? And why embark on the act of definition at all if the result of the definition is insignificant? Like many wonderful and rewarding things in this mysterious world, it is not either / or but and / plus. It is like asking why we learnt to crawl, if all we are going to do is unlearning it to walk? And further, when on occasion, we are required to crawl in later life, are we regressing? Learning is a process, not a thing. If we must look at it as a thing we must look at it as lying-sitting-standingcrawling-walking-running. To define words, and define them exactly, is very important at the outset. When one is learning a language and even through the process of getting familiar with it, definitions and boundaries are crucial, just as following a broad road to a place is critical before we know our way there. Once we do the rules aren’t important; once we have found a dozen shorter or pleasanter ways to the place, the highway may be of little use to us. Like a protective cage around a little sapling, definitions protect us in our fledgling days, from the predators of license and ambiguity. And they are important. In its place, everything is important. Once we have a certain command of the language, however, rules are meant to be broken; particularly, if we are riding the crazy roller coaster of the English language. It is then what we thought was a packet turns out to be a receptacle. In the clearer light of day, when there is less confusion and obscurity, what appeared to be a serpent in the dusty light, is now quite clearly rope. In Alice in Wonderland, Humpty Dumpty says it quite brilliantly. “I don’t know what you mean by `glory’,” Alice said. Humpty Dumpty smiled contemptuously.

125

Career Avenues

“Of course you don’t – till I tell you. I meant there is a nice knock-down argument for you!” “But `glory’ doesn’t mean a nice knock-down argument,” Alice objected. “When I use a word,” Humpty Dumpty said, in a rather sorrowful tone, ‘it means just what I choose it to mean – neither more nor less’.” The question is,’ said Alice “whether you can make words mean so many different things.’’ The question is,” said Humpty Dumpty, “which is to be the master – that’s all.” Be that as it may, words in your ear before the words stop. 6.

Which of the following is true in the context of the passage? A. Anything delightful is paradoxical. B. Anything self-referential is paradoxical. C. Anything in shades of gray is paradoxical. D. Anything in a rich and fertile ground is paradoxical. E. Anything paradoxical is self-referential.

7.

Which of the following is most plausibly inferred from the passage? A. To understand a system, we should investigate within its boundaries. B. To understand a system, we should investigate beyond its boundaries. C. To understand a system, we should be rigorous and empirical. D. To understand a system, we should investigate both within and beyond its boundaries. E. A system has no relation with its external world.

8.

Which of the following can be understood from Humpty Dumpty’s statements? A. Alice does not know what ‘glory’ means. B. He is Alice’s master. C. He imparts to a word the meaning he intends. D. His words are ambiguous. E. It is difficult to impart meaning through words.

9.

Which of the following is implied in the passage? A. When some people say ‘bad’, they mean the opposite. B. God will complete the removal of all atoms from the hydrogen molecule in a million years. C. One can simultaneously measure both the velocity and position of sub-atomic particles. D. Planck called words, ‘quanta’. E. Self contradiction in meanings is central to all languages.

10.

According to the passage, paradox is useful because A. word meanings can be misleading and confusing. B. it nourishes what we intend to communicate. C. it falls in between any two given words used to describe it. D. essentially words are indefinable and self-referential. E. It doesn’t reveal the actual meaning.

11.

By referring to words as ‘receptacles of meaning’, the author implies that A. words have clear definite meanings conforming to universal grammar. B. words are paradoxical and at the same time indefinable. C. we can give new meanings to words that restrict or expand the established meanings. D. words entail a certain rigidity that blocks any unwanted change in their semantics. E. established meanings cannot be changed.

12.

The author likens the original meaning of a word to A. a hypothesis. B. a reference point on a matrix. C. a matrix. D. an irrevocable statement. E. a theory. T

126

Career Avenues

TEST 18 DIRECTIONS: After each passage you will find a series of questions. Select the best answer choice for each of the questions. Answer the questions based on what is stated or implied in the passage. PASSAGE-1 Vedanta philosophy was one of several thought currents from abroad that reached New England in the early decades of the 19th century and contributed to the thinking of Emerson and Thoreau. Emerson’s interest in the sacred writings of the East probably began during his Harvard days and continued throughout his life. He knew Laws of Manu, Vishnupurana, the Bhagvad-Gita, and Katha Upanished: There are numerous references to these scriptures in his Journals and Essays. Thoreau too, was introduced to Oriental writing while still at Harvard. His initial contact was with an essay in Oriental poetry by Sir William Jones. In 1841, at the age of 24, he began an intensive study of Hindu religious books. In the January 1843 issue of The Dial, Thoreau published selected passages from Laws of Manu. From a French version of the Sanskrit Harivansa, he translated a story, “The Transmigration of Seven Brahmans,” and in The Dial of January 1844, he published excerpts from Buddhist scriptures under the title “The Preaching of Buddha.” Emerson, Thoreau, and other transcendentalists interested in the concept of “selfhood” found in Hindu scripture a well-elaborated doctrine of Self. Hindu scripture tells us that the central core of one’s self (antaratman) is identifiable with the cosmic whole (Brahman). The Upanishads state: “The self within you, the resplendent, immortal person is the internal self of all things and is the Universal Brahman.” Concepts similar to this cardinal doctrine of Vedanta appear in the writings of the Transcendentalists. But there are many ideological similarities among oriental literature, the neoplatonic doctrines, Christian mysticism, and the philosophy of the German Idealists such as Kant and Schelling. And, since the Transcendentalists were acquainted with all of these writings, it is not always possible to identify specific influences. Nevertheless, the striking parallels between Transcendentalist writing and Oriental thought make it clear that there was a spiritual kinship. In “Plato, or, the Philosopher,” Emerson writes that “the conception of fundamental Unity” – the “ecstasy” of losing “all being in one Being” – find its highest expression “chiefly in the Indian Scriptures, in the Vedas, the Bhagavad Gita, and the Vishnu Purana.” In this essay, Emerson quotes Krishna speaking to a sage : ‘You are fit to apprehend that you are not distinct from me ……That which I am, you are, and that also is this world, with its gods and heroes and mankind. Men contemplate distinctions because they are stupefied with ignorance,’ ‘…… What is the great end of all, you shall now learn from me. It is soul,- one in all bodies, pervading, uniform, perfect, preeminent over nature, exempt from birth, growth and decay, omnipresent, made up of true knowledge, independent, unconnected with unrealities, with name, species and the rest, in time past, present and to come. The knowledge that this spirit, which is essentially one, is in one’s own and in all other bodies, is the wisdom of one who knows the unity of things.’” In formulating his own concept of the Over-Soul, Emerson might well be quoting Krishna once again: ‘We live in succession, in division, in parts, in particles. Meantime within man is the soul of the whole; the wise silence; the universal beauty, to which every part and particle is equally related; the eternal ONE. And this deep power in which we exist and whose beatitude is all accessible to us, is not only self-sufficing and perfect in every hour, but in the act of seeing and the thing seen, the seer and the spectacle, the subject and the object, are one. We see the world piece by piece, as the sun, the moon, the animal, the tree; but the whole, of which these are shining parts, is the soul. Only by the vision of that wisdom can the horoscope of the ages be read …” In some respects, Henry David Thoreau was even more than Emerson attracted to Oriental thought and philosophy. For while Emerson found the Hindu doctrines of soul congenial to his own ideas about man’s relationship to the universe, Thoreau found in Hindu Scriptures a way of life with which he felt a profound affinity. When Thoreau began his intensive study of Hindu Scriptures, he wrote in his Journal. “I cannot read a sentence in the book of the Hindus without being elevated upon the table-land of the Ghauts … The impression which those sublime sentences made on me last night has awakened me before any cock crowing … The simple life herein described confers on us a degree of freedom even in perusal … wants so easily and gracefully satisfied that they seem like a more refined pleasure and replete ness.” Later, in his first book he said: “Any moral philosophy is exceedingly rare. This of Manu addresses our privacy more than most. It is a more private and familiar and at the same time a more public and universal work, than is spoken in parlour or pulpit nowadays. As our domestic fowls

127

Career Avenues

are said to have their original in the wild pheasant of India, so our domestic thoughts have their prototypes in the thoughts of her philosophers … Most books belong to the house and street only, and in the fields their leaves feel very thin … But this, as it proceeds from, so it addresses, what is deepest and most abiding in man. It belongs to the noontide of the day, the midsummer of the year, and after the snows have melted, and the waters evaporated in the spring, still its truth speaks freshly to our experience...” 1.

It can be inferred from the passage that the transcendentalist movement referred to in the passage revolves around A. Vishnupurana B. the doctrine of self C. the laws of Manu D. Oriental writing E. Occidental thoughts

2.

The difficulty in identifying particular influences on transcendentalism arises from A. its eclectic nature of acquainting with all philosophical doctrines. B. its being equidistant from both oriental and western schools of religion. C. the apparent paradoxes in Christian mysticism and Buddhist ideology. D. its over-dependence on Bhagavad Gita and Vishnupurana. E. its proximity to other ancient civilizations.

3.

Which of the following best explains Vedanta? A. The sacred writings of the East. B. The self is identifiable with the universal whole. C. The spiritual side of human life. D. The process of being and becoming. E. The myths and legends of the far East.

4.

The doctrines of ‘Brahman’ and ‘Antaratman’ are synonymous with A. God and Hindu B. Mystic and ideal C. Self and universal soul D. Sanskrit and Hindu E. Soul and Body

5.

The lines “you are fit to apprehend that....is the wisdom of one who knows the unity of things” can be summarized as A. there is no birth to a soul and it is the ignorant man who comes to life again and again. B. true knowledge is independent, uniform and that which dominates over nature. C. past, present and future are non-existent and are merely constructs of the mind. D. a wise man knows that the soul is immortal and is present in everyone and everything and that it merges with the cosmic whole. E. a man who knows the present need not know the future.

6.

‘Oversoul’ referred to in the passage is A. Krishna’s perception of the ‘self’. C. coinedby transcendentalists to mean the ‘cosmos’. E. paradoxical with soul.

B. Emerson’s concept of the eternal soul. D. synonymous with the wise man.

7.

“Only by the vision of that wisdom can the horoscope of the ages be read...” The statement implies that A. one is able to understand the essence of life only when one sees the soul as part of the whole. B. the future of mankind can be predicted by understanding the origin of the universe. C. we live in parts and in particles and only the wise go into eternity. D. it is possible to understand the past eras by visualising the future of mankind. E. the origin of universe is not understood through worldly wisdom.

8.

Which of the following features of the Hindu scriptures appealed more to Thoreau? A. The doctrine of soul. B. The concept of universal religion. C. The philosophy of life depicted in them. D. The degree of freedom. E. The belief in surrealism.

128

Career Avenues

TEST 19 DIRECTIONS: After each passage you will find a series of questions. Select the best answer choice for each of the questions. Answer the questions based on what is stated or implied in the passage. PASSAGE-1 Great literature is all about life and a great writer writes about the world as if all the people in it were his relations. With Tolstoy, this was very nearly the case and, Anna Karenina is the perfect example. Tolstoy had the capacity to identify with all kinds of people, more so because he had relatives in all walks of life – an unfair advantage over most modern novelists who know so little of their scattered families. Thus, they are unable to portray the life of a single class, let alone a whole society, and are restricted to a single set – the jet set, the university set, the media set, and so on. The disintegration of great families has impoverished society in countless ways, even in its literature. Tolstoy’s portrayal of the world and of life – birth, friendship, love, marriage, separation, community, solitude, betrayal, pity, disillusionment and finally the void – his portrayal of the inner life of women, so complete in the large and in the small is second to none in the history of modern literature. Like the great 19th Century Russian novel, Anna Karenina is not intellectual, but emotional. (In the classical Russian novel, people committed to insane asylums are not those who have lost their reason, but those who have suddenly acquired it). But, all the same, Tolstoy’s approach is analytical. He is concerned far less with what his characters do – the why is all-important and to this extent, Anna is one of the great psychological novels of the 19th Century. Also, because Anna is so massive in its dimensions and because it exercises an immediate control over our emotions, the sophistication of individual detail tends to escape us. Hence the justifiable belief that the great Russian novelists can be grasped in their generality and little is to be gained from the close academic study that we apply to say, Conrad or Proust. Sad as this might be (because Tolstoy’s pure narrative power based on experience, observation and sheer imagination is unequalled) it is the idea(s) and the question(s) that the novel raises that really matter(s). It is the famous opening line of Anna that sets the primary theme of the novel: “All happy families are alike, but an unhappy family is unhappy in its own fashion”. Love, marriage and family provide the links to the basic theme: the problem of marriage in a modern society. When Tolstoy was writing Anna, he had provisionally entitled it as Two Marriages or Two Couples. And this is precisely how the theme is uncovered – not through one story but two which, as the novel progresses, go quite separate ways. The novel’s heroine, Anna, appears in only one of the stories while the novel’s hero, Levin, appears in the other; they meet once, in a scene of seemingly no great consequence. The personal separateness of Anna and Levin marks a fundamental difference between the social worlds they each inhabit. Anna lives in the top circle of Petersburg society, where her husband’s official position in the imperial administration, together with her beauty, gave her access into the most influential sets: senior officials and intellectual men known as the “conscience of Petersburg”, and that of society proper – the world of balls, dinner parties and all the jazz. Levin, by contrast, lives in the country and is best at home with farming and the peasants: he does not go to the capital and when he does, the whole place is repugnant to him. With Tolstoy there is a deliberate avoidance of a formal neatness because life was not a neat little game, at least not while it is being played. Besides the two families, whose affairs link the story of Anna with that of Levin, there are the Oblonskys and the Sheherbatskys and through them Levin’s bride, Kitty. Initially, Kitty finds herself in the role of the defeated, jealous rival to Anna, while Anna’s lover, Vronsky, seems to stand in the way of Levin. This rivalry does lead to no external conflicts nor to any significant contact; it is primarily felt by Kitty and Levin, confined to their minds and feeling, which have to be overcome inwardly, through a quiet process of moral growth which makes them discover themselves and their true love for each other. Anna is a novel of the mind first and last, and its greatness lies precisely in Tolstoy’s capacity to get under the skin of each of his characters. “Under the skin” is a suggestive phrase and tells us how they feel. It is the physicality of the experience that is strongly conveyed (time and again) and it surrounds and somehow humanises the mind. Tolstoy understood better the real effect of conversation under the stress of great emotion; so nothing sounds more life-like than Vronsky’s efforts to figure out the twists and turns of Anna’s speech.

129

Career Avenues

What about Karenin, the cad that he was, how did he take it when Anna walks out on him? Karenin was face to face with life – with the possibility of his wife loving someone else – and this seemed to be very irrational and incomprehensible because this was life itself. All his life he had lived and worked in official sphere, having to do with mere reflections of life. And every time he had come up against life itself, he had stepped aside. Now he experienced a sensation such as a man might feel who, quietly crossing a bridge over a chasm, suddenly discovers that the bridge is broken and the abyss yawns below. The abyss was real life; the bridge of artificial existence that Karenin had been leading. Apart from the emotional catharsis of his characters – Tolstoy’s imagination is always alive at the emotional components of a situation which could be like a witch’s brew of jealousy, suspicion, resentment, desperation, pride, and much else besides he is best at describing scenes of family life and what “happiness” might mean. Levin is exasperated with Kitty’s preoccupation with trivial matters: “Like all men, he had unconsciously pictured married life as the happy enjoyment of love which nothing should be allowed to hinder and from which no petty cares should detract. He should, he thought, do his work and then rest on his labours in the happiness of love. His wife was to be loved and nothing more. But, like all men, he forgot that she too needed occupation”. Anna’s relationship with Vronsky sours rapidly – he was soon for her no more than the womanizer we had glimpsed – and she is driven to suicide because she is unable to imagine “any new feeling” and she did not want to be treated ”kindly and gently out of a sense of duty”. The question that is often asked is whether the greatest society novel in all literature is “an anti-society novel”. The Bible text “Vengeance is mine, I will repay”, stands as the epigram to the novel. The moral momentum of the novel was certainly to lash society for the cold, cruel rebuff inflicted on a woman who goes astray through passion but is fundamentally proud and high-minded, instead of leaving to God the punishment for her sins. After all, what has Anna done? By social standards, she is guilty of nothing more than a refusal to compromise and conform. But this raises the question as Thomas Mann did in a seminal essay on Anna Karenina. How far custom and morality are distinguishable? how far are they – in effect – one and the same, how far do they coincide in the heart of a socially circumscribed human being? This question hovers unanswered over the whole novel. But, any great work is not compelled to answer questions. Its task is to bring them out, coldly, cruelly in whatever it thinks and feels best. If a work performs this task – and Anna Karenina does it in full and ample measure – Tolstoy’s love for his Anna leaves us in no doubt at al, no matter how much suffering he painfully and relentlessly visits on her. Anna is, all said and done, one of the most movingly beautiful characters who ever stepped off the printed page to live in our imagination. 1.

The author perceives Tolstoy’s Anna Karenina as an analytical novel because A. it is concerned, to a greater extent, with the actions of characters. B. it reflects the social milieu to which its main characters belonged. C. it deals with the basic theme of problems of marriage in a modern society. D. it is concerned with the reasons for the actions of the characters and exercises an immediate control over our emotions. E. the author doesn’t believe in training emotions.

2.

The author eulogises Tolstoy’s capacity to get ‘under the skin’ of his characters. This suggests that A. Tolstoy was portraying each character assuming himself as the protagonist of the novel. B. Tolstoy was humanising the mind through the physicality of experiences of his characters. C. Tolstoy was portraying the social worlds to which the central characters Anna and Levin belonged. D. Tolstoy was attempting to perceive the great divide that existed between urban and rural lives. E. Human emotions are mystical and inexplicable.

130

Career Avenues

3.

In the author’s analysis, Tolstoy has not answered which of the following questions? A. Is it possible for a novelist to identify with all kinds of people in the society? B. Can the inner lives of women be portrayed completely and successfully? C. How far is conversation effective under the stress of great emotions? D. How far are custom and morality distinguishable and to what extent are they one and the same? E. What are the limitations of ornamental style of writing novels?

4.

In the author’s opinion the task of a literary work is to A. bring issues out realistically. C. portray society and its morality disinterestedly. E. indoctrinate and incite.

B. answer questions conclusively. D. quench the intellectual thirst of the readers.

5.

The analyst predicts that modern novelists would have been able to create Anna Kareninas A. if they followed the analytical school of fiction writing in which characters eclipse their actions. B. if they were able to identify with all kinds of people and portray a class or a whole society. C. if they catered less to emotional lives and more to intellectual spheres. D. if they did a thorough review of the literature of the 19th century and other classic writings. E. if there was no class affinities among literary men.

6.

In the author’s view, one of the phenomena that contributed to the weakening of literature is A. the narrative technique employed by most writers. B. an escapism from real life. C. the disintegration of families. D. the overemphasis on style. E. thelack of common rules for literary composition.

7.

The author cites Conrad and Proust in order to A. point out the similarities between their works and in Tolstoy’s. B. show that unlike Conrad’s or Proust’s, Tolstoy’s novel has little to be gained from close academic study. C. point out that Tolstoy’s use of experience-based narrative technique is alien to most. D. argue that the imagination of Tolstoy is unequalled and unimitated till date. E. criticize the vainness of academic pursuits.

8.

Which of the following is not true of Tolstoy’s Anna Karenina? A. The theme of the novel has two stories that develop separately. B. The fundamental difference between the social classes is portrayed through the personal separateness of Anna and Levin. C. There are descriptive portrayals of family life and of the meanings of happiness. D. Tolstoy employs a deliberate use of formal neatness of life. E. It doesn’t reflect human righteousness.

131

Career Avenues

TEST 20 PASSAGE-1 Twenty years after his death, the extremely well known, but much less read, existentialist philosopher Jean Paul Sartre is back in the news. It is a time for a revaluation. Magazines are rushing to bring out special issues, writers are publishing their analytical studies, each trying to present his or her own Sartre. There is even a brand new voluminous biography. Called the greatest French mind of the millennium, the excesses of Sartre, his break up with communism and his sympathy for the Maoists, his so-called philosophical and political errors of judgment, the friction with Albert Camus (decidedly a much more widely read author even today), are all forgotten. The much-quoted remark of Jean Daniel, “Is it better to be wrong with Sartre than right with Camus?” indicates the enormous charisma of this literary and philosophical figure. He may have been, compared to Raymond Aron or Albert Camus, wrong on most counts, but one who, nevertheless, succeeded in carrying public opinion with him. Today, there seems to be no apparent reason to pull Sartre out of his purgatory and posthumous disgrace and whitewash all his past mistakes, except that he is being increasingly recognised as the greatest French thinker of the 20th Century. Till recently, his plays were considered outdated and his literature didactic, but the French intelligentsia seems convinced now that despite the uncertainty and contradictory nature of his political commitments, there is something more to this dethroned king of French philosophy, his work needs a second look. Bernard-Henri Levy, founding father of the “new philosophers” movement, leads the way. A well known figure of the French intellectual world, controversial and omnipresent on television, BHL, as he is commonly known, devotes a mammoth study to the three avatars of Sartre – Sartre the theoretician of individualism, Sartre the committed intellectual and Sartre discovering Judaism. In fact, one of the better-known texts of Sartre in India, What Is Literature? has always been defamed and slandered by critics as the beginning of indoctrination. BHL strongly defends it as a book on the freedom of writing. He reiterates what Sartre always maintained – that writers must write about their times and for their times and not for posterity. Literature is like a banana. It has to be consumed straight away and too bad for those who cherish the idea of immortality for their texts. Since the mid-1970s, when BHL came centre stage with a bang, he has never been a supporter or defender of Sartre, whom he had always assumed to be an old humanist intellectual trapped in an antiquated philosophy. Brought up on Foucault, Althusser and Lacan, who represented modernity to his generation, BHL readily admits that Sartre’s philosophical work has been a fairly recent revelation for him. In fact, nine years ago, when BHL brought out The Adventures Of Freedom, a work that studies the writings and novels of the great intellectuals of the 20th Century, there was one striking omission-that of Jean-Paul Sartre. Today, BHL has more than made up for this absence by devoting ample space and time to Sartre. It is true that Sartre – through the written word, through articles and through books, on stage, in his speeches and as a militant – got involved with the major events and debates of his time. He commented and wrote extensively. His work is enormous considering that from ages eight to 68, he wrote every single day. Nothing took him away from his strict schedule, except his failing eyesight later in life. Thousands of people joined the funeral procession of Jean-Paul Sartre to the Montparnasse cemetery. The burial ceremony was a major event, a festive and joyous occasion for “Sartre’s people” as they were called. In almost all articles and studies that are being discussed and dissected, what comes to the fore is Sartre the philosopher. Sartre the novelist, Sartre the literary figure is relegated to the background. If at all his literary work is mentioned, it is limited to Nausea and Words, the latter being “the favourite book of those who do not like Sartre”. According to Bernard-Henri Levy, people do not read Sartre because he is too famous. “This is the curse of fame the misfortune of writers who become more famous than their works. Even during his lifetime, Sartre suffered this fate. Remember, The Family Idiot: Gustave Flaubert 1821-1857? Not one reader or almost none. Or the story of one particular edition of Being And Nothingness, a whole section was missing and no one realised it”.

132

Career Avenues

He concludes by saying: “Sartre was the man of the century. A man towards whom all forces, all intensities of an epoch converge; a secret axis, a magnet, not someone who makes a century, but who magnetises it …” 1.

It can be inferred from the passage that, Sartre’s ‘What is Literature’ is condemned for A. its overemphasis on existentialistic notions. B. its doctrinaire elements. C. its outdated and old humanistic views. D. its sympathetic lenience toward Maoism. E. its mythical elements.

2.

Which of the following would Sartre be most likely to agree with, as inferred from the passage? A. A writer must author books for future generations. B. A writer must attain immortality through his works. C. A writer must write about his times and for his times. D. Literature is better cherished when it is consumed by generation after generation. E. A writer should write about his convictions.

3.

When Jean Daniel remarked “Is it better to be wrong with Sartre than to be right with Camus”?, he most plausibly meant that A. Camus was much more widely read than was Sartre. B. Sartre was known for excesses – his allying with Maoists and breaking up with communists. C. Sartre and Camus were archrivals and their friction was quite open. D. though wrong on most counts, Sartre was charismatic and carried public opinion with him. E. it is difficult to decide the worth or lack of it in Sartre’s writing.

4.

It can be inferred from the passage that, Bernard-Henri Levy’s earlier indifference toward Sartre stems from A. Levy’s criticism of Sartre’s indoctrination in “What is Literature”. B. the fact that Levy had not recognised the philosophical worth of Sartre’s works. C. Levy’s hostility toward communism and Sartre’s allegiance toward it. D. Sartre’s friction with Camus whom Levy idolised and in whom he found a true individualist. E. his being influenced by other critics of Sartre.

5.

Levy’s defense of ‘What is Literature’ was based on the fact that A. the critics who slandered Sartre did not understand him well enough. B. the critics of ‘What is Literature’ were not part of the ‘new philosophers’ movement. C. the book is on the freedom of writing, a view Levy shared with Sartre. D. Sartre’s opponents were peeved with Sartre’s communist lineage. E. the book conforms to the standard themes of the age.

6.

Which of the following best describes ‘the misfortune’ of writers who become more famous than their works? A. People do not read the books of authors who are too famous. B. They are criticised during their lifetimes and worshipped posthumously. C. People fail to notice the philosophical worth of such books and read them superficially. D. People tend to overlook the errors of judgment in the books and consider the books as classics. E. Such books are out of the reach of the regular reader.

7.

According to the passage an overdose of Foucault, Lacan and Althusser led to Levy’s A. failing to notice the philosophical depth of Sartre, assuming he was an old fashioned humanist. B. founding the ‘new philosophers’ movement. C. accusing Camus of writing communist doctrine and influencing the young. D. later becoming a controversial, omnipresent intellectual. E. indirectly fawning a controversy surrounding the literary cult that Levy subscribes to. T

133

Career Avenues

TEST 21 DIRECTIONS: Each passage in this group is followed by questions based on its content. After reading a passage, choose the best answer to each question. Answer all questions following a passage on the basis of what is stated or implied in that passage. PASSAGE-1 Eugenics, Dean Inge writes in one of his essays, is capable of becoming the most sacred ideal of the human race, as a race; one of the supreme religious duties. In this I entirely agree with him. Once the full implications of evolutionary biology are grasped, eugenics will inevitably become part of the religion of the future, or of what every complex of sentiments may in the future take the place of organized religion. It is not merely a sane outlet for human altruism, but is of all outlets for altruism that which is most comprehensive and of longest range. However, in addition to holding out these emotional possibilities, the eugenic movement must obey practical necessities. If it is to grow into a soul compelling ideal, it must first achieve precision and efficiency as a branch of applied science. At the moment, it is idle to pretend that it had advanced very far in either direction. True that to a limited number of men and women, it is already an inspiring ideal: but for the bulk of people, if not a subject for a jest, it remains either mistrusted or wholly neglected. True that thanks to the genius of Darwin and his cousin Galton, the notion of evolutionary improvement through selection has provided a firm scientific base for eugenics, and that in recent years distinct progress had been made in applying the triumphant discoveries of modern genetics to the human species: yet for the bulk of scientists, eugenics is still hardly reckoned as a science. It may be that, as a scientist myself, I overrate the importance of the scientific side. At any rate, it is my conviction that eugenics cannot gain power as an ideal and a motive until it has improved its position as a body of knowledge and a potential instrument of control; and in this essay I shall endeavour to point out what, in my opinion is the next step towards the graduation of eugenics into the dignity of an established science. It will be an inquiry into the methodology of our subject. Eugenics falls within the province of the social sciences, not of the natural sciences. It shares with the rest of them a suspicion, often very frankly expressed by the pundits of more respectable branches of study, such as physics or pure biology, of being not quite scientifically respectable. Some, indeed, go as far as to assert that the social sciences can never be truly scientific, and imply that they have illegitimately used the word science in their title in order to exploit the prestige attaching to it in this scientific age. Personally, I do not think that this criticism is justified. All young sciences are attacked by their elders on the ground of irregularity in their canons of scientific behaviour; but they cannot expect to establish rigorous canons until they are no longer young, any more than an untried adolescent can be expected to possess the assurance and practical skill of a man in the prime of life. In addition, young sciences are not merely young like young human beings owing to the accident of the date of their birth. The date of their birth is no accident: they are young because they are more complex and more difficult. Physics is an older science than biology because in physics it is easier to isolate phenomena and to discover simple but fundamental laws. The social sciences are younger than the natural sciences because of the appalling complexity of variables which make up their subject-matter. This, however, is not all. The social sciences in certain respects differ radically from the natural sciences; they cannot expect to achieve success by applying the same simple methods as served their elder sister, but must work out new methods of their own. In the natural sciences, we isolate phenomena in order to analyze them. If possible we isolate them in the form of a controlled experiment as in physics or genetics; if this cannot be achieved, we isolate them in thought, make deductions and test out conclusions by empirical observations, as in astronomy or stratigraphical geology. By refinements of technique, we can eliminate for practical purposes all irrelevant variables: the geneticist wanting to understand some new type which has appeared in his cultures can eliminate, say, the variable environment, then the variable of single-gene mutations, then the variable of addition or subtraction of whole chromosomes, and finally pin responsibility for the phenomenon on, for example, the inversion of a particular chromosome-section.

134

Career Avenues

But the social scientist cannot do this sort of thing: he can at the best find a correlation between several variables. In terms of causation, the natural scientist can sometimes find a single definite cause for a phenomenon; the social scientist must always be content with several partial causes. He has to work out a system, based on the ideal of multiple causations. The attractive simplicity of simple and single causation is for him a false simplicity: he needs a different intellectual technique. Any one who asserts that so-and-so is the cause of a social phenomenon is bound to be wrong: it can at best be a cause. Let us, as eugenists therefore beware of making such assertions as that the celibacy of the clergy was the cause of the decadence of Spain, or that the deferential birth-rate is the cause of the increase of feeblemindedness: for by so doing we are being scientifically disreputable. And, of course, the inevitable obverse of the principle of multiple causes is the principle of multiple effect. I need not labour the point, save to stress the need for the working out of suitable methods, of partial correlation and the like, to deal with this multiple complexities. 1.

In the passage, the author is primarily concerned with A. explaining the importance of Eugenics. B. establishing Eugenics as part of an established science. C. refuting an argument that social sciences are similar to natural sciences. D. putting forth reasons the exclusion of eugenics from the realm of sciences. E. questioning the notion that a regimented mind is not suited for scientic pursuit.

2.

According to the passage, for Eugenics to become an ideal it must I. improve its position as a body of knowledge. II. achieve precision as a branch of natural science. III. come to be recognised by scientists as a true science. A. I only B. I, II D. I, II, III E. III only

3.

‘Young’ sciences are characterised by all of the following except I. They are more complex than older sciences. II. They are more difficult than older sciences. III. They are criticized for their irregular behaviour. A. I and II only B. II only D. III only E. I and III only

C. II, III

C. I, II and III

4.

The author uses the example of an experiment in gene culture in order to A. illustrate a point. B. draw attention to a startling discovery. C. suggest the importance of this science. D. show how complex it is. E. point out a problematic practice.

5.

The cause for a social phenomenon is A. abnormal B. unidentifiable D. singular E. inexplicable

C. multiple

6.

With which of the following statements is the author most likely to agree? A. Further information on the thinking in social science is unnecessary. B. Social science is an infant branch of science. C. Only natural sciences are pure sciences. D. The attitudes of a few are irrelevant to the overall development of natural science. E. social sciences are different from natural sciences in their definition of methodology.

7.

Which of the following is not a difference between Natural and Social Sciences? A. Multiple effects B. Multiple causations C. Systematic study D. Methodology E. Scientific nature

135

8.

Career Avenues

The tone of the passage can best be described as A. argumentative B. noncommittal D. enthusiastic E. moralistic

C. hortatory

PASSAGE-2 Medieval alchemists found in the end that they could not create gold. Modern geochemists have a similar problem. They find it hard to understand how natural gold deposits form. There is much hand waving about gold-rich fluids from deep in the earth and chemical precipitation, but the physics does not add up. The answer may be that what is happening is not geo-chemical at all, but biochemical. And a casual experiment by a bacteriologist may hold the key. Derek Lovley, of the University of Massachusetts, Amherst, has been studying metal-eating bacteria for two decades. These bacteria make their living by converting (or reducing) the dissolved ions of metallic elements from one electrical state to another. This reduction releases energy, which the bacteria extract for their own purposes. Unsurprisingly, such bacteria tend to prefer common metals such as iron and manganese for lunch, though some species are able to subsist on such exotica as uranium. A few months ago, though, as a bit of a lark, Dr. Lovley decided to put some of his bacteria into a solution of gold chloride. He was fully prepared for nothing to happen, as gold compounds are generally toxic to bacteria. Instead, the test tube containing the solution turned a beautiful shade of purple, the colour of metallic gold when it is dispersed very finely in water. Bacteria are already known to be involved in the formation of an iron ore called limonite, and Dr. Lovley has argued that they are also involved in the creation of certain ores of uranium. His jokey experiment, reported in the July issue of Applied and Environmental Microbiology, opens up the possibility that gold deposits, too, may have a bacterial origin, with the microbes acting as the agent that concentrates gold from sources such as volcanic springs into a form that people can mine. 9.

The author is primarily concerned with A. resolving a controversy. B. pointing out a problematic consequence. C. presenting an alternate solution. D. discussing a hypothesis. E. describing a cause effect relationship.

10.

Dr. Lovley was “fully prepared for nothing to happen” (para 3). Which of the following can be attributed to this response? A. Previous knowledge B. Incomplete evidence C. Novelty of the situation D. Findings of his own experiments E. Lack of theoretical knowledge

11.

The author states that ‘the physics did not add up’ (para 1). What does he mean by the statement? A. The phenomenon may not be physics-related. B. Physicists have not conducted the required study. C. Physicists disagree on the nature of gold available in the earth. D. The experiments should be conducted by geochemists. E. Bacteriologists have conducted casual experiments. T

136

Career Avenues

TEST 22 DIRECTIONS: Each passage in this group is followed by questions based on its content. After reading a passage, choose the best answer to each question. Answer all questions following a passage on the basis of what is stated or implied in that passage. PASSAGE-1 Scientists’ decades old quest to develop a vaccine against tooth decay has been hindered both by thorny scientific problems and by a perception that some tooth decay is inevitable though largely controlled by fluoride in water supplies and toothpaste. While fluoride has reduced the amount of tooth decay in children, experts say, many adults and poor families in the US and the developing world remain afflicted with the disease. Because few people view decay for what it is a chronic infectious disease – there has been no huge demand for a cure, said Dr. Jason M Tanzer, head of the division of oral medicine at the University of Connecticut Health Center, who also has done research on dental caries vaccines. “One of the problems is that in the eye of the public, the disease is equated with the lesions,” Tanzer said, “The lesion is a cavity – it is an effect of the disease. The disease itself is the infection. As long as dentists only talk about a cavity, it doesn’t seem all that scary”. At Forsyth, scientists have been developing cures for dental diseases for about 50 years. Today, they are taking dead aim at mutans streptococci, the bacteria that are the primary cause of dental caries. As these bacteria digest sugars, they produce lactic acid, which eats into the enamel of teeth. The targets of the Forsyth vaccine are a group of enzymes called glycosyltransferases, which are produced by the decay-causing bacteria and help them accumulate on the surface of teeth. Animal studies have shown that blocking these enzymes can nearly eliminate dental caries. Taubman and Smith want to boost production of antibodies contained in saliva known to blunt the effects of the enzyme. So they synthesized two segments of the enzyme that are responsible for the enzyme’s function. The joined synthetic enzyme segments are inhaled in a spray and trigger an immune reaction in saliva, preventing bacteria from binding to tooth surfaces. Unlike other vaccines under development that target young adults, the researchers want federal approval to vaccinate children at about 1 year of age. Vaccines will work best before bacteria take hold in the mouth, usually between 18 and 36 months of age, Taubman said. 1.

The author attributes the delay in developing a vaccine against tooth decay to I. A mindset that tooth decay is unavoidable. II. Difficulties pertaining to science. III. Researcher’s lack of interest. A. II only B. II and III only C. I and II only D. I and III only E. I, II and III

2.

If the assertions in the passage are true, dentists would most probably endorse a toothpaste with A. whitish colour B. fluoride compounds C. long shelf life D. cream base E. enzymes

3.

According to the research study reported in the passage, an oral spray against tooth decay should result in which of the following? A. It introduces glycosyl transferases enzymes into the cavities of a decayed tooth. B. It protects the teeth enamel by giving support which is usually absent due to calcium deficiency. C. It should act as an catalyst in reducing tooth cavities otherwise caused by increased consumption of sugar. D. It should prevent tooth diseases becoming chronic, especially if sprayed in an infant’s mouth. E. It can accelerate salival resistance to decay causing enzymes and remove bacteria from tooth.

137

Career Avenues

PASSAGE-2 Nearly 20 years ago, in speaking of her craft, the novelist Margaret Atwood observed that “a character in a book who is consistently well behaved probably spells disaster for the book. “She might have asserted the more general principle that consistent anything in a character can prove tedious. If we apply the old Forsterian standard, that round characters are the ones, “capable of surprising in a convincing way, “Atwood’s new novel, for all its multilayered story-within-a-story construction, must be judged flat as a pancake. In The Blind Assassin, overlong and badly written, our first impressions of the dramatis personae prove not so much lasting as total. The setup and setting are promising enough. This is chiefly the story of the Chase sisters Iris and Laura, grand-daughters of the benevolent founder of a button factory in the Canadian town of Port Ticonderoga. The girls grew up in the 1920’s in a large 19th century house named Avilion after the “island-valley” in “Idylis of the King”. After their war-wounded father exchanges his religious faith for disreputable rambles in Toronto and their mother dies of a miscarriage, the girls are more or less raised by Iris’s old nursemaid, Reenie, a starchy font of home truths and old saws. Iris is the older and more sensible of the two sister, Laura the more alluring and ungraspable. “Laura touches people”, Iris explains. “I do not”. With her capacity for sudden, passionate attachments to people and beliefs, the younger sister is a bafflingly nervy girl who gets away with things: “Laura had such a direct gaze, such blankly open eyes, such a pure, rounded forehead, that few ever suspected her of duplicity”. Iris is obliged to watch our for her, and on one occasion even stops Laura from drowning herself. This stagy moment, as Iris recalls it, allows for the stark juxtaposition of one girl’s self-destructiveness with the other’s suppressed resentment. “I couldn’t get out of my mind the image of Laura, in the icy black water of the Louveteau – how her hair had spread out like smoke in a swirling wind, how her wet face had gleamed silvery, how she had glared at me when I’d grabbed her by the coat. How hard it had been to hold on to her. How close I had come to letting go.” The Chase girl’s childhood – tutors, kitchen conversation, factory picnics – is played out at great length, and while social information should be more highly prized than it is in the modern literary novel, Atwood sometimes operates with the indiscriminate retrieval of an internet search engine set to “display all”. In the last decade, she has been able to keep things moving in her short stories (some of the stories in “Wilderness Tips” are terrifically, economical), but her novels have bogged down in repetition and clutter. The previous one, “Alias Grace,” reads like a tale told twice within itself. When the Depression lays siege to the economy, even Mr. Cahse’s paternalistic ways can’t save the button factory from labour unrest. The building is damaged by a fire, apparently set by Alex Thomas, an orphan, and ex-divinity student of shadowy origins, a prematurely hard-bitten figure whom the teenage Chase girls proceed to hide in the cellar and the attic. Here the novel most strongly exhibits its peculiar blend of the low and high, like a Nancy Drew story written by one of the Brontes. “I didn’t see him at first; he was behind the apple barrel. Then I could make him out. A knee, a foot, ‘it’s all right’, I whispered. “it’s only me”. Economic rescue is extended to the Chases by the heavy hand of Richard Griffen, a competing industrialist and arising right-wing politician who scorns the mollycoddling employment practices of the girls’ father. Eighteen year old Iris is presented to this cardboard villain like a fee and readied for the wedding by Richard’s brittle, ambitious sister, Winifred, the novel’s most appealingly awful creation, a sort of Miss Murdstone with plucked eyebrows in the event, the groom seems more interested in the willful 15-year-old Laura who goes to live with the Griffens after her father’s death but is soon running away or getting into trouble at school or being dispatched to an asylum. Laura dies – we learn this on page 1 – 1945, at the age of 25, when her car plunges off a Toronto bridge. She leaves behind a science fiction novel, The Blind assassin, which becomes a great posthumous success. Atwood’s enveloping novel of the same name alternates between the aged Iris’s narration of all the aforementioned events and extracts from this book, whose interplanetary matter is presented as a story being told by a hard-bitten fellow on the run to a young woman who steals hours with him in his various hideouts. Alex Thomas and Laura Chase, we presume. The less said about Planet, Zycron the better, Atwood, alas, says plenty. She lengthily taxonomizes the city of SakielNorn and its class-stratified inhabitants. (“the Snifards dress luxuriously and were connoisseurs of music. The smallholders, serfs and salves were called the Ygnirods”) But she proves unable to relieve the reader’s

138

Career Avenues

tedium with the place’s kinkier features, like children who weave carpets until they are blind and then go on to become throat-cutting hired killers. (Their sense of hearing was acute) Despite her assertion in a recent interview that this new book’s fantasy material has its roots in popular prewar genre magazines, Atwood seems to be sleepwalking through a Doris Lessing phase, or exercising once again her own allegorical bent, most famously displayed in “The Handmaid’s Tale”. 4.

Atwood’s characters in “The Blind Assassin” Iris and Laura can best be described as A. the selfish and the selfless B. the angelic and the crude C. contrast personas D. the dreamy and the pragmatic E. the idiosyncratic and the eclectic

5.

The author is convinced with which of the following elements of “The Blind Assassin”? A. Its sub-plots B. Its back drop C. Its narrative style D. Its story within the story E. Its metaphysical theme

6.

Which of the following types of characterization is Margaret Atwood most likely to admire in a novel? A. A character with a consistent positive image and quality about him. B. A character who, in the beginning, is an antihero and who reforms in the climactic scene. C. A character, who is a mirror image of the society he represents. D. A character, who is a prototype of the social class he represents. E. A character, who is an antithesis of his times.

7.

The author finds Atwood’s style of writing A. piercing and trenchant C. culture-centered and didactic E. indoctrinating and pedantic

8.

B. feminist and racial D. redundant and clogged

The critic’s attitude toward Atwood’s ‘The Blind Assassin’ is one of A. pointed criticism B. unbiased approval C. obvious indifference D. objective admiration E. scrupulous betrayal T

139

Career Avenues

READING COMPREHENSION ANSWER KEY Test 1 1-B

2-C

3-E

4-B

5-C

6-C

7-D

8-A

9-C

10-D

Test 2 1-D

2-E

3-A

4-C

5-C

6-C

7-A

8-D

Test 3 1-C

2-C

3-C

4-E

5-D

6-C

7-E

8-E

Test 4 1-A

2-C

3-B

4-D

5-C

6-C

7-E

2-D

3-A

4-C

5-C

6-E

7-D

8-B

2-D

3-C

4-E

5-C

6-D

7-C

8-A

Test 7 1-C 11-A

2-B

3-E

4-C

5-B

6-A

7-C

Test 8 1-D 11-E

2-B

3-A

4-E

5-C

6-B

2-E

3-B

4-D

5-E

2-B

3-B

4-C

2-D

3-C

2-B

2-C

Test 5 1-B 11-D Test 6 1-C

Test 9 1-A Test 10 1-C 11-D Test 11 1-B Test 12 1-D 11-A Test 13 1-C

11-B

9-D

9-B

10-E

8-A

9-C

10-D

7-A

8-D

9-C

10-B

6-C

7-B

8-B

5-D

6-C

7-B

8-A

9-B

10-A

4-C

5-B

6-B

7-E

8-A

3-A

4-C

5-D

6-A

7-C

8-B

9-A

10-D

3-C

4-D

5-C

6-B

7-E

8-D

9-C

10-A

140

Career Avenues

Test 14 1-D

2-D

3-E

4-A

5-D

6-B

7-A

8-C

9-C

10-D

Test 15 1-C 11-A

2-E 12-A

3-A 13-B

4-B

5-D

6-B

7-C

8-A

9-A

10-B

Test 16 1-C 11-B

2-A 12-B

3-B

4-B

5-C

6-A

7-B

8-B

9-D

10-A

Test 17 1-C 11-C

2-B 12-B

3-B

4-A

5-C

6-B

7-D

8-C

9-A

10-B

Test 18 1-D

2-B

3-A

4-B

5-C

6-D

7-B

8-A

Test 19 1-D

2-B

3-D

4-A

5-B

6-C

7-B

8-D

Test 20 1-B

2-C

3-D

4-B

5-C

6-A

7-A

2-A

3-C

4-A

5-C

6-B

7-C

8-A

9-D

10-A

2-B

3-E

4-3

5-2

6-2

7-4

8-1

Test 21 1-B 11-A Test 22 1-C

141

Career Avenues

CRITICAL REASONING

142

Career Avenues

Critical Reasoning A Critical Reasoning question is presented as a short paragraph, followed by one or two questions. The short paragraph is an argument. An argument consists of three parts: i) the premise – the evidence ii) the conclusion – the main point of the argument and iii) an assumption – the unstated or missing premises with out which the argument would be untenable. In certain cases the conclusion may not be given and the question may require that you supply the conclusion. The premises are pieces of evidence that the author uses to arrive at the conclusion. The conclusion is valid depending on the strength of the premises or evidences. Let’s look at an example to understand these terms better. Revenue from this magazine is at an all time low. Newsstand sales are dropping and there are no new subscribers. Therefore sweeping editorial changes are necessary if the magazine is to survive. Conclusion: Sweeping editorial changes are necessary for the magazine to survive. Premises:

Revenue down Newsstand sales low No new subscribers

Assumption: Only old editorial policy was responsible. Structural clues that help you locate a conclusion in an argument. Certain key words can help you isolate the conclusion and the evidence in a stimulus; clues that signal evidence includes - since, because, due to etc. Clues that signal conclusion include – so, therefore, thus, consequently, hence, as a result etc. Basic Approach Always look at the question first. The basic question types are:       

Find conclusion Supply conclusion Identify assumption Strengthen argument Weaken argument Find an inference Mimic argument

Find the conclusion In this question type the conclusion is already given. You have to choose from the choices the statement that is already given in the passage as the conclusion of the argument. Look out for key words like hence, so, thus, therefore etc. At the same time conclusion may also begin without these key words. Illustration When young students first look at modernist abstract painting, their eyes are assailed by a seemingly meaningless mass of squiggles. It is only after a study of the history of art and the forces which led up to abstraction that it is possible to appreciate the intellectual sophistication of modern art. Thus, a high-school study of modern art should always begin with a study of the history of art.

143

Career Avenues

Which of the following is the main point of the passage above? A. To understand the history of art, it is necessary to study modern art. B. Young students are unable to appreciate fully the complexities of modern art. C. An understanding of the history of art is essential to an understanding of modern art. D. To understand abstract art, students must first study the history of art. Analysis: Premise #1 : Students don’t understand modern art. Premise #2 : To understand modern art it is necessary to study art history Conclusion: Students of modern art must first study art history. Answers: A. Reverses P # 2

B. P # 1 C. P # 2 D. correct answer

Supply the conclusion In this question type the argument consists of merely a set of evidences. You have to supply the conclusion. This implies that the answer is not a sentence given in the passage but must include and rest on all the evidences. Illustration Fewer elected officials are supporting environmental legislation this year than at any time in the last decade. In a study of thirty elected officials, only five were actively campaigning for new environmental legislation. This comes at a time when the public’s concern for the environment is growing by leaps and bounds. Which of the following conclusions is best supported by the passage above? A. More elected officials are needed to support environmental legislation. B. Elected officials have lost touch with the concerns of the public. C. The five elected officials who actively campaigned for new environmental legislation should be congratulated. D. If the environment is to be saved, elected officials must support environmental legislation. Note: 1. Answer not a sentence from passage. 2. The answer must include and rest on all premises. Analysis Premise #1 : Fewer elected officials are supporting environmental legislation. Premise #2 : Public concern for the environment is growing. Answers: A. Ignores premise #2 B. Ignores environment C. Ignores public concern D. Correct answer. Find the assumption To answer this question, you have to identify the conclusion that is already given in the paragraph. An assumption is a statement that is not given as the premise but is required if the conclusion has to be valid. With out the assumption we cannot arrive at the conclusion. As the assumption is an unstated premise, the answer is not a statement given in the paragraph. Choose the choice that would connect to the conclusion as choice, therefore conclusion. i.e. Assumption = choice, therefore conclusion Illustration

144

Career Avenues

Many people believe that gold and platinum are the most valuable commodities. To the true entrepreneur, however gold and platinum are less valuable than opportunities that can enable him to further enrich himself. Therefore, in the world of high-finance, information is the most valuable commodity. The author of the passage above makes which of the following assumptions? A. Gold and Platinum are not the most valuable commodities. B. Entrepreneurs are not like most people. C. The value of information is incalculably high. D. Information about business opportunities is accurate and will lead to increased wealth. Approach: i. There could be several assumptions. ii. You are required to find the assumption on which the conclusion depends. Analysis Conclusion: In the world of high finance information is the most valuable commodity. Answers A. Irrelevant

B. Irrelevant

C. Same as conclusion

D. Correct answer.

In strengthen - weaken the argument, the type of premises given should be examined. The premises could be presented in the form of statistical data – numbers, percentages, ratios etc. causal data - that is the data may be in the form of cause- effect relationship, or analogy – arriving at a conclusion through likening it to a similar situation. Strengthen argument Here the conclusion is already given in the argument. One of the choices if true further supports or reinforces the conclusion. This means that the conclusion is further strengthened if a choice is true. The questions is phrased in one of the following ways: Which of the following, if true, strengthens the argument? Which of the following, if true, least weakens the argument? Which of the following, if true, adds credence to the argument? Which of the following, if true, reinforces the argument? Illustration It has recently been proposed that we adopt an all volunteer army. This policy was tried on a limited basis several years ago, and was a miserable failure. The level of education of the volunteers was unacceptably low, while levels of drug use and crime soared among army personnel. Can we trust our national defense to a volunteer army? The answer is clearly, “No”. Which of the following statements, if true, most strengthens the author’s claim that an all volunteer army should not be implemented? A. The general level of education has risen since the first time an all volunteer army was tried. B. The proposal was made by an organization called Citizens for Peace. C. The first attempt to create a volunteer army was carried out according to the same plan now under proposal and under the same conditions as those that exist today. D. A volunteer army would be less expensive than an army that relies on the draft. Approach: Look out for 3 types of logic. 1. Statistical

2. Analogy

3. Causal

Analysis: In the above argument the logic is analogy. “the idea did not work then, it won’t work now”.

145

Answers: A. Weakens

Career Avenues

B. Irrelevant

C. Correct answer

D. Irrelevant

Weaken the argument In this question type the conclusion is already in the argument but with a questionable assumption. One of the choices if true, weakens the conclusion. The question is phrased in one of the following ways. Which of the following, if true, weakens the conclusion? Which of the following, if true, least strengthen the argument? Which of the following, if true, jeopardizes the argument? Which of the following, if true, questions the validity of the argument? Which of the following, if true, damages the argument? Illustration The recent turn around of the LEX corporation is a splendid example of how an astute chief executive officer can re-channel a company’s assets toward profitability. With the new CEO at the helm, LEX has gone, in only three business quarters, from a 10 million dollar operating loss to a 22 million dollar operating gain. A major flaw in the reasoning of the passage above is that A. The passage assumes that the new CEO was the only factor that affected the corporation’s recent success. B. The recent success of the corporation may be only temporary. C. The chief executive officer may be drawing a salary and bonus that will set a damaging precedent for this and other corporations. D. the author does not define ‘profitability’. Approach : Look out for the type of logic 1. Statistical 2. Analogy

3. Causal

Analysis The logic is causal. Cause : CEO, Effect : Turnaround Answers A. Correct answer B. Irrelevant C. Irrelevant D. Irrelevant Find the inference In this question type the conclusion is already given. If the statements in the argument are true, one of the choices also must be true. If there are two premises it is possible for us to make two inferences. The questions appear as follows: If the statements in the argument are true, which of the following must also be true? Which of the following can be inferred from the passage? Which of the following is implied in the passage? Illustration In film and videotape, it is possible to induce viewers to project their feelings onto characters on the screen. In one study when a camera shot of a woman’s face was preceded by a shot of a baby in a crib, the audience thought the woman’s face was registering happiness. When the same shot of the woman’s face was preceded by a shot of a lion running toward the camera, the audience thought that the woman’s face was registering fear. Television news teams must be careful to avoid such manipulation of their viewers.

146

Career Avenues

Which of the following can be inferred from the passage? A. B. C. D.

Television news teams have abused their position of trust in the past. The expression on the woman’s face was, in actuality, blank. A camera shot of a baby in a crib provoked feelings of happiness in the audience. Audiences should strive to be less gullible.

Approach: 1. Inferences often have very little to do with conclusion. 2. For each premise you can have an inference. Answers: A. Outside scope B. Outside scope C. Correct answer D. Outside scope Another possible inference is A camera shot of a lion provoked feeling of fear in the audience. Mimic the argument: Here one of the choices is similar / parallel to the given argument in logic and structure. Illustration World class marathon runners do not run more than six miles a day when they are training. Therefore, if you run more than six miles a day, you are not world-class. Which of the following statements supports its conclusion in the same manner as the argument above? A. Sprinters always run in the morning. If it is morning, and you see someone running, it will not be a sprinter. B. Paint never dries in less than three hours. If it dries in less than three hours it is not paint. C. Little league games are more fun for the parents than for the children who actually play. Therefore, the parents should be made to play. D. If a car starts in the morning, chances are it will start again that evening. Our car always starts in the morning, and it always starts in the evening as well. Approach: Simply terms If A not B, if B not A Analysis A. If A, B. If B not A B. If A not B. If B not A C. If A, B not C. D. If A, B. A therefore B

147

Career Avenues

Exercise Questions: 20 1.

Time: 20 Minutes

Directions: Read each of the following passages carefully, and answer the questions that follow. High levels of pesticides, needed to produce high yields of the same crop year after year, pollute water supplies. Therefore, farmers are urged to diversify their crops and to rotate their plantings yearly. To receive governmental benefits, farmers must have produced the same crop for the past several years. Which of the following can be logically concluded from the above statements? A. The rules for governmental support of farm prices work against efforts to reduce water pollution. B. The only solution to the problem of water pollution from pesticides is to take farmland out of production. C. Farmers can continue to make profit by rotating diverse crops. D. New farming techniques will be developed to make it possible for farmers to reduce the use of pesticides.

2.

Low GDP growth does not cause high trade deficits. If it did, countries with the lowest GDP growth would also, have the highest trade deficits. When deficit figures are adjusted so that different countries are comparable, there is no such correlation. Which of the following can be properly inferred from the passage? A. Nations with low GDP growth tend to restrict foreign trade. B. Reliable comparisons between countries are impossible. C. Increasing GDP growth will not necessarily result in lowering of any trade deficit that country may have. D. Smallest countries generally have the least trade deficits.

3.

Which of the following best completes the passage below? The more worried investors are about loosing their money, the more they will demand a high potential return on their investment; great risks must be offset by the chance of great rewards. This principle is the fundamental one in determining interest rates and is illustrated by the fact that___________ A. successful investors are distinguished by an ability to make risky investments. B. lenders receive higher interest rates on unsecured loans than on loans backed by collateral. C. in times of high inflation, the interest paid to depository by banks can be below the rate of inflation. D. a bank will have a single rate of interest for all borrowers.

4.

In tests for AIDS, a false positive result indicates that people have AIDS, when, in fact, they do not; a false negative result indicates that people do not have AIDS, when, in fact, they do., To detect AIDS most accurately, doctors should use the laboratory test that has the lowest proportion of false positive results. Which of the following, if true, supports the suggestion above? A. The accepted treatment for AIDS does not have damaging side effects. B. In treating AIDS patients, it is essential to begin treatment as early as possible, since even a week of delay can be fatal. C The proportion of inconclusive test results is equal for all laboratory tests used to detect AIDS. D. All laboratory tests to detect AIDS have the same proportion of false negative results.

148

Career Avenues

5.

The program to control the entry of illegal drugs into India was a failure in 1991. If the program had been successful, the wholesale prices of most illegal drugs would not have dropped substantially. The observation in the passage depends on which of the following assumptions? A. The supply of illegal drugs dropped substantially in 1991. B. A drop in demand for most illegal drugs in 1991 was not the sole cause of the drop in their wholesale price. C. The wholesale price of a few illegal drugs increased substantially in 1991. D. The price paid for most illegal drugs by the average consumer did not drop much in 1991.

6.

In 1987 sinusitis was the most common chronic medical condition in India, followed by arthritis and blood pressure increase with age, but the incidence rate for sinusitis is the same for people of all ages. The average age of Indian population will increase between 1987 and 2000. Which of the following can be inferred from the passage? A. Sinusitis will be more common than either arthritis or blood pressure in 2000. B. Arthritis will be the most common chronic medical condition in 2000. C. The average age of people suffering from sinusitis will increase between 1987 and 2000. D. Fewer people will suffer from sinusitis in 2000 than in 1987.

7.

When a high-pressure front meets a low-pressure front, some form of precipitation usually occurs. Meteorologists are able to predict precipitation by measuring the speed at which two fronts are traveling towards one another and determining when they will meet. It can be inferred from the above passage that A. high pressure fronts are a sign of rain. B. the chief function of meteorologists is to predict precipitation. C. some predictions of precipitation are based on typical reactions of pressure fronts. D. low pressure fronts always travel in the direction of high pressure fronts.

8.

Studies show that there is hardly any difference between human beings and apes in their psychological capacities. The studies are, no doubt, inadequate. The argument above assumes that A. human beings and apes do not have highly developed psychological capacities. B. the studies on mammals are often misinterpreted. C. objectivity is nearly impossible in psychological studies. D. there is clear difference between the psychological capacities of human beings and apes.

9.

The mean per capita income for an individual was 3.5 percent higher in 1996 than in 1995.At the same time, average per capita income declined for individuals at the lower and middle income levels. Which of the following can be most logically inferred from the above? A. There were more individuals in 1996 than in 1995. B. There were fewer individuals at the upper income level in 1996 than in 1995. C. Average per capita income for individuals at the upper level rise by more than 3.5 percent between 1995 and 1996. D. Average per capita income for individuals was declining prior to 1995.

10. People who oppose the law that makes it mandatory for two wheeler riders to wear helmets argue that in a democracy, people have the right to take risks as long as they don’t harm others as a result of taking risks. They say that it should be each person’s decision whether or not to wear a helmet. Which of the following, if true, most weaken the argument above? A. Many new bikes that are available now are skid-resistant. B. Insurance rates are higher because of the need to pay for the injuries or death of people not wearing helmets. C. The rate of accidents in states with mandatory helmet laws is lower than in states with no laws. D. A great number of people die in road accidents even otherwise.

149

Career Avenues

11.

The cost of producing tillers in India is eight percent less than the cost of producing tillers in China. Even after tariff and transportation charges are added, it is still cheaper for China to import tillers from India than produce them. The above passage supports which of the following? A. Labour costs in India are eight percent below those in China. B. It takes eight percent less time to manufacture a tiller in India than it does in China. C. The tariff on a tiller imported from India to China is less than eight percent of the cost of producing the tiller in China. D. Importing tillers from India to China will eliminate eight percent of production jobs in China.

12.

Increases in the level of a certain hormone in the body lower stress-related disorders. Increase in the level of these hormones in some individuals can be brought about by a strict regimen of aerobics. Which of the following can be inferred from the passage above? A. People with fewer encounters of stress need not practice aerobics. B. Only a regimen of aerobics decreases stress-related disorders. C. Aerobics is the most promising method to reduce stress. D. A regimen of aerobics lowers stress-related disorders in some individuals.

13.

Which of the following best completes the passage? The elite buy status when they buy a designer product. They want to be seen in designer labels. In such cases mass marketing methods should not be used because_____________ A. the elite is concerned with quality as well as price of a product. B. continued sales depend mainly on retaining a halo of exclusivity. C. elite buyers represent only a small segment of customers. D. expansion of the market to include a broader range will help increase profits.

14.

Households in Chennai have more purchasing power than do Bangalore households at the same income level, as some of the income Bangalore households use for food and accommodation can be used by Chennai households for other needs. Which of the following inference can be drawn from the passage above? A. The mean income of Bangalore households is higher than that of Chennai. B. Chennai households include more people than those in Bangalore. C. Chennai households have lower food and accommodation costs than do Bangalore households. D. Bangalore households have more purchasing power than those of Chennai.

15.

Almost one out of four ABC students enter the top management schools every year and over half are absorbed by the second rung schools. Enroll for ABC course today! An aspirant who acts in accordance with the above advertisement, draws which of the following inferences? A. The students mentioned were helped to join top schools by ABC. B. ABC provides the most detailed and comprehensive training. C. Top management students cannot join a business school without ABC's intervention. D. The advertisement is placed where those who will be likely to read are management aspirants.

16.

Research in sciences and social sciences often complement each other. At the same time, both retain significant differences because of the basic questions each deals with. Sciences explain the causes of systems whereas social sciences explain the implications of actions. Which of the following is implied in the passage? A. The implications of actions do not explain why cause effects occur. B. The differences between life sciences and social sciences are decreasing. C. Implications of actions are not subject to scientific causes. D. Research in social sciences is not concerned with causes of systems.

150

Career Avenues

17.

We know that Anu is not well much of the time because whenever she calls us, she tells that she is home from office with a headache or cold. The above passage assumes that A. Anu's work cannot be done at home. B. Anu is not sick when she is at the office. C. Anu makes frequent calls. D. Anu talks for long on the phone.

18.

The invention of modern weapons involved creativity much more than that involved in the development of early crude weapons. Which of the following supports the statement above? A. Early weapons were discovered accidentally, rather than by application of creative knowledge. B. Necessity is the motivation for creativity. C. The needs of contemporary world are complex. D. Modern weapons are sophisticated but less durable.

19.

A film, to be successful at the box office, must satisfy the audience, by reflecting its values. Hit films of the 1960s, are indices to the typical tastes and values of their time. The author of the passage assumes that A. Movies of the '60s do not appeal to modern audience. B. Films are a superior form of creative art. C. 60's audiences represented the whole population of their time. D. Film viewers are people with distinct tastes.

20.

People who take drug x for obesity to reduce weight could end up defeating their purpose, since research shows that high levels of x may induce a craving for starch-based foods. Which of the following can be inferred from the passage? A. Drug x can be more dangerous than starch-based foods. B. People who do not take x are not likely to crave for starch foods. C. People tend to prefer foods high in starch. D. Weight gaining due to the intake of starch-based foods is significant.

21.

In the 1980s it was believed that those who study engineering have successful careers. Now, it is claimed that learning software is a sure way of career success. Currently the career market is flooded with engineers than it can absorb. It can be inferred from the analogy above that A. There will soon be more jobs for engineers than there are now. B. Software professionals will need services of engineers. C. There will soon be more software professionals than there are jobs for them. D. Software professionals and engineers will be competing for the same jobs.

22.

Poor resistance to common ailments in children who were breast-fed only for a few months is a common phenomenon. A positive correlation between duration of breast-feeding and children’s resistance to diseases discovered through an experimental study substantiates this observation. Which of the following least damages the author’s assertion? A. There is no relationship between the duration of breast-feeding and children’s health status. B. Symptoms of ailments do not appear until the child is six months old. C. The estimates of duration of breast-feeding and resistance to illness are gathered from a hospital. D. Infants who are breast-fed for shorter duration are generally weak and anemic.

151

Career Avenues

23.

The destruction of crops caused by the cyclone in Orissa is poised to result in a famine crisis. The minister for food and civil supplies claims that food production in unaffected districts slightly exceeds the amount of food required to provide a minimally adequate quota for its affected population. And adds that any crisis will result not from production but from distribution problems. The minister’s statement is based on the assumption that A. The state’s food production will continue to be sufficient to meet or exceed needs. B. The state’s food requirements are greater now than they will be in the future. C. The distribution of food supplies in the state would improve in the future. D. A crisis in food availability can be controlled by better distribution.

24.

Economists argue that India should slow down on its import of food grain and invest in irrigating vast stretches of desert lands to make them arable. Current estimates show that crop produced in these dry lands can feed the population for decades even if population and therefore consumption, grow at an accelerating rate. Which of the following, if true, most weakens the above argument? A. Importing food grains from outside will eliminate ten percent of farm labour. B. The cost of irrigating dry lands is ten percent more than the cost of importing food grains. C. The cost of irrigating dry lands is less than the cost of importing food grains from other countries. D. Labour costs in India are higher than in other countries for farm labour.

25.

Siddarth: I have tried Eve Lynwood techniques of speed-reading suggested by ABC: three techniques - underlining hand motion, vertical hand motion and Z-hand motion. When I tried these three methods, my concentration was lower than in the silent reading method. Therefore, these speed techniques do not work. Which of the following statements, if true, would most weaken Siddarth’s conclusion? A. Comprehension is better in silent reading than in loud reading. B. People practising speed techniques that ultimately work, often lose concentration while trying out. C. People who try different techniques of speed-reading cannot decide which one works and which one does not. D. People who try different speed techniques find that only one of them work.

152

Answer Key 1–A 2–C 11–C 12–D 21-C 22-D

Career Avenues

3–B 13–B 23-A

4–D 14–C 24-B

5–B 15–A 25-B

6–C 16–A

7–C 17–C

8–D 18–A

9–C 19–C

10–B 20–D

Explanatory Answers - Critical Reasoning 1. (A) It is evident from the passage that diversifying crops would not result in governmental support. 2. (C) If low GDP growth doesn't cause high trade deficits, the converse is also true. 3. (B) Expecting high returns on investment that is risk-ridden is illustrated in (B). 4. (D) The properties of false negative results is equal to that of the false positive tests. 5. (B) It is evident that factors other than a drop in demand for the illegal drugs may be responsible for the drop in wholesale price. 6. (C) If the average age of Indian population increases, the average age of people suffering from sinusitis will also increase. 7. (C) If precipitation can be predicted by measuring the speed at which the fronts are traveling it implies that typical reactions are used for prediction. 8. (D) If the studies are considered adequate, it is believed that there is a clear difference in psychological capacities of the two, which refutes the finding. 9. (C) When average per capita income declined for low and middle groups, per capital income for upper level has to rise if 3.5 percent increase was observed in 1996. 10. (B) The key statement is 'not harming others'. Any condition that possibly affects others works against the conclusion of the argument. 11. (C) If it is cheaper for China to import tillers from India, the expenses should be less than the cost of producing them. 12. (D) The generalisation is restricted to some individuals, therefore D. Other choices are outside the scope of the passage. 13. (B) When designer products are considered status symbols, exclusivity is the attraction, Mass marketing methods can hinder exclusivity. 14. (C) The argument that some of the income Bangalore households use for food and accommodation points to the fact that Chennai households spend less on these. 15. (A) Only A can be plausibly drawn from the argument. 16. (A) The significant differences between the two disciplines is given in only (A). 17. (C) The passage permits us to logically assume only (C). 18. (A) If a choice has to support the argument it should support the 'creativity' involved in the invention of modern weapons. 19. (C) If 60's films are indices to the values of that time, the audience might have represented the entire population. 20. (D) The key idea is 'defeating the purpose'. This implies that craving for starch-based foods is detrimental to the purpose - weight reduction. 21. (C) 22. (D) 23. (A) 24. (B) 25. (B)

View more...

Comments

Copyright ©2017 KUPDF Inc.
SUPPORT KUPDF